Straight Line

Download as pdf or txt
Download as pdf or txt
You are on page 1of 84

8.

STRAIGHT LINE

1. INTRODUCTION
Co-ordinate geometry is the branch of mathematics which includes the study of different curves and figures by
ordered pairs of real numbers called Cartesian co-ordinates, representing lines & curves by algebraic equation. This
mathematical model is used in solving real world problems.

2. CO-ORDINATE SYSTEM
Co-ordinate system is nothing but a reference system designed to locate position of any point or geometric
element in a plane of space.

2.1 Cartesian Co-ordinates


Let us consider two perpendicular straight lines XOX’ and YOY’ passing through the origin
y
O in the plane. Then, =90
x
Axis of x: The horizontal line xox’ is called axis of x. x’ O

Axis of y: The vertical line yoy’ is called axis of y. y’

Co-ordinate axis: x-axis and y-axis together are called axis of co-ordinates or axis of Figure 8.1
reference.
Origin: The point ‘O’ is called the origin of co-ordinates or just the origin.
Oblique axis: When xox’ and yoy’ are not at right angle, i.e. if the both axes are not perpendicular, to each other,
then axis of co-ordinates are called oblique axis.

2.2 Co-ordinate of a Point


The ordered pair of perpendicular distances of a point from X- and Y-axes are called co-ordinates of that point.
If the perpendicular algebraic distance of a point p from y-axis is x and from x-axis is y, then co-ordinates of the
point P is (x, y). Here,
(a) x is called x-co-ordinate or abscissa.
(b) y is called y-co-ordinate or ordinate.
(c) x-co-ordinate of every point lying upon y-axis is zero.
(d) y-co-ordinate of every point lying upon x-axis is zero.
(e) Co-ordinates of origin are (0, 0).
Note: A point whose abscissa and ordinate are both integers is known as lattice point.
8 . 2 | Straight Line

2.3 Polar Co-ordinates y


P (r ,)
Let OX be any fixed line, known as initial line, and O be the origin. If the distance of any
point P from the origin O is ‘r’ and ∠XOP = θ, then (r, θ) are known as polar co-ordinates r
of point P. If (x, y) are the Cartesian co-ordinates of a point P, then x = rcos θ; y = rsin θ and

x
y
| r | = x2 + y 2 , θ = tan–1   θ ∈ ( −π , π )
O
x Figure 8.2

Illustration 1: If the Cartesian co-ordinates of any point are ( 3, 1) , find the polar co-ordinates.  (JEE MAIN)
y
Sol: Polar co-ordinates of any point are (r, θ), where r = x2 + y 2 and θ = tan–1   .
x
x= 3;y=1
Let their polar co-ordinates be (r, θ) ⇒ x = r cos θ; y = r sinθ

So r ⇒ x2 + y 2 r = 3+1

y  1  π
θ ⇒ tan−1   = 2 θ ⇒ tan−1  =
x  3 6
 π
∴(r, θ) =  2 ,  .
 6
y
3. DISTANCE FORMULA
Q(x2,y2)
B(r2,2)
d
The distance between two points P(x1, y1) and Q(x2, y2) is ,y 1)
P(x 1
PQ = (x1 – x2 )2 + (y1 – y 2 )2 = (x2 − x1 )2 + (y 2 − y1 )2 (r1,1) A
x2 - x1
Distance of a point P(x1, y1) from the origin O(0, 0) is
O x2
OP = 2
X +y 2
1 1
Figure 8.3
Distance between two polar co-ordinates A(r1, q1) and B(r2, q2) is
given by
AB = r12 + r22 − 2r1r2 cos(θ1 − θ2 )

Proof: AB = (x2 − x1 )2 + (y 2 − y1 )2 x1 = r1 cos q1, x2 = r2 cos q2, y1 = r1sin q1, y2 = r2sinq2

AB = (r2 cos θ2 − r1 cos θ1 )2 + (r2 sin θ2 − r1 sin θ1 )2

AB = (r2 cos θ2 )2 − 2r1r2 cos θ1 cos θ2 + (r1 cos θ1 )2 + (r2 sin θ2 )2 − 2r1r2 sin θ1 sin θ2 + (r1 sin θ1 )2

AB = r12 + r22 − 2r1r2 cos(θ1 − θ2 )

MASTERJEE CONCEPTS
Distance between two polar co-ordinates A(r1, q1) and B(r2, q2) is given by
AB = r12 + r22 − 2r1r2 cos(θ1 − θ2 )
Vaibhav Krishnan (JEE 2009, AIR 22)
M a them a tics | 8.3

 π  π
Illustration 2: Find the distance between P  2, −  and Q  3,  .  (JEE MAIN)
 6  6

Sol: The distance between two points = r12 + r22 − 2r1r2 cos(θ1 − θ2 ) Therefore,
.
 π π π 1
PQ = r12 + r22 − 2r1r2 cos(θ1 − θ2 ) = 4 + 9 − 2.2.3cos  − −  = = 4 + 9 − 12cos   = 13 − 12. = 7
 6 6 3 2

Illustration 3: The point whose abscissa is equal to its ordinate and which is equidistant from the points A(1, 0),
B(0, 3) is  (JEE MAIN)

Sol: Given, abscissa = ordinate. Therefore distance can be found by considering the co-ordinates of required point
be P(k, k).
Now given PA = PB ⇒ (k − 1)2 + k 2 = k 2 + (k − 3)2
2k2 – 2k + 1 = 2k2 – 6k + 9 ⇒ 4k = 8 ⇒ k = 2

4. SECTION FORMULA
Let R divide the two points P(x1, y1) and Q(x2, y2) internally in the ratio m:n.
Let (x, y) be the co-ordinates of R.
y
Draw PM, QN, RK perpendicular to the x-axis. Q
R
F
Also, draw PE and RF perpendicular to RK and QN. P E

PR m
Here, = .
RQ n
x’ O M K N x
Triangles PRE and RFQ are similar.
PR PE PE m y’
∴ = ⇒ =
RQ RF RF n
Figure 8.4
But PE = x – x1 and RF = x2 – x.

x − x1 m mx2 + nx1
∴ = ⇒ x=
x2 − x n m+n
ER m
In the same way, =
FQ n
y − y1 m my 2 + ny1  mx2 + nx1 my 2 + ny1 
i.e., = ⇒y= The co-ordinates of R are  , 
y2 − y n m+n  m+n m + n 
PR ' m
If R’ divides PQ externally, so that = , triangles PER’ and QR’F are similar. y
QR ' n
PR ' PE F R’
∴ =
R 'Q R 'F Q
P E
But PE = x – x1 and R’F = x – x2.
x
x − x1
m mx2 − nx1 x’ O M N K

= i.e., x =
x − x2 n m−n
my 2 − ny1 y’
Similarly, y = .
m−n
Figure 8.5
 mx − nx1 my 2 − ny1 
The co-ordinates of R’ are  2 , 
 m−n m − n 
8 . 4 | Straight Line

PR ' m m
Alternate Method: =− = By changing n into –n in the co-ordinates of R, we can obtain the co-ordinates
R 'Q n –n
of R’:

mx2 − nx1 my 2 − ny1


,
m−n m−n
 x + x 2 y1 + y 2 
Cor. The mid-point joining the two points (x1, y1) and (x2, y2) is  1 , 
 2 2  .

 x + λx2 y1 + λy 2 
Cor. From the above cor., the co-ordinates of a point dividing PQ in the ratio λ:1 are  1 ,  . Considering
 1+ λ 1+ λ 
λ as a variable parameter, i.e. of all values positive or negative, the co-ordinates of any point on the line joining the

points (x1, y1) and (x2, y2) can be expressed in the above forms.

5. SPECIAL POINTS OF A TRIANGLE

5.1 Centroid C(x3, y3)

Let the vertices of the triangle ABC be (x1, y1), (x2, y2) and (x3, y3), respectively. F 1 D
 x + x3 y 2 + y 3  2 G
The mid-point D of BC is  2 ,  G, the centroid, divides AD internally
2 2 
in the ratio 2:1.  (x1, y1)A E B(x2, y2)

Let G be (x, y), Figure 8.6

2. ( (x2 + x3 ) / 2 ) + 1.x1 x1 + x2 + x3
then x = = and
2+1 3
2. ( (y 2 + y 3 ) / 2 ) + 1.y1 y1 + y 2 + y 3  x + x 2 + x3 y1 + y 2 + y 3 
y= = ∴ G is  1 ,  .
2+1 3  3 3 

5.2 Incentre
Let A (x1, y1), B (x2, y2), C (x3, y3) be the vertices of the triangle.
Let AD bisect angle BAC and cut BC at D.
A
BD AB c
We know that = =
DC AC b
cx3 + bx2 cy 3 + by 2 I
Hence the co-ordinates of D are , C
c+b c+b
D
Let (x, y) be the incentre of the triangle
B
CD b BC b + c ca AI AB c b+c
= ∴ = ∴ BD = = = = Figure 8.7
BD c DB c b + c ID BD ( ca / (b + c) ) a
(b + c) ( (cx3 + bx2 ) / (c + b) ) + ax1 ax1 + bx2 + cx3
= x = ,
b+c+a a+b+c
(b + c) ( (cx3 + bx2 ) / (c + b) ) + ax1 ax1 + bx2 + cx3 (b + c) ( (cy 3 + by 2 ) / (c+ b) ) + ay1 ay1 + by 2 + cy 3
∴ x = = , y =
b+c+a a+b+c b+c+a a+b+c
(b + c) ( (cy 3 + by 2 ) / (c+ b) ) + ay1 ay1 + by 2 + cy 3
y =
b+c+a a+b+c
M a them a tics | 8.5

5.3 Ex-centres
A
The centre of the circle which touches the side BC and the extended portions of sides I3 I2
AB and AC is called the ex-centre of ∆ABC with respect to the vertex A. It is denoted by
I1 and its co-ordinates are as follows: B C

 −ax1 + bx2 + cx3 −ay1 + by 2 + cy 3 


I1 =  , 
 −a + b + c −a + b + c 
I1

Similarly ex-centres of ∆ABC with respect to vertices B and C are denoted by I2 and I3,
respectively, and Figure 8.8

 ax – bx2 + cx3 ay1 − by 2 + cy 3 


I2 =  1 ,  ,
 a−b + c a−b + c 

 ax + bx2 − cx3 ay1 + by 2 − cy 3 


I3 =  1 ,  .
 a+b−c a+b−c
 

5.4 Circumcentre A(x1, y1)

It is the point of intersection of perpendicular bisectors of the sides of the triangle.


It is also the centre of a circle passing through the vertices of the triangle. If O is the
O
circumcentre of any ∆ABC, then, OA = OB = OC.
B E C(x3, y3)
 x sin2A + x2 sin2B + x3 sin2C y1 sin2A + y 2 sin2B + y 3 sin2C 
Circumcentre:  1 ,  (x2, y2)
 Σ sin2A Σ sin2A 
Figure 8.9
Note: For a right-angled triangle, its circumcentre is the mid-point of hypotenuse.

A C

Figure 8.10
A(x1, y1)
5.5 Orthocentre
The point of intersection of altitudes of a triangle that can be obtained by solving the D F
equation of any two altitudes is called Orthocentre. It is denoted by H
H

 x tanA + x2 tanB + x3 tanC y1 tanA + y 2 tanB + y 3 tanC  B E C


Orthocentre:  1 , 
(x2, y2) (x3, y3)
 Σ tanA Σ tanA  Figure 8.11
Note: In a right angle triangle, orthocentre is the point where right angle is formed.
A
Remarks:
(a) In an equilateral triangle, centroid, incentre, orthocentre, circumcentre coincide.
G H
(b) Orthocentre, centroid, and circumcentre are always collinear. Centroid divides the O
Orthocentre and circumcentre joining line in a 2: 1 ratio.
B C
Proof: H, G and O are collinear and ∆’s OGD & AGH are similar. D N

But OD (distance of c.c. from BC) = R cos A Figure 8.12


8 . 6 | Straight Line

HA = distance of orthocentre from vertex A = 2R cos A

AH AG HG
∴ =2= = ⇒ G divides line joining H and O in 2:1.
OD GD GO
(c) In an isosceles triangle centroid, orthocentre, incentre, circumcentre lie on the same line.

5.6 Nine-Point Circle


Nine-point circle can be constructed for any given triangle, and is so named because it touches nine significant
concyclic points throughout the triangle.
These nine points are as follows:
•• Mid-point of each side of the triangle
•• Foot of each altitude
•• Mid-point of the line segment from each vertex of the triangle to the orthocentre.

Figure 8.13

MASTERJEE CONCEPTS

•• The centroid, incentre, orthocentre and circumcentre coincide in an equilateral triangle.


•• In an isosceles triangle, centroid, orthocentre, incentre and circumcentre lie on the same line.
•• Orthocentre, centroid and circumcentre are always collinear, and centroid divides the line joining
orthocentre and circumcentre in the ratio 2:1.

Saurabh Gupta (JEE 2010, AIR 443)

Illustration 4: If G be the centroid of the triangle ABC, prove that AB2 + BC2 + CA2 = 3(GA2 + GB2 + GC2).
 (JEE MAIN)

Sol: Distance formula of two points can be used to prove AB2 + BC2 + CA2 = 3(GA2 + GB2 + GC2).
In triangle ABC, let B be the origin and BC the x-axis. Let A be (h, k) and

a+h k  A(h, k)
C be (a, 0). Then centroid G is  , .
 3 3
LHS
= AB2 + BC2 + CA2 = (h – 0)2 + (k – 0)2 + a2 + (h – a)2 + (k – 0)2
C (a, 0)
= 2h2 + 2k2 + 2a2 – 2ah B

Figure 8.14
M a them a tics | 8.7

∴ RHS
 a + h 2
 k
2
 a+h
2
 k
2
 a+h
2
 k 
= 3  − h +  − k  +  − 0 +  − 0 +  − a  +  − 0 
 3  3   3  3   3  3  

= 1/3 [(a – 2h)2 + 4k2 + (a + h)2 + k2 + (h – 2a)2 + k2] = 2h2 + 2k2 + 2a2 – 2ah
Hence, it is equal on both sides.
C
y
5.7 Area of a Triangle
Let A, B, C be the vertices of the triangle having (x1, y1), (x2, y2) and (x3, y3) as their B

respective co-ordinates. Draw AL, BM, CN perpendicular to the x-axis. A

Then ∆ABC = trapezium ALNC + trapezium CNMB – trapezium ALMB

1 1 1
= (LA + NC) LN + (NC + MB) NM – (LA + MB) LM
2 2 2 x
1 1 1 O L N M
= (y1 + y3) (x3 – x1) + (y3 + y2) (x2 – x3) – (y1 + y2) (x2 – x1)
2 2 2
Figure 8.15
1
= {x1(y2 – y3) + x2(y3 – y1) + x3(y1 – y2)}
2
x1 y1 1
1
= x2 y2 1
2
x3 y3 1

If the area of the triangle formed by the points (x1, y1), (x2, y2) and (x3, y3) is zero, the points lie on a straight line.
Using this, we can determine whether three point are in a straight line. i.e. the condition for (x1, y1), (x2, y2) and (x3,

x1 y1 1
y3) to be collinear is that x2 y 2 1 = 0.
x3 y3 1
1
For example, the area of the triangle formed by the points (1, 4), (3, –2) and (–3, 16) is {1(–2 –16) + 3} {(16 – 4) – 3
(4 + 2)} = 0. The three points lie on a straight line. 2

Illustration 5: The vertices of a triangle ABC are A(p2, –p), B(q2, q) and C(r2, –r). Find the area of the triangle.
 (JEE MAIN)
x1 y1 1
1
Sol: Area of the triangle formed by the points (x1, y1), (x2, y2) and (x3, y3) is x2 y 2 1 . Substituting the given
co-ordinates, we can obtain area of given triangle. 2
x y 1
3 3

p2 −p 1 p2 − q2 −(p + q) 0 p − q −1 0
1 2 1 2 2 1
D= q q 1 = q −r q+r 0 = (p + q) (q + r) q − r 1 0
2 2 2 2
r −r 1 r2 −r 1 r2 −r 1

1 1
= (p + q) (q + r) [(p – q) + (q – r)] = (p + q) (q + r) (p – r)
2 2
8 . 8 | Straight Line

Note:
(a) If area of the triangle is zero, then the three points are collinear.
(b) The area of a polygon with vertices A1(xi, yi), i = 1, …, n (vertices taken in anti-clockwise order)
1
[(x y – x2y1) + (x2y3 – x3y2) +…+(xny1 – x1yn)]
2 1 2

6. LOCUS
Locus is a set of points which satisfies a given geometrical data. Thus, for example, locus of a point moving at a
constant distance from a given point is a circle. Locus of a point which is equidistance from two fixed points is a
perpendicular bisector of the line joining the two points.
All the points in a locus can be represented by an equation. For example,
(a) If the distance of the point (x, y) from (2, 3) is 4, then
(x – 2)2 + (y – 3)2 = 42.
i.e. x2 + y2 – 4x – 6y – 3 = 0.
This equation will represent a circle with its centre at (2, 3) and radius 4.

(b) If (x, y) be the point equidistant from the points (3, 4) and (2, 1), then
(x – 3)2 + (y – 4)2 = (x – 2)2 + (y – 1)2
i.e. x + 3y = 10.
From the geometrical constraint, which governs the motion, we can find a relation (locus) between the co-
ordinates of the moving point in any of its positions. Equation of locus is therefore merely on equation
relating the x and y co-ordinates of every point on the locus.

Steps to find locus


(i) Assume the co-ordinates of point for which locus is to be determined as (h, k).
(ii) Apply the given geometrical conditions.
(iii) Transform the geometrical conditions into algebraic equation and simplify.
(iv) Eliminate variables (if any).
(v) Replace h → x and k → y to get the equation of locus.

Note:
•• Locus should not contain any other variables except x and y.
•• The algebraic relation between x and y satisfied by the co-ordinates at every point
on the curve and not off the curve is called the equation of curve.

y
Illustration 6: Find the equation of locus of a point which moves so that its distance P(x, y)
(0, 1)
from the point (0, 1) is twice the distance from x-axis. (JEE MAIN)
N
Sol: Here we can obtain the equation of locus of given point by using given condition
and distance formula of two points.
x
Let the co-ordinates of such a point be (x, y). Draw PM ⊥ to x-axis. x’ 0 M
y’
Hence, PM = y
Figure 8.16
PN = 2PM (given)
M a them a tics | 8.9

i.e. (x – 0)2 + (y – 1)2 = 4y2


i.e. x2 – 3y2 – 2y + 1 = 0.

Illustration 7: Locus of the centroid of the triangle whose vertices are (a cos t, a sin t), (b sin t, – b cos t) and (1, 0),
where t is a parameter is  (JEE MAIN)
(A) (3x – 1)2 + (3y)2 = a2 + b2 (B) (3x + 1)2 + (3y)2 = a2 + b2

(C) (3x + 1)2 + (3y)2 = a2 – b2 (D) (3x – 1)2 + (3y)2 = a2 – b2

Sol: The centroid (h, k) of a triangle formed by points (x1, y1), (x2, y2) and (x3, y3) will be
x1 + x2 + x3 y1 + y 2 + y 3
h = and k .
3 3
(A) If (h, k) is the centroid, then

acos t + bsint + 1 asint − bcos t + 0


h= ,k= ⇒ (3h – 1)2 + (3k)2 = (a cos t + b sin t)2 + (a sint t – b cos t)2 = a2 + b2
3 3
∴ Locus of (h, k) is (3x – 1)2 + (3y)2 = a2 + b2

7. STRAIGHT LINE
Definition: It is defined as the locus of a point such that any two points of this locus have a Y
constant inclination (gradient).
Inclination: If a straight line intersects the x-axis, the inclination of the line is defined

as the measure of the smallest non-negative angle which the line makes with the X’
positive direction of the x-axis 180- 
 π
Slope (or gradient): If the inclination of a line (i.e. non-vertical line) is θ and  θ ≠  ,
then the slope of a line is defined to be tan θ and is denoted by m.  2
Y’
∴ m = tanq Figure 8.17
(a) Slope of x-axis is zero.
(b) Slope of y-axis is not defined.

7.1 Slope
(x2, y2)Q l
Let P(x1, y1) and Q(x2, y2) be two points on a line, then slope will be
y2 - y1
(x1, y1)P
MQ M
m
= tan=
θ x2 - x 1
MP
y 2 − y1 
m
= tan=
θ
x2 − x1 O
x y 1
Figure 8.18
(a) Line passing through two points (x1, y1) and (x2, y2) is x1 y1 1 = 0
x2 y2 1

Note: Above-mentioned matrix form is a condition for three points to be collinear. (x1, y1) (x2, y2)
(b) Equation of the median through A(x1, y1) is
Figure 8.19
8 . 1 0 | Straight Line

A(x1, y1)

P(x, y)

B C
M (x3, y3)
(x2, y2)
( x2 +x3 y2 +y3
2
,
2 )
Figure 8.20

x y 1 x y 1 x y 1
x1 y1 1 = 0 or x1 y1 1 + x1 y1 1 = 0
x 2 + x3 y 2 + y3 x2 y2 1 x3 y3 1
1
2 2

(c) Equation of internal and external angle bisectors of A are

A(x1, y1)
x y 1 x y 1
b x1 y1 1 ± c x1 y1 1 =
0

P(x, y)
x2 y2 1 x3 y3 1
P(x, y)

(x2, y2)B b E
D C(x3, y3)

( bx2 + cx3 by2 + cy3


b+c
,
b+c ) ( bx2 - cx3 by2 - cy3
b-c
,
b-c )
Figure 8.21

7.2 Angle between Two Lines


Two lines intersecting each other make two angles between them, one acute and the other obtuse. Figure 8.22
shows lines L1 and L2 intersecting each other, acute angle θ and obtuse angle φ.
Let line L1 makes angle q1 with x-axis and L2 makes q2.

y
L2

L1




1 -2
A x
O B

Figure 8.22
M a them a ti cs | 8.11

Therefore slope of L1 is m1= tan q1


Slope of L2 is m2 = tanq2
Now in ∆ABC,
q1 + π – q2 + θ = π
θ = θ2 – θ1
tan θ2 − tan θ1 m2 − m1
tanθ = tan (q2 – q1) ⇒ tanθ = ⇒ tan θ = , this gives the acute angle between lines.
1 + tan θ2 . tan θ1 1 + m1m2
Note:
(i) If m1 = m2, then θ = 0º, i.e. lines are parallel or coincident.
(ii) If m1m2 = −1, then θ = 90 º , i.e. lines are perpendicular to each other.

π 1
Illustration 8: If the angle between two lines is and slope of one of the lines is . Find the slope of the other.
 4 2 (JEE MAIN)
m2 – m1
Sol: We know that, tan θ = , where m1 and m2 are the slope of lines and θ is the angle between them.
1+m1m2

1 π π m – (1 / 2) m – (1 / 2)
Let m1 = , m2 = m and θ = So, tan = ⇒ 1 =± ⇒ m = 3 or –(1/3)
2 4 4 1+(1 / 2)m 1+(1 / 2)m

Illustration 9: Line through the point (–2, 6) and (4, 8) is perpendicular to the line through the point (8, 12) and
(x, 24). Find the value of x.  (JEE MAIN)

Sol: Given two lines are perpendicular to each other. Therefore, product of their slope will be -1.
8–6 2 1
Slope of the line through the points (–2, 6) and (4, 8) is m1= = =
4 – (–2) 6 3
24 – 12 12
Slope of the line through the points (8, 12) and (x, 24) is m2 = =
x–8 x–8
Since two lines are perpendicular m1m2 = – 1
1 12
⇒ × = –1 ⇒x=4
3 x–8

7.3 Collinearity

C
B

Figure 8.23

If three points A, B, C are collinear, then


Slope of AB = Slope of BC = slope of AC
8 . 1 2 | Straight Line

MASTERJEE CONCEPTS

Collinearity of three given points:


Three given points A, B, C are collinear if any one of the following conditions is satisfied.
•• Area of triangle ABC is zero.
•• Slope of AB = Slope of BC = Slope of AC.
•• AC = AB + BC.
•• Find the equation of the line passing through two given points, if the third point satisfies the equation of
the line, then three points are collinear.
1
If any one line is parallel to y-axis, then the angle between two straight lines is given by tan θ = ± ,
where m is the slope of other straight line. m

A line of gradient m is equally inclined with the two lines of gradient m1 and m2.
m1 – m m2 – m
Then =- .
1+m1m 1+m2m
Aman Gour (JEE 2012, AIR 230)

7.4 Equation of a line


(a) Point slope form: Suppose P0(x0, y0) is a fixed point on a non-vertical line L whose P(x, y)
slope is m. Let P(x, y) be an arbitrary point on L. Then by definition, the slope of L
y – y0 P0(x0, y0)
is given by m = ⇒ y – y 0 = m(x – x0 )
x – x0
This is called point slope form of a line.
Figure 8.24
(b) Two point form: Let line L passes through two given points P1(x1, y1) and P2(x2, y2).
Let P(x, y) be a point on the line. So slope P1P = slope P1P2
y – y1 y 2 – y1
⇒ = P2(x2, y2)
x – x1 x2 – x1
P(x, y)
y 2 – y1
⇒ y – y1 = (x – x1 ) P1(x1, y1)
x2 – x1

This is called two-point form of the line.


Figure 8.25

(c) Slope intercept from: Case-I: If slope of line is m and makes y-intercept c, then equation is
(y – c) = m (x – 0) ⇒ y = mx + c
Case-II: If slope of line is m and makes x-intercept d, then equation is
y = m(x – d)
These equations are called slope intercept form. (0, c)

(d, 0)

Figure 8.26
Mathematics | 8.13

(d) Intercept form: Suppose a line L makes intercept a on x-axis and intercept b on L
y-axis, i.e. the line meets x-axis at (a, 0) and y-axis (0, b). (0, b)

b–0
So, y – 0 = (x – a) b
0–a
x y
i.e. + = 1 . This is called intercept form of the line.
a b
a (a, 0)

Figure 8.27

(e) Normal form: If P is perpendicular distance from origin to the line AB and makes angle α with x-axis, then
equation of the line is x cos α + y sin α = P
OM y
Proof: cos α = OM = OL cos α = x cos α
OL
B
In DPNL,
PN
sin α =
PL
Q
PN = PL sin α = y sin α M P(x,y)
90-
MQ = PN = y sin α  N 
x
O x A
Now P = OQ = OM + MQ = x cos α + y sin α L
Figure 8.28
So x cos α + y sin α = P

This is called normal form of the line.

(f) Parametric form or distance form: The equation of the line passing through (x1, y1) and making an angle θ
with the positive x-axis is
x – x1 y – y1
= = r , where ‘r’ is the signed value.
cos θ sin θ

Hence, the co-ordinate of any point at a distance r on this line is

x = x1 + r cos θ y = y1 + r sin θ

MASTERJEE CONCEPTS

Point of intersection of two lines a1x + b1y + c1 = 0 and a2x + b2y + c2 = 0 is given by
 b1 b2 c1 c2 
 
 b1c2 – b2c1 c1a2 – c2a1   c1 c2 a1 a2 
( x', y')=  a b – a b , a b – a b  =  a1 a2
,
a1 a2 
 1 2 2 1 1 1 2 1  
 b1 b2 b1 b2 

Saurabh Gupta (JEE 2010, AIR 443)


8 . 1 4 | Straight Line

Illustration 10: A straight line is drawn through the point P(2, 3) and is inclined at an angle of 30° with positive
x-axis. Find the co-ordinate of two points on it at a distance 4 from P on either side of P.  (JEE MAIN)
x – x1 y – y1
Sol: By using formula = = r , we can obtain co-ordinates of point.
cos θ sinθ
The equation of line
x – x1 y – y1 x–2 y –3
= = ±r ⇒ = = ± 4 ⇒ x = 2 ± 2 3, y = 3 ± 2
cos θ sinθ cos 30° sin 30°

So, co-ordinate of two points are 2± 2 3 ,3± 2 ( )


Illustration 11: If two vertices of a triangle are (–2, 3) and (5, –1). Orthocentre of the triangle lies at the origin and
centroid on the line x + y = 7, then the third vertex lies at  (JEE MAIN)
(A) (7, 4) (B) (8, 14)
(C) (12, 21) (D) None of these

Sol: (D) The line passing through the third vertex and orthocentre must be perpendicular to line through (–2, 3)
and (5, –1). Therefore, product of their slope will be -1.
Given the two vertices B(–2, 3) and C(5, – 1); let H(0, 0) be the orthocentre; A(h, k) the third vertex.
Then, the slope of the line through A and H is k/h, while the line through B and C has the slope
(–1 –3)/(5 + 2) = – 4/7. By the property of the orthocentre, these two lines must be perpendicular,
 k  4  k 7
So we have    −  = – 1 ⇒ =  … (i)
h
  7  h 4

5 − 2 + h −1 + 3 + k
Also + = 7 ⇒ h + k = 16 … (ii)
3 3
Which is not satisfied by the points given in (A), (B) or (C).

Illustration 12: In what direction should a line be drawn passing through point (1, 2) so that its intersection point
6
with line x + y = 4 is at a distance of units.  (JEE ADVANCED)
3
Sol: By using x = x1 + r cos θ and y = y1 + r sin θ, we can obtain the required angle.
For co-ordinates of B

6 6 6
Substitute r= ∴ x = 1+ cos θ & =
y 2+ sin θ B
3 3 3
A(1,2)
Substituting in x + y =4 6
 3
6 6 3
⇒ 1+ cos θ + 2+ sin θ = 4 ∴ (cos θ + sin θ) =
3 3 6
1 1 3 1
∴ cos θ + sin θ = (Multiple by )
2 2 2 2 Figure 8.29

sin (45° + θ) = sin 60°


∴ θ = 15°
or, sin (45° + θ)= sin 120° ∴ θ = 75°
Mathematics | 8.15

Illustration 13: If sum of the distances of the points from two perpendicular lines in a plane is 1, then find its locus.
(JEE ADVANCED)
Sol: If (h, k) be any point on the locus, then |h| + |k| = 1
Y
Let the two perpendicular lines be taken as the co-ordinate axes.
⇒ locus of (h, k) is |x| + |y| = 1 (0, 1)

This consists of four line segments which enclose a square as -1

x
y=

+
y
x-
shown in figure.

=1
X
 (-1, 0) O (1, 0)

x
=1

+
y

y
x-

=1
(0, -1)

Figure 8.30

Illustration 14: If the circumcentre of a triangle lies at the origin and the centroid is the mid-point of the line
joining the points (a2 + 1, a2 + 1) and (2a, –2a), then the orthocentre lies on the line.  (JEE ADVANCED)
(A) y = (a2 + 1)x (B) y = 2ax (C) x + y = 0 (D) (a – 1)2 x – (a + 1)2 y = 0

Sol: (D) We know from geometry that the circumcentre, centroid and orthocentre of a triangle lie on a line. So the
 (a + 1)2 (a − 1)2 
orthocentre of the triangle lies on the line joining the circumcentre (0, 0) and the centroid  , 
 2 2 
2 2 
(a + 1) (a − 1)
y= x or (a – 1)2 x – (a + 1)2 y = 0.
2 2

MASTERJEE CONCEPTS

Equation of parallel and perpendicular lines:


• Equation of a line which is parallel to ax + by + c = 0 is ax + by + k = 0.
• Equation of a line which is perpendicular to ax + by + c = 0 is bx – ay + k = 0.
• If y = m1x + c1, y = m1x + c2, y = m2x + d1 and y = m2x + d2 are sides of a parallelogram then its
(c1 – c2 )(d1 – d2 )
area is .
m1 – m2
x y
• The equation of a line whose mid-point is (x1, y1) in between the axes is + =2.
x1 y1
c2
• Area of the triangle made by the line ax + by +c = 0 with the co-ordinate axes is .
2 | ab |

• A line passing through (x1, y1) and if the intercept between the axes is divided in the ratio m:n at this
nx my
point then the equation is + = m+n.
x1 y1
• The equation of a straight line which makes a triangle with the co-ordinates axes whose centroid is
x y
(x1, y1) is + =1.
3x1 3y1
B Rajiv Reddy (JEE 2012, AIR 11)
8 . 1 6 | Straight Line

7.5 Foot of the Perpendicular


A(x1, y1) ax+by+c=0
The foot of the perpendicular (h, k) from (x1, y1) to the line ax + by + c = 0 is given by
h – x1 k – y1 –(ax1 +by1 + c)
= = .
a b a2 +b2
Hence, the co-ordinates of the foot of perpendicular is

 b2 x – aby – ac a2 y – abx – bc 
 1 1
, 1 1 .
 a2
+b 2
a2
+b 2  B(h, k)
 
Figure 8.31

The image of a point with respect to the line mirror: The image of A(x1, y1) with respect to the line mirror ax +
h – x1 k – y1 –2(ax1 +bx1 + c)
by + c = 0, B (h, k) is given by = =
a b a2 +b2 .
Special Cases
(a) Image of the point P(x1, y1) with respect to x-axis is (x1, – y1).
(b) Image of the point P(x1, y1) with respect to y-axis is (– x1, y1).
(c) Image of the point P(x1, y1) with respect to the line mirror y = x is Q(y1, x1).
(d) Image of the point P(x, y) with respect to the origin is the point (-x, -y).

Illustration 15: Find the equation of the line which is at a distance 3 from the origin and the perpendicular from
the line makes an angle of 30° with the positive direction of the x-axis.  (JEE MAIN)

Sol: By using xcosα + ysinα = P, we can solve this problem. Here α = 30° and P = 3.
3 y
So equation is x cos 30° + y sin 30° = 3 x + =3 ⇒ 3x + y = 6
2 2
Position of a point w.r.t. a line
Let the equation of the given line be ax + by + c = 0 and let the co-ordinates of the two given points be P(x1, y1)
and Q(x2, y2). Let R1 be a point on the line.
m –ax1 – by1 – c
The co-ordinates of R1 which divides the line joining P and Q in the ratio m:n are =
n ax2 +by 2 + c
.

Thus, the two points (x1, y1) and (x2, y2) are on the same (or opposite) sides of the straight line ax + by + c = 0
m
whether Point R1 divides internally or externally or sign of .
n
Note:
⇒ A point (x1, y1) will lie on the side of the origin relative to a line ax + by + c = 0, if ax1 +by1 + c and c have the
same sign.
⇒ A point (x1, y1) will lie on the opposite side of the origin relative to the line ax + by + c = 0, if
ax1 + by1 + c and c have the opposite sign.

Illustration 16: For what values of the parameter α does the point M (α, α + 1) lies within the triangle ABC of
vertices A(0, 3), B(– 2, 0) and C(6, 1).  (JEE ADVANCED)
Sol:Here, the point M will be inside the triangle if and only if |Area ∆MBC| + |Area ∆MCA| + |Area ∆MAB| = |Area
∆ABC|. And each individual area must be non-zero.
M a them a ti cs | 8.17

α α +1 1
1 1
Area MBC = –2 0 1 = 7α + 6
2 2
6 1 1

α α +1 1
1 1
Area MCA = 6 1 1 = –8α +12
2 2
0 3 1

α α +1 1
1 1
Area MAB = 0 3 1 = α+4
2 2
–2 0 1
Figure 8.32
0 3 1
1 1
Area ABC = –2 0 1 = .22
2 2
6 1 1
6 3
The above equation has critical points – 4, – and .
7 2
For α ≤ –4, the equation is
–7α – 6 – 8α + 12 – α – 4 =22
5 5
⇒ α=– which is not a solution, since – > –4
4 4
 6 6
For a∈  –4, –  , then equation is – 7α – 6 – 8α + 12 + α + 4 = 22 ⇒ α = –
 7 7
6
which is solution of equation but area MBC = 0 ⇒ M lies on BC ⇒ α = – is not the desired value.
7
 6 3
For a∈  – ,  , the equation is 7α + 68α + 12 + α + 4 = 22.
 7 2
 6 3
⇒ All α in the interval  – ,  satisfy the equation.
 7 2
3  3
Finally over  , ∞  , we get α = implies area MCA become zero.
 2  2
 6 3
⇒ The desired values of α lie in the interval  – ,  .
 7 2

7.6 Length of the Perpendicular P(x1,y1)


The perpendicular distance ‘p’ of a point P(x1, y1) from the line p
| ax1 +by1 + c |
ax + by + c = 0 is p= ax+by+c
a2 +b2 M
Figure 8.33
(a) Distance between parallel lines: The distance between the parallel lines a

x+ by + c1 = 0 and ax + by + c2 = 0 is c1 – c2
a2 +b2 y=mx+c
( 
(b) Lines making angle α with given line: The equations of the two straight lines (

passing through P(x’, y’) and making an angle α with the line y = mx + c P(x’,y’)

(where m = tan θ) are Figure 8.34


8 . 1 8 | Straight Line

y – y’ = tan (θ + α) (x – x’)
π
Note: If θ + α or θ – α is an odd multiple of , the corresponding line has equation x = x’.
2

(c) Concurrency of lines: Lines aix + biy + c1 = 0, where i = 1, 2, 3 are concurrent if they meet at a point. The

a1 b1 c1
condition for concurrency is
a2 b2 c2 = 0
a3 b2 c3

Illustration 17: The equation of the two tangents to the circle are 3x – 4y + 10 = 0 and 6x – 8y + 30 = 0. Find
diameter of the circle.  (JEE MAIN)

c1 – c2
Sol: By using formula of distance between two parallel line, i.e. , we can find the
a2 +b2
diameter of given circle. d

These are two parallel lines


3x – 4y + 10 = 0 .....(i) Figure 8.35

6x – 8y + 30 = 0 .....(ii)
15 − 10
Dividing second equation by 2 gives 3x – 4y + 15 = 0; ∴ d = =1
32 + 42

MASTERJEE CONCEPTS

(a) A triangle is isosceles if any two of its median are equal.


(b) Triangle having integral co-ordinates can never be equilateral.
(c) If arx + bry + cr = 0 (r = 1, 2, 3) are the sides of a triangle then the area of the triangle is given
2
a1 b1 c1
1
by a2 b2 c2 where C1, C2 and C3 are the cofactors of c1, c2 and c3 in the determinant.
2C1C2C3
a3 b3 c3

(d) Area of parallelogram:


(i) Whose sides are a and b and angle between them is θ is given by ab sin q. Area of
ABCD = ab.sinθ a
D C

b b

A a B

Figure 8.36

(ii) Whose length of perpendicular from one vertices to the opposite sides are p1 and p2 and angle
PP
between sides is θ is given by Area = 1 2
Sin θ

p2
p1
θ

Figure 8.37

Krishan Mittal (JEE 2012, AIR 199)


M a them a ti cs | 8.19

8. FAMILY OF LINES
Consider two intersecting lines L1: a1x + b1y + c1 = 0 and L2: a2x + b2y + c2 = 0, then
Type-1: The equation of the family of lines passing through the intersection of the lines
L1 + λL2 = 0
⇒ (a1x +b1y + c1) + λ(a2x +b2y + c2) = 0 where λ is a parameter.
Type-2: Converse, L1 + λL2 = 0 is a line which passes through a fixed point, where L1 = 0 and L2=0 are fixed lines
and the fixed point is the intersection of L1 and L2.

Type-3: Equation of AC ≡ u2u3 – u1u4 = 0 and BD ≡ u3u4 – u1u2= 0

u1a1x+b1y+c1=0
B C

u2a2x+b2y+d2=0 u4a2x+b2y+d1=0

A D
u3a1x+b1y+c2=0
Figure 8.38

Note that second degree terms cancel and the equation u2u3 – u1u4 = 0 is satisfied by the co-ordinate points B
and D.

Illustration 18: If a, b, c are in A.P., then prove that the variable line ax + by + c = 0 passes through a fixed point.
 (JEE MAIN)
Sol: By using given condition we can reduce ax + by + c = 0 to as L1 + λL2 = 0. Hence we can obtain co-ordinate
of fixed point by taking L1 = 0 and L2=0.
2b =a + c ⇒ c = 2b – a ⇒ ax + by + 2b – a = 0
∴ a (x – 1) + b(y + 2) = 0 This is of the form L1 + λL2 = 0, where b/a = l
∴ Co-ordinates of fixed point is (1, – 2).

9. ANGULAR BISECTOR

9.1 Bisectors of the Angle Between Two Lines


(a) Equations of the bisectors of angle between the lines ax + by + c = 0 and a1x + b1y + c1 = 0 are
ax +by + c a1 x +b1 y + c1
=± (ab1 ≠ a1b)
a2 +b2 a12 +b12
(b) To discriminate between the bisectors of the angle containing the origin and that of angle not containing
the origin, rewrite the equations, ax + by + c = 0 and a’x+ b’y + c’ = 0 such that the terms c, c’ are positive,
ax +by + c a'x +b'y + c'
then =+ gives the equation of the bisector of the angle containing origin and
2 2
a +b a'2 +b'2
ax +by + c a'x +b'y + c'
=– gives the equation of the bisector of the angle not containing origin.
2 2
a +b a'2 +b'2

(c) Acute angle bisector and obtuse angle bisector can be differentiated from the following methods:
Let two lines a1x + b1y + c1 = 0 and a2x + b2y + c2 = 0 intersect such that constant terms are positive.
8 . 2 0 | Straight Line

If a1a2 + b1b2 < 0, then the angle between the lines that contain the origin is acute and the equation for
a x +b1 y + c a x +b2 y + c2 ax1 +by1 + c a x +b2 y + c2
the acute angle bisector is 1 =+ 2 . Therefore =– 2 is the
2 2 2 2 2 2
a +b a2 +b2 a +b a22 +b22
equation of other bisector. If, however, a1a2 + b1b2 > 0, then the angle between the lines containing the origin
a1 x +b1 y + c1 a2 x +b2 y + c2
is obtuse and the equation of the bisector of the obtuse angle is =+ ; therefore
a1 x +b1 y + c1 a2 x +b2 y + c2 a12 +b12 a22 +b22
=– is acute angle bisector.
a12 +b12 a22 +b22

(d) Few more methods of identifying an acute and obtuse angle bisectors are as follows:
Let L1 = 0 and L2 = 0 are the given lines and u1 = 0 and u2 = 0 are the bisectors L1=0
between L1 = 0 and L2 = 0. Take a point P on any one of the lines L1 = 0 or L2 = 0 and
draw a perpendicular on u1 = 0 and u2 = 0 as shown. If P p
q
|p| < |q| ⇒ u1 is the acute angle bisector. L2=0

|p| > |q| ⇒ u1 is the obtuse angle bisector.


|p| = |q| ⇒ the lines L1 and L2 are perpendicular.

Note: The straight lines passing through P(x1, y1) and equally inclined with the lines u2=0
u1=0
a1x + b1y + c1 = 0 and a2x + b2y + c2 = 0 are those which are parallel to the bisectors
between lines and passing through the point P. Figure 8.39

MASTERJEE CONCEPTS

(a) Algorithm to find the bisector of the angle containing the origin: Let the equations of the
two lines be a1x + b1y + c1 = 0 and a2x + b2y + c2 = 0. The following methods are used to find the
bisector of the angle containing the origin:
Step I: In the equations of two lines, check if the constant terms c1 and c2 are positive. If the terms are
negative, then make them positive by multiply both the sides of the equation by –1.

Step II: Obtain the bisector corresponding to the positive sign, i.e. a1 x +b1 y + c1 a2 x +b2 y + c2
=
a12 +b12 a22 +b22
L1

Acute bisector

L2

Obtuse bisector
Figure: 8.40

This is the required bisector of the angle containing the origin, i.e. the bisectors of the angle between the
lines which contain the origin within it.
(b) Method to find acute angle bisector and obtuse angle bisector
(i) Make the constant term positive by multiplying the equation by –1.
(ii) Now determine the sign of the expression a1a2 + b1b2.
Mathematics | 8.21

MASTERJEE CONCEPTS

(iii)If a1a2 + b1b2 > 0, then the bisector corresponding to ‘+ve’ and ‘–ve’ signs give the obtuse and acute
angle bisectors, respectively, between the lines.
(iv)If a1a2 +b1b2 < 0, then the bisector corresponding to ‘+ve’ and ‘–ve’ signs give the acute and obtuse
angle bisectors, respectively.
Both the bisectors are perpendicular to each other. If a1a2 + b1b2 > 0, then the origin lies in the
obtuse angle and if a1a2 + b1b2 < 0, then the origin lies in the acute angle.

T P Varun (JEE 2012, AIR 64)

MASTERJEE CONCEPTS

Incentre divides the angle bisectors in the ratios (b + c):a, (c + a):b and (a + b):c . Angle bisector divides
the opposite sides in the ratio of remaining sides.

Figure: 8.41
BD AB c
= =
DC AC b
Aishwarya Karnawat (JEE 2012, AIR 839)

9.2 Bisectors in Case of Triangle


Two possible models are as follows:
Case-I: When vertices of a triangle are known, compute the sides of the
A(x1,y1)
triangle and the incentre. All the internal bisectors can be known, using the
co-ordinates of incentre and vertices of triangle.
Note: If the triangle is isosceles/equilateral, then one can easily get the c b
incentre.
Case-II: When the equations of the sides are given,compute tan A, tan B,
(x2,y2)B C(x3,y3)
tan C by arranging the lines in descending order of their slope. Compute the a
acute/obtuse angle bisectors as the case may be. Plot the lines approximately
Figure 8.42
and bisectors containing or not containing the origin.
8 . 2 2 | Straight Line

Illustration 19: The line x + y =a meets the x- and y-axes at A and B, respectively. A triangle AMN is inscribed in
the triangle OAB, O being the origin, with right angle at N. M and N lie respectively on OB and AB. If the area of
the triangle AMN is 3/8 of the area of the triangle OAB, then AN/BN is equal to. (JEE ADVANCED)
(A) 3 (B) 1/3 (C) 2 (D) 1/2

Sol: (A) Here simply by using the formula of area of triangle, Y


1 B(0, a)
i.e. {x1(y2 – y3) + x2(y3 – y1) + x3(y1 – y2)} , we can solve the problem.
2
N
AN  a λa 
Let = λ . Then the co-ordinates of N are  , ,
BN  1 + λ 1 +λ M
where (a, 0) and (0, a) are the co-ordinates of A and B, respectively.
Now equation of MN perpendicular to AB is X
O A(a, 0)
λa a 1–λ  λ –1 
y– =x– or x – y = a So the co-ordinates of M are  0, a
1+ λ 1+ λ 1+λ  λ +1  Figure 8.43

Therefore, area of the triangle AMN is

1   –a  1 – λ 2 λa2
= a
   + a  =
2   λ + 1  (1 + λ )
2
 (1 + λ )2

Also area of the triangle OAB = a2/2.


λa2 3 1 2
So that according to the given condition: = . a ⇒ 3l2 – 10λ + 3 = 0 ; ⇒ λ = 3 or λ = 1/3.
(1 + λ ) 2 8 2
For λ = 1/3, M lies outside the segment OB and hence the required value of λ is 3.

10. PAIR OF STRAIGHT LINES

10.1 Pair of Straight Lines Through Origin


(a) A homogeneous equation of degree two of the type ax2 + 2hxy + by2 = 0 always represents a pair of straight
lines passing through the origin and if
(i) h2 > ab ⇒ lines are real and distinct.
(ii) h2 = ab ⇒ lines are coincident.
(iii) h2 < ab ⇒ lines are imaginary with real point of intersection, i.e. (0, 0)
(b) If y = m1x and y = m2x be the two equations represented by ax2 + 2hxy + by2 = 0, then

2h a
m1 + m2 =
– and m1m2 =
b b

Angle between two straight lines:


(c) If θ is the acute angle between the pair of straight lines represents by ax2 + 2hxy + b, then

2 h2 – ab
tan θ =
a+b

The condition that these lines are:


(i) At right angles to each other if a + b = 0, i.e. sum of coefficients of x2 and y2 is zero.
Mathematics | 8.23

(ii) Coincident if h2 = ab and (ax2 + 2hxy + by2) is a perfect square of ( ax + by)2 .


(iii) Equally inclined to the axis of x if h = 0, i.e. coefficient of xy =0.

Combined equation of angle bisectors passing through origin: The combined equation of the bisectors of
the angles between the lines ax2 + 2hxy + by 2 = 0 (a pair of straight lines passing through origin) is given by
x2 – y 2 xy
= .
a–b h

10.2 General Equation for Pair of Straight Lines


(a) ax2 + 2hxy + by 2 + 2gx + 2fy + c = 0 represents a pair of straight lines if:
a h g
2 2 2
abc + 2fgh – af – bg – ch = 0 , if h b f = 0
g f c

(b) The slope of the two lines represented by a general equation is the same as that between the two lines
represented by only its homogeneous part.

10.3 Homogenisation
The equation of the two lines joining the origin to the points of intersection of the line lx + my +n = 0 and the curve
ax2 + 2hxy + by 2 + 2gx + 2fy + c = 0 is obtained by homogenising the equation of the curve using the equation of
the line.
The combined equation of pair of straight lines joining origin to the points y
of intersection of the line given by lx + my + n = 0  …. (i)
The second degree curve:
lx+my+n=0
ax2 + 2hxy + by 2 + 2gx + 2fy + c = 0  …. (ii)
Using equation (i) and (ii) x
O
2
 lx + my   lx + my   lx + my 
ax2 + 2hxy + by 2 + 2gx   + 2fy   + c  =0  …. (iii) Figure 8.44
 –n   –n   –n 

 lx + my 
Obtained by homogenizing (ii) with the help of (i), by writing (i) in the form:   = 1.
 –n 

MASTERJEE CONCEPTS

Through a point A on the x-axis, a straight line is drawn parallel to y-axis so as to meet the pair of straight
lines.
ax2 + 2hxy + by 2 = 0 in B and C. If AB = BC, then 8h2 = 9ab.
Krishan Mittal (Jee 2012, Air 199)
8 . 2 4 | Straight Line

Illustration 20: The orthocentre of the triangle formed by the lines xy = 0 and x + y = 1 is  (JEE MAIN)
(A) (1/2, 1/2) (B) (1/3, 1/3) (C) (0, 0) (D) (1/4, 1/4)

Sol: (C) Here the three lines are x = 0, y = 0 and x + y = 1.


Since the triangle formed by the line x = 0, y = 0 and x + y = 1 is right angled, the orthocentre lies at the vertex
(0, 0), the point of intersection of the perpendicular lines x = 0 and y = 0.

Illustration 21: If θ is an angle between the lines given by the equation 6x2 + 5xy – 4y 2 + 7x + 13y – 3 = 0 then
equation of the line passing through the point of intersection of these lines and making an angle θ with the positive
x-axis is  (JEE ADVANCED)
(A) 2x + 11y + 13 = 0 (B) 11x – 2y + 13 = 0
(C) 2x – 11y + 2 = 0 (D) 11x + 2y – 11 = 0

Sol: (B) By taking the term y constant and using the formula of roots of quadratic equation, we can get the

2 h2 – ab
equation of two lines represented by the given equation and then by using tan θ = , we will get the
required result. a+b

Writing the given equation as a quadratic in x, we have

–(5y + 7) ± (5y + 7)2 + 24(4y 2 – 13y + 3)


6x2 + (5y + 7) x – (4y 2 – 13y + 3) =
0⇒ x=
12
2
–(5y + 7) ± 121y – 242y + 121 –(5y + 7) ± 11(y – 1) 6y – 18 –16y + 4
= = = or
12 12 12 12

⇒ 2x – y + 3 = 0 and 3x + 4y – 1 = 0 are the two lines represented by the given equation and the point of
intersection is (– 1, 1), obtained by solving these equations.

2 h2 – ab 2 (5 / 2)2 – 6(–4) 121 11


Also tan θ = , where a = 6, b = – 4, h = 5/2 = = =
a+b 6–4 4 2
11
So the equation of the required line is y=
–1 (x + 1) ⇒ 11x – 2y + 13 = 0
2

Illustration 22: If the equation of the pair of straight lines passing through the point (1, 1), and making an angle
θ with the positive direction of x-axis and the other making the same angle with the positive direction of y-axis is
x2 – (a + 2)xy + y 2 + a(x + y – 1) =
0 , a ≠ – 2, then the value of sin 2θ is  (JEE ADVANCED)
2 2
(A) a – 2 (B) a + 2 (C) (D)
(a + 2) a

Sol: (C) As both line passes through (1, 1) and one line makes angle θ with x-axis and other line with y–axis, slopes
of line are tan θ and cot θ
Equations of the given lines are y – 1 = tan θ (x – 1) and y – 1 = cot θ (x – 1)
So, their combined equation is [(y – 1) – tan θ (x – 1)] [(y – 1) – cot θ (x – 1) ] = 0
⇒ (y – 1)2 – (tan θ + cot θ) (x – 1) (y – 1) + (x – 1)2 = 0
⇒ x2 – (tan θ + cot θ) xy + y2 + (tan θ + cot θ – 2) (x + y – 1) = 0
Comparing with the given equation we get tan θ + cot θ = a + 2

1 2
⇒ = a + 2 ⇒ sin2θ =
sin θ cos θ a+2
Mathematics | 8.25

Illustration 23: If two of the lines represented by x 4 + x3 y + cx2 y 2 – xy 3 + y 4 = 0 bisect the angle between the
other two, then the value c is (JEE ADVANCED)
(A) 0 (B) –1 (C) 1 (D) – 6

Sol: (D) As the product of the slopes of the four lines represented by the given equation is 1 and a pair of line
represents the bisectors of the angles between the other two, the product of the slopes of each pair is –1.
So let the equation of one pair be ax2 + 2hxy – ay2 = 0.

x2 – y 2 xy
The equation of its bisectors is = .
2a h
By hypothesis x 4 + x3 y + cx2 y 2 – xy 3 + y 4
≡ (ax2 + 2hxy – ay2) (hx2 – 2axy – hy2) = ah(x4 + y4) + 2(h2 – a2) (x3y – xy3) – 6ahx2y2
Comparing the respective coefficients, we get ah =1 and c = –6ah = –6

11. TRANSLATION AND ROTATION OF AXES Y Y’


P

11.1 Translation of Axes


O’ X’
Let OX and OY be the original axes, and let the new axes, parallel to original axes, be O’X’
and O’Y’. Let the co-ordinates of the new origin O’ referred to the original axes be (h, k). If X
the point P has co-ordinates (x, y) and (x’, y’) with respect to original and new axes,
O

respectively, then x = x’ + h; y = y’ + k Figure 8.45

11.2 Rotation of Axes


Let OX and OY be the original system of axes and let OX’ and OY’ be the new system Y
of axes and angle XOX’ = θ (the angle through which the axes are turned). If the point
P
Y’ X’
P has co-ordinates (x, y) and (x’, y’) with respect to original and new axes, respectively,
then
x = x’ cos θ – y’ sin θ and y = x’ sin θ + y’ cos θ

X
O
in matrix form it is as follows:
Figure 8.46
 x  cos θ − sin θ   x' 
 =  
 y   sin θ cos θ   y '

MASTERJEE CONCEPTS

If origin is shifted to point (α, β) , then new equation of curve can be obtained by putting x + α in place
of x and y + β in placed of y.
Vaibhav Krishnan (JEE 2009, AIR 22)

Illustration 24: The line L has intercepts a and b on the co-ordinate axes. The co-ordinate axes are rotated through
a fixed angle, keeping the origin fixed. If p and q are the intercepts of the line L on the new axes, then
1 1 1 1
– + – is equal to  (JEE MAIN)
2 2 2
a p b q2
(A) –1 (B) 0 (C) 1 (D) None of these
8 . 2 6 | Straight Line

Sol: (B) By using intercept form of equation of line, we will get equation of line before and after rotation. As their
perpendicular length from the origin does not change, by using distance formula the result can be obtained.
x y X Y
Equation of the line L in the two co-ordinate system is + = 1, + = 1 Where (x, y) are the new co-ordinates
a b p q
of a point (x, y) when the axes are rotated through a fixed angle, keeping the origin fixed. As the length of the
perpendicular from the origin has not changed:
1 1 1 1 1 1 1 1 1 1
= ⇒ + = + ⇒ – + – =0
2 2 2 2 2 2 2
(1 / a2 ) + (1 / b2 ) (1 / p2 ) + (1 / q2 ) a b p q a p b q2

Illustration 25: Let 0 < α < π/2 be a fixed angle. If P = (cos θ, sin θ) and Q = (cos (α – θ), (sin (α – θ)). Then Q is
obtained from P by  (JEE ADVANCED)
(A) Clockwise rotation around origin through an angle α.
(B) Anti-clockwise rotation around origin through an angle α.
(C) Reflection in the line through the origin with slope tan α.
(D) Reflection in the line through the origin with slope tan α/2.

Sol: As we know angle decreases during clockwise rotation and increases during anticlockwise rotation.
D Clockwise rotation of P through an angle α takes it to the point (cos (θ – α), sin(θ – α)) and anticlockwise takes it
to (cos (α + θ), sin (α + θ))
sin θ – sin(α – θ) 2cos(α / 2)sin(θ – α / 2)
Now slope of PQ = = = − cot ( α / 2 )
cos θ – cos(α – θ) –2sin(α / 2) – sin(θ – α / 2)
⇒ PQ is perpendicular to the line with slope tan(α/2). Hence, Q is the reflection of P in the line through the origin
α
with slope tan   .
2

MASTERJEE CONCEPTS

RELATION BETWEEN THE COEFFICIENT


Conditions for two lines to be coincident, parallel, perpendicular and intersecting: Two lines
a1x + b1y + c1 = 0 and a2x + b2y + c2 = 0 are
a1 b1 c1
• Coincident, if = =
a2 b2 c2

a1 b1 c1
• Parallel, if = ≠
a2 b2 c2

a1 b1
• Intersecting, if ≠
a2 b2
• Perpendicular, if a1a2 + b1b2 = 0
The three lines a1x + b1y + c1 = 0, a2x + b2y + c2 = 0 and a3x + b3y + c3 = 0 are concurrent, if
a1 b1 c1
a2 b2 c2 = 0
a3 b3 c3
Aman Gour (JEE 2012, AIR 230)
M a them a ti cs | 8.27

PROBLEM-SOLVING TACTICS

(a) In most of the questions involving figures like triangle or any parallelogram, taking origin as (0,0) helps a lot
in arriving at desired solution. One must ensure that conditions given are not violated.
(b) One must remember that in an isosceles triangle, centroid, orthocentre, incentre and circumcentre lie on the
same line.
(c) The centroid, incentre, orthocentre and circumcentre coincide in an equilateral triangle.
(d) If area of the triangle is zero, then the three points are collinear.
(e) Find the equation of the line passing through two given points, if the third point satisfies the equation of the
line, then three points are collinear
(f) Whenever origin is shifted to a new point (α, β), then new equation can be obtained by putting x + α in place
of x and y + β in placed of y.

FORMULAE SHEET

(a) Distance Formula: The distance between two points P(x1, y1) and Q(x2, y2) is

PQ = (x1 – x2 )2 + (y1 – y 2 )2 = (x1 – x2 )2 + (y1 – y 2 )2

And between two polar co-ordinate A(r1, q1) and B(r2, q2) is AB = r12 + r22 − 2r1r2 cos(θ1 − θ2 )

(b) Section Formula: If P(x1, y1), Q(x2, y2) and the point R(x, y) divide the line PQ internally in the ratio m:n then
the co-ordinates of R will be
mx2 + nx1 my 2 − ny1  mx + nx1 my 2 + ny1 
x= and y = i.e. R  2 , 
m+n m−n ,  m+n m + n 
 x + x 2 y1 + y 2 
And if R is a mid-point of line PQ, then the co-ordinates of R will be  1 , 
 2 2 

(c) Centroid of Triangle: If A(x1, y1), B(x2, y2) and C(x3, y3) be the vertices of the triangle ABC and G is
 x + x 2 + x3 y1 + y 2 + y 3 
Centroid, then co-ordinate of G will be  1 ,  .
 3 3 A
  I3 I2
ax1 + bx2 + cx3 ay1 + by 2 + cy 3
(d) Co-ordinates
= of Incentre: x = , y B C
a+b+c a+b+c

(e) Co-ordinates of Ex-centre: As shown in figure, ex-centres of ∆ABC I1


with respect to vertices A, B and C are denoted by
I1, I2 and I3, respectively,
Figure 8.47

 −ax1 + bx2 + cx3 −ay1 + by 2 + cy 3   ax1 – bx2 + cx3 ay1 − by 2 + cy 3 


I1 =  ,  ; I2 =  ,  ,
−a + b + c −a + b + c  a−b + c a−b + c 
 
 ax + bx2 − cx3 ay1 + by 2 − cy 3 
I3 =  1 , 
 a+b−c a+b−c 
8 . 2 8 | Straight Line

(f) Co-ordinates of Circumcentre: If O is the circumcentre of any ∆ABC, then its co-ordinates will be

 x sin2A + x2 sin2B + x3 sin2C y1 sin2A + y 2 sin2B + y 3 sin2C 


O =  1 , 
 Σ sin2A Σ sin2A 

(g) Co-ordinates of Orthocentre: If H is the orthocentre of any ∆ABC, then its co-ordinates will be

 x tanA + x2 tanB + x3 tanC y1 tanA + y 2 tanB + y 3 tanC 


H=  1 ,
 Σ tanA Σ tanA 
 
y 2 − y1
(h) Slope of Line: Slope of line made by joining of points P(x1, y1) and Q(x2, y2) is given by m
= tan=
θ
x2 − x1
m2 − m1
(i) Angle between two Lines: tan θ =
1 + m1m2
( j) Equation of a Line:
y 2 – y1
(i) Slope point form: y – y1 = m(x – x1) ; (ii) Two point form: y – y1 = (x – x1 )
x2 – x1
x y
(iii) Slope intercept form: y = mx + c; (iv) Intercept form: + =1
a b
(v) Normal form: x cos α + y sin α = P
x – x1 y – y1
(vi) Parametric form or distance form: = = r ; and x = x1 + r cos θ and y = y1 + r sin θ
cos θ sin θ

(k) Length of Perpendicular: The perpendicular distance ‘p’ of a point P(x1, y1) from the line ax + by + c = 0 is
| ax1 +by1 + c |
p =
a2 +b2
c1 – c2
(i) Distance between parallel lines:
a2 +b2
(ii) Lines making angle α with given line: y – y’ = tan (θ + α) (x – x’) and y – y’ = tan (θ – α) (x – x’)
a1 b1 c1
(iii) Concurrency of lines: The lines are concurrent if a2 b2 c2 = 0
a3 b2 c3

ax +by + c a1 x +b1 y + c1
(l) Equation of bisector of the angle between two lines: =± (ab1 ≠ a1b)
a2 +b2 a12 +b12
(m) Pair of straight line:
(i) ax2 + 2hxy + by2 = 0 always represents a pair of straight lines passing through the origin and if
•• h2 > ab ⇒ lines are real and distinct.
•• h2 = ab ⇒ lines are coincident.
•• h2 < ab ⇒ lines are imaginary with real point of intersection, i.e. (0, 0)
2h a
(ii) m1 + m2 =
− and m1m2 =
b b

2 h2 – ab
(iii) tan θ =
a+b

(n) General equation for pair of straight lines: ax2 + 2hxy + by 2 + 2gx + 2fy + c =0 represents a pair of straight
a h g
lines, if h b f = 0
g f c
M a them a ti cs | 8.29

Solved Examples

JEE Main/Boards 20 2 – 10 2 + 30 2 5
= =
5 2 +5 2 + 6 2 2
Example 1: Find the ratio in which y – x + 2 = 0 divides
the line joining A (3, – 1) and B (8, 9). ay1 + by 2 + cy 3
y=
a+b+c
Sol: By considering the required ratio be λ:1, and using
section formula, we can solve above problem. –10 2 + 20 2 + 30 2 5
= =
The point of division P is internal as A and B lie on 5 2 +5 2 + 6 2 2
opposite sides of given line.
Let required ratio be λ:1. Example 3: A rectangle PQRS has its side PQ parallel to
the line y = mx and vertices P, Q, S lie on lines y = a, x = b
 8λ + 3 9λ – 1  and x = –b, respectively. Find the locus of the vertex R.
Since, P  ,  lies on y – x + 2 = 0,
 λ +1 λ +1 
Sol: Here sides PQ and QR must be perpendicular to
9λ – 1 8λ + 3 2
∴ – +2 = 0 or λ = each other. Therefore product of their slopes will be –1.
λ +1 λ +1 3
Let R(h, k) be any point on the locus and let S and Q
y-x+2=0
have co-ordinates (–b, β) and (b, α), respectively, as T is
mid-point of SQ and PR.
 1 α–a
A B Thus P has co-ordinates (–h, a) =m
P b+h
⇒ α = a + m (b + h)
1 α –k
– = slope of QR =
Hence, required ratio is 2/3:1 or 2:3 m b–h
1
⇒α=k– (b – h)
Example 2: Find the incentre I of ∆ABC, if A is (4, – 2) B m
is (– 2, 4) and C is (5, 5). 1
a + m(b + h) = k – (b – h)
ax1 + bx2 + cx3 ay1 + by 2 + cy 3 m
Sol: Using x
= = , y , ∴ locus of R is
a+b+c a+b+c
we can obtain the incentre. x(m2 – 1) 2 my + b + am + bm2 = 0.

a = BC = (5 + 2)2 + (5 – 4)2 =
5 2 Example 4 Two equal sides AB and AC of an isosceles
b = CA = 2 2
(5 – 4) + (5 + 2) = 5 2 triangle ABC have equation 7x – y + 3 = 0 and x + y – 3
= 0, respectively. The third side BC of the triangle passes
c = AB = (–2 – 4)2 + (4 + 2)2 = 6 2 through point P(1, – 10). Find the equation of BC.

A Sol: For isosceles triangle ABC, AD is perpendicular


bisector of side BC and it also bisects angle BAC. Hence
by using equation of bisector formula, i.e.
ax +by + c a1 x +b1 y + c1
=± ,
a2 +b2 a12 +b12
B C
we can obtain slope of AD.
If incentre I is (x, y) , then, Equations of AB and AC are 7x –y + 3 = 0 and
– x – y + 3 = 0, respectively.
ax1 + bx2 + cx3
x= a1 = 7; b1 = – 1, c1 = 3;
a+b+c
8 . 3 0 | Straight Line

a2 = b2 = – 1, c2 = 3. Comparing the coefficients on both sides, we have


As c1 > 0, c2 > 0 and a1a2 + b1b2 = –6< 0 2h a
m 1 + m 2 = – ; m1m 2 =  …..(i)
b b
Equation of the bisector of the acute angle BAD is
Now, bx2 – 2hxy + ay2 = 0
7x – y + 3 –x – y + 3
= , i.e. 3x + y = 3 2h a
49 + 1 2 ⇒ x2 – xy + y 2 =
0
b b
A
⇒ x2 + (m1 + m2) xy + m1m2y2 = 0 (by (i))

⇒ (x + m1y) (x + m2y) = 0
–1 –1
⇒ y
= = x;y x
m1 m2
B D C P
1 Example 7: Find the angle φ between the straight lines
As slope of AD is –3, slope of BC is
3 π
Equation of BC through P(1, –10) is (x2 + y2) sin2α = (x cos θ – y sin θ)2, where 0 < 2α < .
1 2
y + 10 = (x – 1) or x – 3y = 31.
3 2 h2 – ab
Sol: We know tan φ = . Solving it, angle φ
can be obtained. (a + b)
Example 5: Find the equation of the line passing
through the intersection of lines x – 3y + 1 = 0, 2x +
x2 (cos2 θ – sin2 α ) – 2xy cos θ sin θ + y 2 (sin2 θ – sin2 α ) =
0
5y – 9= 0 and whose distance from the origin is 5.
a = cos2θ – sin2α, 2h = – 2 cos θ sin θ,
Sol: Equation of any line passing through the
b = sin2θ – sin2α
intersection of two other lines will be L1 + λL2 = 0.
Therefore, by using perpendicular distance formula of 2 h2 – ab
point to line, i.e = tan φ =
(a + b)
| ax1 +by1 + c |
p=
a2 +b2 2 cos2 θ sin2 θ – (cos2 θ – sin2 α )(sin2 θ – sin2 α )
=
we can obtain required equation of line. | (cos2 θ – sin2 α ) + sin2 θ – sin2 α |
Any line through the point of intersection of given lines is
2sin α cos α sin2α
= = = tan2α ∴ φ = 2a
x – 3y + 1 + λ (2x + 5y – 9) = 0 cos2α cos2α
or (1 + 2λ) x + (– 3 + 5λ) y + 1 – 9λ = 0
Example 8: The point A divides the line joining P ≡ (–5, 1)
| 0 + 0 + 1 – 9λ | and Q = (3, 5) in the ratio k:1. Find the two values of k for
5=
(1 + 2λ )2 + (–3 + 5λ )2 which the area of ∆ABC where B ≡ (1, 5), C ≡ (7, 2) is equal
to two square units.
7
Squaring and simplifying, we get λ = .
8 Sol: By using section formula, we can obtain the co-
ordinates of point A and then values of k by using the
Hence, required line has the equation 2x + y – 5 = 0.
triangle formula.
Co-ordinates of A, dividing the line joining points
Example 6: Show that bx2 – 2hxy +ay2 = 0 represent a
P ≡ (– 5, 1) and Q ≡ (3, 5) in the ratio k:1, are given by
pair of straight lines which are at right angles to the pair
of lines given by ax2 + 2hxy + by2 = 0. (3k – 5 / k + 1, 5k + 1 / k + 1) . Also, area of the ∆ABC
1
Sol: Here if the product of slopes of a pair of straight is given by ∆ = ∑ x1 (y 2 – y 3 )
2
lines represented by the given equations is –1, then
1
they are right angle to each other. = | [x (y – y 3 ) + x2 (y 3 – y1 ) + x3 (y1 – y 2 )] |
2 1 2
Let ax + 2hxy + by
2 2

1  3k – 5  5k + 1   5k + 1  
= (y – m1x) ( y – m2x)  (7) +  –2 –  + 7 – 5  = 2
2  k + 1  k + 1   k + 1  
M a them a ti cs | 8.31

3k – 5  5k + 1   5k + 1  JEE Advanced/Boards
(7) +  –2 –  + 7 – 5 =
±4
k +1  k +1   k +1 
Example 1: If A(–1, 5), B(3,1) and C(5, 7) are vertices of
⇒ 14k – 66 =
4k + 4,10k =
70,k =
7 a ∆ABC and D, E, F are the mid-points of BC, CA and AB,
respectively, then show that area ∆ABC = 4 times area
or 14k – 66 = – 4k – 4, 18k = 62,
(∆DEF).
k = (31/9).
Sol: Co-ordinates of D, E and F are first obtained by
Therefore, value of the k = 7, 31/9
using mid-point formula, and prove the given equation
Example 9: Prove that the sum of the reciprocals of the by using formula of area of triangle.
intercepts made on the co-ordinate axes by any line Co-ordinates of D, E, F are (4, 4), (2, 6) and (1, 3),
not passing through the origin and through the point respectively.
of intersection of the lines 2x + 3y = 6 and 3x + 2y = 6 –1 5 1
is constant. 1
∴ Area of ∆ABC = 3 1 1 = 16
2
5 7 1
Sol: Equation of any line through the points of
intersection of the given lines is L1 + λL2=0.
4 4 1
2x + 3y – 6 + k (3x + 2y – 6) =
0 1
Area of ∆DEF = 2 6 1 =4
2
(2 + 3k) x + (3 + 2k)y – 6(k + 1) =
0 1 3 1
x y A
⇒ + 1
=
((6(k + 1)) / (2 + 3k)) ((6(k + 1)) / (3 + 2k))
F E
Where k ≠ – 1
and in this case, sum of the reciprocals of the intercepts
made by this line on the co-ordinate axis is equal to
B C
D

2 + 3k + 3 + 2k 5(k + 1) 5 Hence, area of ∆ABC = 4 area (∆DEF)


= = .
6(k + 1) 6(k + 1) 6
Example 2: Point P(a2, a + 1) is a point of the angle (which
However, for k = – 1, the line become
contains the origin) between the lines 3x – y + 1 = 0,
x = y, which passes through the origin. x + 2y – 5 = 0. Find interval for values of ‘a’.

Sol: Given origin and P lie on same side of each line.


Example 10: Find the straight lines represented by
Substituting P in the given equation of line, we can
6x2+ 13xy + 6y2 +8x + 7y + 2 = 0 and also find their
obtain the required interval.
point of intersection.
a2 + 2a + 2 – 5< 0 and 3a2 – (a + 1) + 1 > 0
Sol: Taking term y as a constant and using quadratic
i.e.(a + 3) (a – 1) < 0 and a (3a – 1) > 0
2
−b ± b − 4ac 1 
roots formula, i.e. x = , we can obtain ∴ a ∈ (–3, 1) and a ∈ (– ∞, 0) ∪  , ∞ 
2a 3 
equations of required straight lines and after that by 1 
∴ a ∈ (– 3, 0) ∪  , 1 
3 
solving them we will get their point of intersection.
Rewrite the given equation as Example 3: Find the equations of the lines passing
through P(2, 3) and making an intercept AB of length
–(13y + 8) ± (13y + 8)2 – 24(6y 2 + 7y + 2)
x= 2 units between the lines y + 2x = 3 and y + 2x = 5.
12
Sol: Using equation of line in parametric form, i.e.
–(13y + 8) ± (5y + 4) –(2y + 1) –(3y + 2)
= = , x – x1= r cos θ and y – y1 = r sin θ, we can obtain the
12 3 2 required equation of line.
Hence, 3x + 2y + 1 = 0 and 2x + 3y + 2 = 0
1 4  Let equation of the line, in parametric form, be
are the required lines and they intersect at  , –  . x – 2 = r cos θ; y – 3 = r sin θ.
5 5 
8 . 3 2 | Straight Line

Then, A(2 + r1cos θ, 3 + r1 sin θ) and B(2 + r2 cos θ, ab2 a2b h k 


3 + r2 sin θ) lie on y+2x=3 and y+2x=5, respectively. By (i) and (ii) h = ,k= F is  , 
2 2
2 2 2 2
a +b a +b
∴ (3 + r1 sin θ) + 2 (2 + r1 cos θ) = 3 …. (i) ab
Slope of BE = .
and (3 + r2 sin θ) + 2 (2 + r2 cos θ) = 5 …. (ii) a2 + 2b2
∴ (r2 – r1) (sin θ + 2 cos θ) = 2 a2 + 2b2
Slope of AF =
–ab
Product of slopes of BE and AF is equal to (–1). Hence
AF ⊥ BE.
y+2x=3 A

Example 5: A square lying above the x-axis and has


y+2x=5 one vertex at the origin. A side passing through the
B  π
origin makes an angle α  0 < α <  with the positive
P  4
direction of x-axis. Prove that the equations of its
∴ sin θ + 2 cos θ = ± 1 (as |r2 – r1| = 2) diagonals are y (cos α – sin α) = x (sin α + cos α) and
y(sin α + cos α) + x (cos α – sin α) = 0, where a is the
3 cos2θ + 4 sin θ cos θ = 0
length of a side of the square.
π 3
θ= or tan θ = – ∴ Required lines are x = 2 and
2 4 Sol: Using slope point form of equation of line, i.e.
4y + 3x = 18 y – y1 = m(x – x1), we can obtain the result. Here m = tan
θ and x1 , y1 is 0.
Example 4: In a triangle ABC, AB = AC. If D is mid-point  π
Equation of diagonal OB is= y tan  α +  x
of BC, E is the foot of perpendicular from D on AC, and  4
F is the mid-point of DE, show that AF is perpendicular  π  π
or y cos  α += x sin  α + 
to BE.  4  4

Sol: As the geometrical fact to be established does or y (cos α – sin α) = x (cos α + sin α) ….(i)
not depend on position of ABC, we may assume that y
‘’D is the origin; BC and AD are along x and y axes B
respectively (as shown)”. Therefore by using intercept
form of equation of line, we can obtain required result.
C
Let BD = DC = a, and A and E have co-ordinates (0, b)
and (h, k), respectively. A
/4
x y
Line AC has the equation + = 1 
a b O
x
y
A(0,b) From the figure, point A is (a cos α, a sin α).
As diagonal AC is perpendicular to diagonal OB,
equation of AC is
cos α – sin α
E Y – a sin α = (x – acos α )
F cos α + sin α
or x (cos α – sin α) + y (cos α + sin α) = a
x
B(-a,0) D C(a,0)
Example 6: Two sides of a rhombus lying in the first
h k quadrant are given by 3x – 4y = 0 and 12x – 5y = 0. If
∴ + =1  ….(i)
a b the length of the longer diagonal is 12 units, find the
equations of the other two sides of the rhombus.
Also, (k/h) (–b/a) = – 1
( AC ⊥ DE)  ….(ii) Sol: Using formula of equation of bisector of the angle,
we can obtain equation of AC. Given the length AC, we
M a them a ti cs | 8.33

can obtain co-ordinates of C. ⇒ y – 9x = 0 or y + x = 0


Let ABCD be the rhombus with AC as the longer A(, )
diagonal, where A has co-ordinates (0, 0). AC is bisector y=9x
y=-x
of angle ∆BAD. The equations of the two angle bisectors
F E
(of angles formed by given lines) are:
3x − 4y 12x − 5y
= ± or 21x + 27y = 0 and 99x – 77y = 0.
5 13
B(x1,y1) C(x2,y2)
Since diagonal AC has positive slope, its equation is
x y The mid-point E of AC lies on
=
7 9
y y – 9x = 0

C β + y2 – 9(α + x2) = 0  ….(i)
D
12x=5y

Since AC is perpendicular to y – 9x = 0,
y –β
B We have 2 (9) = –1 Rewrite (i) and (ii) as 9x2 – y2
x2 – α
3x - 4y = β – 9a
x
A x2 + 9y2 = α + 9β

In parametric form, we get 9β – 40α 9α + 40β


∴ x2 = y2 =
x y 41 41
= ; Since AC = 12 units,
7 / 130 9 / 130  α + x1 β + y1 
Similarly, F  ,  , the mid-point of AB lies on
 7.12 9.12   2 2 
C has co-ordinates  , .
 130 130  y + x = 0.

Let sides DC and BC have equations 3x – 4y = a and 12x ∴ α + x1 + β + y1 = 0


– 5y = b, respectively. Substituting co-ordinates of C in or x1 +y1 = – (α + β)  …..(iii)
these equations yields
Since, AB is perpendicular to y + x = 0,
3.84 4.108 –180
=a = – ; We have,
130 130 130
β – y1
(–1) = –1 or β – y1 = α – x1
12.84 5.108 468 α – x1
=b = –
130 130 130 or x1 – y1 = α – β  ….(iv)
Solving (iii) and (iv), we have
Hence, equation of sides DC and BC are 3x – 4y +
180 468 x1 = – β and y1 = – a
= 0 and 12x – 5y = , respectively.
130 130 As B(x1, y1), C(x2, y2) and D(f, g) are collinear
we have,
Example 7: The base of a triangle ABC passes through
a fixed point (f, g) and its other two sides are bisected f g 1
at right angles by the lines y2 – 8xy – 9x2 = 0. Find the x1 y1 1 = 0
locus of the vertex x2 y2 1
Sol: Let vertices be A(α, β), B(x1, y1) and C(x2, y2) and let or f(y1 – y2) – g(x1 – x2) + (x1y2 – x2y1) = 0
(f, g) lie on BC. The mid-point of side AB and AC must
be lies on two perpendicular lines represented by y2 –  9α + 40β   9β – 40α 
or f –α –  – g  –β −
8xy – 9x2 = 0, respectively. Hence by solving them we 41  41 

 
will get locus of the vertex.
Let y2 – 8xy – 9x2 = 0 9α + 40β 9β – 40α
–β +α =0
41 41
⇒ (y – 9x) (y + x) = 0
8 . 3 4 | Straight Line

or f(50α + 40β) + g(40α – 50β) + 40(α2 + β2 ) =0 Locus lx2 + my 2 + n BP


⇒ – =λ=  …. (i)
of (α, β) is 4(x2 + y 2 ) + (4g + 5f)x + (4f – 5g)y =
0 lx3 + my 3 + n PC

Similarly, we obtain
Example 8: If the vertices of a triangle have integral
co-ordinates, prove that the triangle cannot be equilateral. CQ lx + my 3 + n
=– 3  …. (ii)
QA lx1 + my1 + n
Sol: Obtaining the area of triangle using D = (1/2) bc
sin A and using the co-ordinate form, we can conclude AR lx + my1 + n
and =– 1  …. (iii)
that the triangle cannot be equilateral if vertices have RB lx2 + my 2 + n
integral co-ordinates.
Multiplying (i), (ii) and (iii), we get the required
Consider a triangle ABC with vertices A ≡ (x1, y1),
result.
B ≡ (x2, y2), C ≡ (x3, y3). Let x1, x2, x3, y1, y2, y3 be the integers.
BC2 = (x2 – x3)2 + (y2 – y3)2 a positive integer. Example 10: The circumcentre of a triangle having
vertices A = (a, a tan α), B = (b, b tan β), C = (c, c tan
If the triangle is equilateral, then AB = BC = CA = a
γ) is the origin, where α + β + γ = π. Show that the
and ∠A =∠B =∠C =60° .
orthocentre lies on the line.
Area of the triangle = (1/2) bc sin A = (1/2) a2 sin 60°
 α β γ  α β γ
= (a2 / 2).( 3 / 2) = ( 3 / 4)a2 which is irrational.  4 cos cos cos  x – 4  sin sin sin  y – y = 0
 2 2 2  2 2 2
 a2 is a positive integer.
Sol: Consider the circumcentre ‘O’ to be the origin and
Now, the area of the triangle in terms of the co-ordinates
the equation of the circumcircle be x2 + y2 = r2. As
= (1/2) [(x1(y2 – y3) + x2 (y3 – y1) + x3 (y1 – y2 )] vertices of triangle lies on this circle, we can obtain the
which is rational number. co-ordinates of centroid by using the respective
formula.
This contradicts that the area is an irrational number, if
the triangle is equilateral. Since vertex A(a, a tan α) is r distance from the
circumcenter.
Example 9: A line L intersects the three sides BC, CA Therefore a2(1 + tan2α) = r2 ⇒ a = r cos α
and AB of a triangle ABC at P, Q and R, respectively.
A = (r cos α, r sin α)
 BP   CQ   AR  Similarly B = (r cos β, r sin β)
Show that      = –1 .
 PC   QA   RB 
C = (r cos γ, r sin γ)
Sol: Using equation of line lx + my + n = 0 and section A
formula, we can prove the given equation.
O
Consider a triangle ABC with vertices A(x1, y1), B(x2, y2) G
and C(x3, y3), and let lx + my + n = 0 be equation of H

the line L. If P divides BC in the ratio λ:1, then the co- B C

 λx + x 2 λy 3 + y 2 
ordinates of P are  3 , 
 λ +1 λ + 1  Centroid G
Also, as P lies on L, we have
 r(cos α + cos β + cos γ ) r(sin α + sin β + sin γ ) 
 λx + x 2   λy 3 + y 2   , 
l  3  + m   3 3 
  + n =0
 λ +1   λ +1 
Circumcentre O’ (0, 0) and let orthocentre H (h, k).
A We know that O, G, H are collinear. Therefore slope of
R OG = slope of OH
Q sin α + sin β + sin γ k
i.e. = '
cos α + cos β + cos γ h
1B P
C Point (h, k) will be on
M a them a ti cs | 8.35

x(sin α + sin β + sin γ) – y (cos α + cos β + cos γ) = 0  x + 3 y2 – 1 


Also D  2 , the mid-point of AC lies on the
 2 , 2 
 α β γ  α β γ  
⇒ x  4 cos cos cos  – y  1 + 4 sin sin sin  = 0 median through B,
 2 2 2  2 2 2
i.e. 6x + 10y – 59 = 0
And hence the result is α + β + γ = π.
 x +3  y2 – 1 
⇒ 6  2  + 10 
  – 59 =
0
Example 11: ABC is a variable triangle with the fixed  2   2 
vertex C(1, 2) and vertices A and B with co-ordinates
(cos t, sin t) and (sin t, – cos t), ⇒ 3x2 + 5y2 – 55 = 0  ….(ii)

Respectively, where t is a parameter. Find the Solving (i) and (ii), we get x2 =10, y2 = 5,

locus of the centroid of the ∆ABC. i.e. the co-ordinates of C are (10, 5) and thus
the equation of AC is 6x – 7y = 25
Sol: We can obtain co-ordinates of centroid G(α, β)
 x + x 2 + x3 y1 + y 2 + y 3  A(3, -1)
using the formula  1 ,  and
 3 3  Angle bisector
we will get required equation of locus of centroid by Median
solving them simultaneously.
Let G(α, β) be the centroid in any position .Then G(α, β) 6x + 80y - 59 = 0
x - 4y + 10= 0
 1 + cos t + sint 2 + sint – cos t  B(x1, y1) C(x2, y2)
= –  or
 3 3 
Let the slope of BC be m1. Since BC and AC are equally
1 + cos t + sint 2 – sint – cos t
=∴ α = ,β inclined to the angle bisector
3 3
x – 4y + 10 = 0,
∴ or 3α – 1 = cos t + sin t  …. (i)
(1 / 4) – m (6 / 7) – (1 / 4) 1 – 4m 17
3β =2 – sin t – cos t  …. (ii) = ⇒ =
1 + (1 / 4)m 1 + (6 / 7) × (1 / 4) 4 + m 34
Squaring and adding equations (i) and (ii), we get
2
(3α – 1)2 + (3β – 2)2 ⇒m= – Equation of BC is
9
= (cos t + sin t)2 + (sin t – cos t)2 2
y – 5 = – (x – 10) and 6x1 + 10y1 = 59
= 2 (cos t + sin t) = 2
2 2
9
∴ the equation of the locus of the centroid is (3x – 1)2+ Solving these equations, we get X1 = – 7/2, y1 = 8
(3y – 2)2 = 2
8 +1
Hence, equation of AB is y + 1 = (x – 3)
9(x2 + y2) –6 x – 12y + 3 = 0 –7 / 2 – 3
∴ 3(x2 + y2) – 2x – 4y + 1 = 0
Example 13: A triangle has the lines y = m1x and y
Example 12: Find equations of the sides of the = m2x for two of its sides, with m1 and m2 being roots
triangle having (3, –1) as a vertex, x – 4y + 10 = 0 and of the equation bx2 + 2hx + a = 0. If H(a, b) is the
6x + 10y – 59 = 0 being the equations of an angle orthocentre of the triangle, show that the equation of
bisector and a median, respectively, drawn from the third side is (a + b) (ax + by) =ab(a + b – 2h).
different vertices.
Sol: Line OD passes from orthocentre. Therefore it
Sol: Consider the vertices of the triangle to be A(3, – 1), must be perpendicular to the side AB. By considering
B(x1, y1) and C(x2, y2). Here the mid-point of AC lies on equation of AB as y = mx +c, we will get co-ordinates
the median through B. of A and B. Using slope point form of equation of line,
we can solve the problem.
Equation of the median through B be 6x + 10y – 59 = 0
and the equation of the angle bisector from C be The given lines y = m1x and y = m2x intersect at the
origin O (0, 0). Thus one vertex of the triangle is at the
x – 4y + 10 = 0; x2 – 4y2 + 10 = 0 ….(i)
origin O. Therefore, let OAB be the triangle and OA and
8 . 3 6 | Straight Line

OB be the lines since m1 and m2 are the roots of the equation


y = m1x  ....(i) bx2 + 2hx + a = 0

and y = m2x  .…(ii) m1+ m2 = – 2h/b and m1m2 = a/b


From (vii), we have
Y
–[a / b + 2hm / b + m2 ]a –[a + 2hm + bm2 ]a
B c= =
m(a / b + 1) m(a + b)
D
y=m2x y=mx+c From (iii), the equation of third side AB is
(a + 2hm + bm2 )a
H(a,b) A y = mx –
m1 (a + b)
y=m1x
X a (a – 2ha / b + ba2 / b2 )a
O or y = – x –
b (–a / b)(a + b)

Let the equation of the third side AB be or (ax + by)(a + b)= ab(a + b – 2h)
y = mx +c  …..(iii)
Given that H(a, b) is the orthocentre of the OAB, Example 14: Find the co-ordinates of the centroid,
circumcentre and orthocentres of the triangle
∴ OH ⊥ AB formed by the lines 3x – 2y = 6, 3x + 4y + 12 = 0 and
⇒ (b/a) × m = – 1 ⇒ m = –a/b  ….(iv) 3x – 8y + 12 = 0.

Solving (iii) with (i) and (ii), the co-ordinates of Sol: Solving the given equations, we can obtain the
 c cm1  co-ordinates of vertices of triangle. Using appropriate
A =  ,  and formula for finding the co-ordinates of centroid,
m
 1 – m m1 –m
circumcentre and orthocentre, the problem can be
 c cm2  solved.
B =  , 
 m2 – m m2 – m  Let sides AB, BC and CA have the

Now equation of line through A equations 3x – 2y – 6 = 0  ….(i)

perpendicular to OB is 3x – 8y + 12 = 0  ….(ii)
3x + 4y + 12 = 0  ….(iii)
cm1 1  c 
y– =–  x –  or Solving (ii), (iii) we get y = 0, x = – 4,
m1 – m m2  m1 – m 
A
x c(m1m2 + 1)
y= – +  ….(v)
m2 m2 (m1 – m)

Similarly, equation of line through B 3x - 2y- 6 =0 3x - 4y+ 12=0

perpendicular to OA is

x c (m1 m2 + 1)
y= − +  ….(vi)
m1 m1 (m2 − m) B C
3x - 8y+12=0
C = (– 4, 0)
The point of intersection of (v) and (vi) is the orthocentre
H (a, b). Solving (i), (ii) we get y = 3, x = 4
∴ Subtracting (vi) from (v), we get
B = (4, 3)
–cm(m1m2 + 1)
x=a=
(m1 – m)(m2 – m) Solving (i,) (iii) we get y = – 3, x = 0;

–[m1m2 – m(m1 + m2 ) + m2 ]a A = (0 – 3)
or c =  …..(vii)
m(m1m2 + 1)
M a them a ti cs | 8.37

 x + x 2 + x3 y1 + y 2 + y 3  Example 15: One diagonal of a square is the position


Centroid G =  1 ,  b
 3 3  m+
of the line tan( ±45°) = a =±1 which is intercepted
where vertices are (x, y), etc. b
1–m
a
 0 + 4 – 4 –3 + 3 + 0  between the axes. Find the co-ordinates of other two
G = ,  (0,0)
 3 3  vertices of the square. Also prove that if two opposite
vertices of a square move on two perpendicular lines,
To find the circumcentre: the other two vertices also move on two perpendicular
lines.
Let M(α, β) be the circumcentre.
m2 − m1
MA = MB = MC Sol: Using tan θ = , we can obtain the slopes
1 + m1m2
(α – 0)2 + (β+ 3)2 = (α – 4)2+ (β – 3)2 = (α + 4)2 + (β – 0)2
of AB and AD. As the slope of the given lines is –b/a,
α2 + β2 + 6β + 9 the two vertices are clearly on the diagonal BD of the
2 2 2 2 square ABCD.
= α + β – 8α – 6β + 25 = α + β – 8α + 16
If m be the slope of the line inclined at an angle of 45°
= 6β + 9 = –8α – 6β + 25 = 8α + 16
to BD,
6β + 9= –8α – 6β + 25 and 6β + 9 = 8α + 16 m + (b / a)
tan( ±45°) = =±1
8 α + 12 β – 16 = 0 1 – m(b / a)

2 α + 3 β – 4 =0  …. (i) C

8 α – 6 β + 7 =0  …. (ii)
1 23
Solving (i) and (ii), we get =
α ,=
β
12 18 (0,b)D
E
45 B
 1 23  45 (a, 0)
Circumcentre =  , 
 12 18 
O

A(0, -3)
A
E
a–b (a + b) a–b
m= or – AB is y – 0 = (x – a)
M a+b a–b a+b
a+b
AD is y– b = – (x – 0)
B(4, 3) D C(-4, 0) a–b
By solving these equation we get
MG 1 a–b b – a
Use = = The point A is 
GH 2 ,  . C is obtained by using the
 2 2 
fact that mid-point of AC and BD is same.
Let H(α, β) be the orthocentre

1 a+b a+b
α+2 C=  , 
12 1  2 2 
=0 = ⇒α –  …. (i)
3 6
The opposite vertices B, D move on two perpendicular
23 lines x-axis and y-axis. Now the point
β+2  –1 –23 
18 23
=0 = ⇒β – Then H  , . a–b b – a a+b a+b
3 9  6 9  A ,  lies on y = –x and point C  , 
 2 2   2 2 
lies on y = x.
8 . 3 8 | Straight Line

Example 16: If the image of the point (x1, y1) with x1 + x2 y1 + y 2


and a. + b. +c =0  ….(ii)
respect to the mirror ax + by + c = 0 be (x2, y2), show 2 2
that x2 – x1 y 2 – y1 –2(ax1 + by1 + c) From (ii), a(x1 + x2) + b(y1 + y2) + 2c = 0
= =
a b a2 + b2
Sol: As the line PQ joining the points P(x1, y1) and Q(x2, or (ax1 +by1 + c) + (ax1 +by1 + c) = 0  ….(iii)
y2) is perpendicular to the line ax + by +c = 0. Also the
x1 – x2 y1 – y 2 a(x1 – x2 ) + b(y1 – y 2 )
mid-point M of PQ is on the lines ax + by +c= 0. Hence From (i), = =
product of their slopes will be –1 and co-ordinates of M a b a2 + b2
lies on ax + by +c= 0.
By ratio and proportion
y1 – y 2  a 
 –  = –1  …..(i)
=
(ax1 + by1 + c) – (ax2 + by 2 + c)
x1 – x2  b 
a2 + b2
P(x1,y1)
2(ax1 + by1 + c)
= , using (iii)
a2 + b2
x2 – x1 y 2 – y1 – 2(ax1 + by1 + c)
ax+by+c=0 = =
a b a2 + b2
M

Q(x2,y2)

JEE Main/Boards

Exercise 1 Q.7 Find the coordinates of the vertices of a square


inscribed in the triangle with vertices A(0, 0), B(2, 1) and
Q.1 Find the slope of the line joining (4, – 6) and (– 2, – 5). C(3, 0); Given the two vertices are on the side AC.

Q.2 Show that the line joining (2, – 3) and (– 5, 1) Q.8 Find the equation of the straight line which passes
is (i) Parallel to the line joining (7, – 1) and (0, 3), through the origin and trisects the intercept of line
(ii) Perpendicular to the line joining (4, 5) and (0, – 2). 3x + 4y = 12 between the axes.

Q.3 A quadrilateral has the vertices at the points Q.9 A straight line passes through the point (3, – 2).
(– 4, 2), (2, 6), (8, 5) and (9, – 7). Show that the mid- Find the locus of the middle point of the portion of the
points of the sides of this quadrilateral are vertices of a line intercepted between the axes.
parallelogram.
Q.10 Find the equation of the straight line which passes
Q.4 Find the values of x and y for which A(2, 0), through the point (3, 2) and whose gradient is 3/4. Find
B(0, 2), C(0, 7) and D(x, y) are the vertices of an isosceles the coordinates of the point on the line that are 5 units
trapezium in which AB ||DC. away from the point (3, 2).

Q.5 Find the equations of the diagonals of the rectangle, Q.11 Find the distance of the point (2, 5) the lines 3x
whose sides are x = a, x = a’, y = b and y= b’. +y + 4 = 0 measured parallel to line having slope 3/4.

Q.6 In what ratio is the line joining the points (2, 3) and Q.12 The extremities of a diagonal of a square are
(4, –5) divided by the line joining the points (6, 8) and (1, 1), (– 2, – 1). Obtain the other two vertices and the
(– 3, – 2) ? equation of the other diagonal.
M a them a ti cs | 8.39

Q.13 In the given figure, PQR is an equilateral Q.23 A ray of light is sent along the line x – 2y – 3 = 0.
triangle and OSPT is a square. If OT =2 2 units, find Upon reaching the line 3x – 2y – 5 = 0, the ray is reflected
the equation of the lines OT, OS, SP, OR, PR and PQ. from it. Find the equation of the line containing the
y reflected ray.

Q.24 Find the image of the point (– 8, 12) with respect


K to the line mirror 4x + 7y + 13 = 0.
15 L
Q.25 The equations of two sides of a triangle are
45 3x – 2y + 6 = 0 and 4x + 5y = 20 and the orthocentre is
S
(1, 1). Find the equation of the third side.
45 45
60 45 45 45
x
M O Q Q.26 Find the equations of the straight lines passing
through the point of intersection of the lines x + 3y +
4 = 0 and 3x + y + 4 = 0 and equally inclined to the
axis.
Q.14 Find the equation of the medians of a triangle
formed by the lines x + y – 6= 0, x – 3y – 2 =0 and Q.27 Show that the straight lines x (a + 2b) + y(a + 3b)
5x – 3y + 2 =0. = a + b, for different values of a and b pass through a
fixed point.
Q.15 Find the coordinates of the orthocentre of the
triangle whose vertices are (0, 0), 2, – 1) and (–1, 3). Q.28 The equation of the base of an equilateral triangle
is x + y = 2 and the vertex is (2, – 1). Find the length of the
Q.16 Two vertices of a triangle are (3, – 1) and (–2, 3) side of the triangle.
and its orthocentre is origin. Find the coordinates of the
third vertix.

Q.17 If the lines 3x + y – 2 =0, px + 2y – 3 = 0 and


Exercise 2
2x – y – 3 =0 are concurrent, find the value of p.
Single Correct Choice Type

Q.18 Find the angle between the lines y – 3x – 5 = 0 and


Q.1 The pair of points which lie on the same side of the
3y – x + 6 =.
0 straight line, 3x – 8y – 7 = 0 is
(A) (0, – 1), (0, 0) (B) (0, 1), (3, 0)
Q.19 Prove that the points (2, – 1), (0, 2), (3, 3) and (5, 0)
are the vertices of a parallelogram. Also find the angle (C) (– 1, – 1), (3, 7) (D) (– 4, –3), (1, 1)
between its diagonals.
Q.2 Equation of the bisector of the acute angle between
Q.20 A and B are the points (–2, 0) and (0, 5). Find the the lines, 3x – 4y + 7 = 0 and 12x + 5y – 2= 0 is
Coordinates of two points C and D such that ABCD is (A) 11x – 3y + 9 = 0 (B) 11x + 3y – 9 = 0
a square.
(C) 3x – 11y + 9 = 0 (D) None

Q.21 Find the equations of the lines through the point


(3, 2), which make an angle of 45° with the line x – 2y = 3. Q.3 A ray of light passing through the point A (1, 2)
is reflected at a point B on the x-axis and then passes
through (5, 3). Then the equation of AB is
Q.22 A line 4x + y = 1 through the point A(2, – 7)
meets the line BC whose equation is 3x – 4y + 1 = 0 at (A) 5x + 4y = 13 (B) 5x – 4y = – 3
the point B. Find the equation of the line AC, so that (C) 4x + 5y = 14 (D) 4x – 5y = – 6
AB = AC.
8 . 4 0 | Straight Line

Q.4 The line x + 3y – 2 = 0 bisects the angle between a Q.11 The straight line, ax + by = 1 , makes with the
pair of straight lines of which one has equation x – 7y + curve px2 + 2axy + qy2 = r a chord which subtends a
5 = 0. The equation of the other line is right angle at the origin. Then:
(A) 3x + 3y – 1 = 0 (B) x – 3y + 2 = 0 (A) r(a2 + b2) = p + q (B) r(a2 + p2) = q + b
(C) 5x + 5y – 3 = 0 (D) None (C) r(b2 + q2) = p + a (D) None

Q.5 A is point (3, – 5) with respect to a given system of


axes. If the origin is moved to (4, – 3) by a translation Q.12 The lines y – y1 = m(x – x1) ± a 1 + m2 are tangents
of axes, then the new co-ordinates of the point A are to the same circle. The radius of the circle is:
given by (A) a/2 (B) a (C) 2a (D) None
(A) (1, – 2) (B) (– 1, 2)
Q.13 The equation of the pair of bisectors of the angles
(C) (– 1, – 2) (D) None of these
between two straight lines is, 12x2 – 7xy – 12y2 = 0. If
the equation of one line is 2y – x = 0, then the equation
x y 3 4 of the other line is
Q.6 The set of lines given by + = 1 , where + = 5
a b a b
(A) 41x – 38y = 0 (B) 38x – 41y = 0
are concurrent at a fixed point, then point is -
(C) 38x + 41y = 0 (D) 41x + 38y = 0
3 4
(A)  ,  (B) (0, b)
5 5 
Q.14 If the point B is symmetric to the point A(4, – 1)
(C) (a, 0) (D) None
with respect to the bisector of the first quadrant, then
the length AB is
Q.7 If P = (1, 0 ); Q = (– 1, 0) & R= (2, 0) are three
given points, then the locus of the points S satisfying (A) 3 2 (B) 4 2 (C) 5 2 (D) None
the relation, SQ2 +SR2 = 2 SP2 is
Q.15 The co-ordinates of the points A, B, C are (– 4, 0),
(A) A straight line parallel to x-axis.
(0, 2) & (– 3, 2) respectively. The point of intersection of
(B) A circle passing through the origin. the line which bisects the angle CAB internally and the
(C) A circle with the centre at the origin. line joining C to the middle point of AB is

(D) A straight line parallel to y-axis.  7 4  5 13 


(A)  – ,  (B)  – , 
 3 3  2 2 
Q.8 Area of the rhombus bonded by the four lines,
 7 10   5 3
ax ± by ± c =0 is: (C)  – ,  (D)  – , 
 3 3   2 2
c2 2c2
(A) (B)
2ab ab
Q.16 The sides of ∆ABC are 2x – y + 5 = 0, x + y – 5 =
2
4c ab 0 and x – 2y – 5 = 0. Sum of the tangents of its interior
(C) (D) angle is
ab 4c2
(A) 6 (B) 27/4 (C) 9 (D) None
Q.9 If the sum of the distances of a point from two
perpendicular lines in a plane is 1, then its locus is
Q.17 Equation of a straight line passing through the
(A) A square (B) Circle origin and making with x-axis an angle twice the size
(C) A straight line (D) Two intersecting lines of the angle made by the line y = 0.2x with the x-axis,
is:
Q.10 If the straight line x + 2y = 9, 3x – 5y = 5 & ax + (A) y = 0.4x (B) y = (5/12)x
by = 1 are concurrent, then the straight line 5x + 2y = 1
(C) 6y – 5x = 0 (D) None of these
passes through the point
(A) (a, – b) (B) (– a, b)
(C) (a. b) (D) (–a, –b)
M a them a ti cs | 8.41

Q.18 The shortest distance from the point M(– 7, 2) to π


(III) Rotation through an angle about the origin in
the circle x2 + y2 – 10x – 14y – 151 = 0 is 4
(A) 1 (B) 2 (C) 3 (D) None the counter clockwise direction. Then, the final position
of the point is given by the coordinates  (1980)

Q.19 The image of the pair of lines represented by


ax2+2hxy + by2 = 0 by the line mirror y=0 is
 1 7 
(A)  ,
 2 2
 (
(B) – 2, 7 2 )
(A) ax2 – 2hxy – by2 = 0 (B) bx2 – 2hxy + ay2 = 0
(C) bx2 + 2hxy + ay2 = 0 (D) ax2 – 2hxy +by2 = 0
 1 7 
(C)  –

, 
2 2
(D) ( 2,7 2 )
Q.20 The pair of straight lines x2 – 4xy + y2 = 0 together
Q.4 The straight lines x + y = 0, 3x + y – 4 = 0,
with the line x + y + 4 6 = 0 form a triangle which is
x + 3y – 4 = 0 form a triangle which is  (1983)
(A) Right angle but not isosceles
(A) Isosceles (B) Equilateral
(B) Right isosceles
(C) Right angled (D) None of these
(C) Scalene
(D) Equilateral Q.5 If the sum of the distance of a point from two
perpendicular lines in a plane is 1, then its locus is
Q.21 Points, A & B are in the first quadrant; point ‘O’ is  (1992)
the origin. If the slope of OA is 1, slope of OB is 7 and (A) Square (B) Circle
OA = OB, then the slope of AB is
(C) Straight line (D) Two intersecting lines
(A) – 1/5 (B) – 1/4 (C) –1/3 (D) – 1/2
Q.6 The orthocentre of the triangle formed by the lines
xy = 0 and x + y = 1, is  (1995)
Previous Years’ Questions 1 1 1 1
(A)  ,  (B)  , 
Q.1 The points (–a, – b), (0, 0), (a, b) and are  (1979) 2 2 3 3

(A) Collinear (C) (0, 0)


1 1
(D)  , 
(B) Vertices of a rectangle 4 4

(C) Vertices of a parallelogram


Q.7 The graph of the function cos x cos (x + 2) – cos2
(D) None of the above (x + 1) is  (1997)
(A) A straight line passing through (0, – sin21) with slope 2.
Q.2 Given the four lines with the equations, x + 2y – 3 = 0,
3x + 4y – 7 = 0, 2x +3y – 4 = 0, 4x + 5y – 6 = 0, then (B) A straight line passing through (0, 0).
 (1980) (C) A parabola with vertex (1, – sin21).
(A) They are all concurrent π 
(D) A straight line passing through the point  , – sin2 1 
(B) They are the sides of a quadrilateral and parallel to the x-axis.  2 

(C) Only three lines are concurrent


(D) None of these Q.8 The diagonals of a parallelogram PQRS are along
the lines x + 3y = 4 and 6x –2y = 7. Then PQRS must
be a  (1998)
Q.3 The point (4, 1) undergoes the following three
transformations successively (A) Rectangle (B) Square
(I) Reflection about the line y = x. (C) Cyclic quadrilateral (D) Rhombus
(II) Translation through a distance 2 unit along the
positive direction of x-axis.
8 . 4 2 | Straight Line

Q.9 Let PS be the median of the triangle with vertices Q.16 Let PS be the median of the triangle with vertices
P (2, 2), Q(6, – 1) and R(7, 3). The equation of the line P(2, 2), Q(6, -1) and R(7, 3). The equation of the line
passing through (1, – 1) and parallel to PS is  (2000) passing through (1, -1) and parallel to PS is (2014)
(A) 2x – 9y – 7 =0 (B) 2x – 9y – 11 = 0 (A) 4x - 7y - 11 = 0 (B) 2x+ 9y+ 7 = 0
(C) 2x + 9y – 11 = 0 (D) 2x + 9y + 7 = 0 (C) 4x+ 7y+ 3 = 0 (D) 2x - 9y - 11 = 0

Q.10 The incentre of the triangle with vertices Q.17 Let a, b, c and d be non-zero numbers. If the point
(1, 3),(0, 0) and (2, 0) is  (2000) of intersection of the lines 4ax + 2ay + c = 0 and 5bx +
2by + d= 0 lies in the fourth quadrant and is equidistant
 3 2 1  from the two axes then  (2014)
(A)  1,  (B)  , 
 2  3 3 (A) 2bc - 3ad= 0 (B) 2bc+ 3ad= 0
 

2 3  1  (C) 3bc - 2ad= 0 (D) 3bc+ 2ad= 0


(C)  ,  (D)  1, 
3 2   3
 
Q.18 Locus of the image of the point (2, 3) in the line
( 2x − 3y + 4 ) + k ( x − 2y + 3) = 0, k ∈ R , is a:  (2015)
Q.11 The number of integer values of m, for which the
x-coordinate of the point of intersection of the lines (A) Straight line parallel to y-axis
3x +4y = 9 and y = mx + 1 is also an integer, is (2001) (B) Circle of radius 2
(A) 2 (B) 0 (C) 4 (D) 1 (C) Circle of radius 3
(D) Straight line parallel to x-axis.
Q.12 Area of the parallelogram formed by the lines
y = mx, y = mx + 1, y = nx and y = nx + 1 equals (2001)
Q.19 The number of points, having both co-ordinates
|m+n| 2 as integers, that lie in the interior of the triangle with
(A) (B)
(m – n)2 |m+n| vertices (0, 0), (0, 41) and (41, 0), is:  (2015)
1 1 (A) 861 (B) 820 (C) 780 (D) 901
(C) (D)
|m+n| |m–n|
Q.20 Two sides of a rhombus are along the lines,
x – y + 1 = 0 and 7x – y - 5 = 0. If its diagonals intersect
Q.13 Let P = (– 1, 0), Q = (0, 0) and R = (3, 3 3) be
at (–1, –2), then which one of the following is a vertex
three points. Then, the equations of the bisector of the of this rhombus?  (2016)
angle PQR is  (2002)
1 8
(A) (-3, -8) (B)  , − 
3  3 3
(A) x+y =0 (B) x + 3y =
0
2  10 7
(C)  − , −  (D) (-3, -9)
3  3 3
(C) 3x + y =0 (D) x + y=
0
2

Q.14 If the line 2x + y = k passes through the point


which divides the line segment joining the points (1, 1)
and (2, 4) in the ratio 3 : 2, then k equals  (2012)

29 11
(A) (B) 5 (C) 6 (D)
5 5

Q.15 The x-coordinate of the incentre of the triangle


that has the coordinates of mid points of its sides as
(0, 1) (1, 1) and (1, 0) is  (2013)

(A) 2 − 2 (B) 1 + 2

(C) 1 − 2 (D) 2 + 2
M a them a ti cs | 8.43

JEE Advanced/Boards

Exercise 1 Q.5 Find the equation of the straight lines passing


through (– 2, – 7) & having an intercept of length 3
Q.1 Points O, A, B, C………are shown in figure where between the straight lines 4x + 3y = 12, 4x + 3y = 3.
OA = 2AB = 4BC =………so on. Let A is the centroid
of a triangle whose orthocentre and circumcentre are Q.6 Two sides of a rhombus ABCD are parallel to the
7 5 lines y = x + 2 & y = 7x + 3. If the diagonals of the
(2, 4) and  ,  respectively. If an insect starts moving rhombus intersect at the point (1, 2) & the vertex A is
2 2
on the y-axis, find the possible coordinates of A.
from the point O(0, 0) along the straight line is zig-zag
fashions and terminates ultimately at point P(α, β), then
find the value of (α + β) Q.7 Let O(0, 0), A(6, 0) and B(3, 3 ) be the vertices
of ∆OAB. Let R be the region consisting of all those
y points P inside OAB which satisfy d(P, OA) = minimum
{d(P, OB)), d(P, AB)}, where d(P, OA), d(P, OB) and
d(P, AB) represent the distance of P from the sides
C OA, OB and AB respectively. If the area of region R is
A 45 9(a – b), where a and b are coprime. Then, find the value of
45
(a + b).
45
45 B
x
O (0, 0) Q.8 Find the equations of the sides of a triangle having
(4, – 1) as a vertex. If the lines x – 1 = 0 and x – y – 1 = 0
are the equations of two internal bisectors of its angles.

Q.2 Let ABC be a triangle such that the coordinates of


Q.9 P is the point (– 1, 2), a variable line through P cuts
A are (– 3, 1). Equation of the median through B is 2x
the x & y axes at A & B respectively. Q is the point on
+ y – 3 = 0 and equation of the angular bisector of C
AB such that PA, PQ, PB are in HP. Find the locus of Q.
is 7x – 4y – 1 = 0. Then match the entries of column-1
with their corresponding correct entries of column-II.
Q.10 The equations of the altitudes AD, BE, CF of a
triangle ABC are x + y = 0, x + 4y = 0 and 2x– y =
Column I Column II
0 respectively. The coordinates of A are (t, –t). Find
(A) Equation of the line AB is (p) 2x + y – 3 = 0 coordinates of B & C. Prove that if t varies the locus of
(B) Equation of the line BC is (q) 2x – 3y + 9 = 0
the centroid of the triangle ABC is x + 5y = 0.

(C) Equation of CA is (r) 4x + 7y + 5 = 0


Q.11 The distance of a point (x1, y1) from each of two
(s) 18x – y – 49 = 0 straight lines which passes through the origin of co-
ordinates is δ; find the combined equation of these
straight lines.
Q.3 The equations of the perpendicular of sides AB and
AC of triangle ABC are x – y – 4 = 0 and 2x – y – 5 = 0
Q.12 Consider a, ∆ABC whose sides AB, BC and CA are
respectively. If the vertex A is (–2, 3) and point of intersection
represented by the straight lines 2x + y = 0. x + py = q
3 5 and x – y = 3 respectively. The point P is (2, 3)
of perpendicular bisector is  ,  , find the equation of
2 2
(i) If P is the centroid, then find the value of (p + q)
medians to the sides AB and AC respectively.
(ii) If P is the orthocentre, then find the value of (p + q)
Q.4 The interior angle bisector of angle A for the (iii) If P is the circumcentre, then find the values of
triangle ABC whose coordinates of the vertices are (p + q)
A(– 8, 5); B(– 15, – 19) and C(1, – 7) has the equation ax
+ 2y + c = 0. Find ‘a’ and ‘c’.
8 . 4 4 | Straight Line

Q.13 The sides of a triangle have the combined equation Q.20 The triangle ABC, right angled at C, has median
x2 – 3y2 – 2xy + 8y – 4 = 0. The third side, which is variable AD, BE and CF. AD lies along the line y = x + 3, BE lies
always passes through the point (– 5, – 1). If the range of along the line y = 2x + 4. If the length of the hypotenuse
values of the slope of the third line so that the origin is is 60, find the area of the triangle ABC.
an interior point of the triangle, lies in the interval (a, b),
 1  Q.21 A triangle has side lengths 18, 24 and 30. Find
then find  a + .
 b2  the area of the triangle whose vertices are the incentre,
circumcentre and centroid of the triangle.
Q.14 Consider a line pair 2x2 + 3xy – 2y2 – 10x + 15y
– 28 = 0 and another line L passing through origin
with gradient 3. The line pair and line L form a triangle Q.22 The points (1, 3) & (5, 1) are two opposite vertices
whose vertices are A, B and C. of a rectangle. The other two vertices lie on the lines
y = 2x + c. Find c & the remaining vertices.
(i) Find the sum of the cotangents of the inter ior angles
of the triangle ABC.
Q.23 A straight line L is perpendicular to the line
(ii) Find the area of triangle ABC. 5x – y = 1. The area of the triangle formed by the line L
(iii) Find the radius of the circle touching all the 3 sides & the coordinate axes is 5. Find the equation of the line.
of the triangle.
Q.24 Two equal sides of an isosceles triangle are given
Q.15 Show that all the chords of the curve by the equations 7x – y + 3 = 0 and x + y –3 = 0 & its
3x2 – y2 – 2x + 4y = 0 which subtend a right angle at the third side passes through the point (1, – 10). Determine
origin are concurrent. Does this result also hold for the the equation of the third side.
curve, 3x2 + 3y2 + 2x + 4y = 0 ? If yes, what is the point
of concurrency & if not, give reasons. Q.25 The equations of the perpendicular bisectors of
the sides AB & AC of a triangle ABC are x – y +5 = 0 &
Q.16 A straight line is drawn from the point (1, 0) to the x + 2y = 0, respectively. If the point A is (1, –2). Find the
curve x2 + y2 + 6x – 10y + 1 = 0, such that the intercept equation of the line BC.
made on it by the curve subtends a right angle at the
origin. Find the equations of the line. Q.26 Let P be the point (3, 2). Let Q be the reflection
of P about the x-axis. Let R be the reflection of Q about
Q.17 The two line pairs y2 – 4y + 3 = 0 andx2 + 4xy + the lines y = – x and Let S be the reflection of R through
4y2 – 5x –10y + 4 = 0 enclose a 4 sided convex polygon the origin. PQRS in a convex quadrilateral. Find the area
find. of PQRS.
(i) Area of the polygon.
Q.27 Two parallel lines 1 and  2 having non-zero
(ii) Length of the diagonals.
slope, are passing through the points (0, 1) and (– 1,0)
respectively. Two other lines 1 and  2 are drawn
Q.18 Find the equations of the two straight lines which through (0, 0) and (1, 0) which are perpendicular to 1
together with those given by the equation 6x2 – xy – y2 and  2 respectively. The two sets of lines intersect in
+ x + 12y – 35 = 0 will make a parallelogram whose four points which are vertices of a square. If the area of
diagonals intersect in the origin. p
this square can be expressed is the form q where p ∈
N, then the least value of (p + q)?
Q.19 A straight line passing through O(0, 0) cuts the
lines x = α, y = β and x + y = 8 at A, B and C respectively
such that OA . OB . OC = 482 and f(α, β) = 0 where Q.28 In an acute triangle ABC, the base BC has the
y 3 equation 4x – 3y + 3 = 0. If the coordinates of the
f(x,=
y) – + (3π – 2y)6 + ex + 2y – 2e – 6 orthocentre (H) and circumcentre (P) of the triangle are
x 2
(1, 2) and (2, 3) respectively, then the radius of the circle
(i) Find the point of intersection of lines x = α and y = β. m
circumscribing the triangle is , where m and n are
(ii) Find the value of (OA + OB + OC) a
relatively prime. Find the value of (m+ n).
(iii) Find the equation of line OA.
M a them a ti cs | 8.45

(You may use the fact that the distance between Q.4 The lines 3x + 4y = 9 & 4x – 3y + 12 = 0 intersect
orthocentre and circumcentre of the triangle is given at P. The first line intersects x-axis at A and the second
R 1 – 8 cos A cosB cosC ) line intersects y-axis at B. Then the circum radius of the
triangle PAB is

Q.29 The points (– 6, 1), (6, 10), (9, 6) and (– 3, – 3) are (A) 3/2 (B) 5/2 (C) 10 (D) None
the vertices of a rectangle. If the area of the portion of
this rectangle that lies above the x axis is a/b, find the Q.5 If the lines ax + y + 1 = 0, x + by + 1 = 0 & x + y +
value of (a +b), given a and b are coprime. c = 0, where a, b & c are distinct real numbers different
from 1 are concurrent, then the value of
Q.30 Consider the triangle ABC with sides AB and AC 1 1 1
+ + =
having the equation L1 = 0 and L2 = 0. Let the centroid. 1–a 1–b 1–c
Orthocentre and circumcentre of the ∆ABC and G, H and
S respectively. L = 0 denotes the equation of sides BC. (A) 4 (B) 3 (C) 2 (D) 1

(i) If L1: 2x – y = 0 and L2: x + y = 3 and G(2, 3) then find Q.6 The points A(a, 0), B(0, b), C(c, 0) & D(0, d) are such
the slope of the line L= 0. that ac = bd & a, b, c, d are all non zero. The points
thus:
(ii) If L1: 2x + y = 0 and L2: x – y + 2 = 0 and H(2, 3) then
find the y-intercept of L = 0. (A) Form a parallelogram (B) Do not lie on a circle

(iii) If L1: x + y – 1 = 0 and L2: 2x – y + 4 = 0 and S(2, 1) (C) Form a trapezium (D) Are concyclic
then find the x-intercept of the line L= 0.
Q.7 The angles between the straight lines joining the
origin to the points common to 7x2 + 8y2 – 4xy + 2x –
Exercise 2 4y – 8 = 0 and 3x – y = 2 is
π π π
(A) tan–1 2 (B) (C) (D)
Single Correct Choice Type 3 4 2

Q.1 Given the family of lines, a (3x + 4y + 6) + b (x + y Q.8 Distance between two lines represented by the line
+ 2) = 0. The line of the family situated at the greatest pair, x2 – 4xy + 4y2 + x – 2y – 6 = 0 is
distance from the point P(2, 3) has equation: 1
(A) (B) 5 (C) 2 5 (D) None
(A) 4x + 3y + 8 = 0 (B) 5x + 3y + 10 = 0 5
(C) 15x + 8y + 30 = 0 (D) None
Q.9 If the straight lines joining the origin and the points
of intersection of the curve
Q.2 On the portion of the straight line, x + 2y = 4 5x2 + 12xy – 6y2 + 4x – 2y + 3 = 0.
intercepted between the axes, a square is constructed
on the side of the line away from the origin. Then the And x + ky – 1= 0 are equally inclined to the co-ordinate
point of intersection of its diagonals has co-ordinates: axes then the value of k:

(A) (2, 3) (B) (3, 2) (C) (3, 3) (D) None (A) Is equal to 1
(B) Is equal to – 1
Q.3 The base BC of a triangle ABC is bisected at the (C) Is equal to 2
point (p, q) and the equation to the side AB & AC are
(D) Does not exist in the set of real numbers
px + qy = 1 and qx + py = 1. The equation of the
median through A is:
Q.10 If the vertices P and Q of a triangle PQR are given
(A) (p – 2q) x + (q – 2p)y + 1 = 0 by (2,5) and (4,–11) respectively, and the point R moves
(B) (p + q) (x + y) – 2 = 0 along the line N: 9x + 7y + 4 = 0, then the locus of the
centroid of the triangle PQR is a straight line parallel to
(C) (2pq – 1)(px+qy –1) =(p2 + q2 – 1)(qx+py – 1)
(A) PQ (B) QR (C) RP (D) N
(D) None
8 . 4 6 | Straight Line

Q.11 Let the co-ordinates of the two points A & B


Q.17 A is a point on either of two lines y + 3 | x |=2 at
be (1, 2) and (7, 5) respectively. The line AB is rotated
through 45° in anticlockwise direction about the point 4
a distance of units from their point of intersection.
of trisection of AB which is nearer to B. The equation of 3
the line in new position is The co-ordinates of the foot of perpendicular from A
(A) 2x – y – 6 = 0 (B) x – y – 1= 0 on the bisector of the angle between them are
(C) 3x – y – 11 = 0 (D) None of these
 2   2 
(A)  – ,2  (B) (0, 0) (C)  ,2  (D) (0, 4)
 3   3 
Q.12 If the line y =mx bisects the angle between the
lines ax2 + 2hxy + by2 = 0, then m is a root of the
quadratic equation: Q.18 All the points lying inside the triangle formed by
the points (1, 3), (5, 6) & (– 1, 2) satisfy
(A) hx2 + (a – b) x – h = 0
(A) 3x + 2y ≥ 0 (B) 2x + y + 1 ≥ 0
(B) x2 + h(a – b) x – 1 = 0
(C) (a – b)x2 + hx – (a – b) = 0 (C) 2x + 3y – 12 ≥ 0 (D) – 2x + 11 ≥ 0

(D) (a – b)x2 – hx – (a – b) = 0 x y
Q.19 Line + = 1 cuts the co-ordinate axes at
a b
x y
Q.13 A Triangle is formed by the lines 2x – 3y – 6 = 0; A(a, 0) & B(0, b) & the line + = –1 at A’ (–a’, 0) &
3x – y + 3 = 0 and 3x + 4y – 12 = 0. If the points P(α, 0) a' b'
and Q(0, β) always lie on or inside the ∆ABC, then B’(0, – b’ ). If the points A, B, A’, B’ are concyclic then the
orthocentre of the triangle ABA’ is
(A) α ∈ [– 1, 2] & β ∈ [– 2, 3]
(B) α ∈ [– 1, 3] & β ∈ [– 2, 4]  aa'   bb' 
(A) (0, 0) (B) (0, b’) (C)  0,  (D)  0, 
(C) α ∈ [– 2, 4] & β ∈ [– 3, 4]  b   a 

(D) α ∈ [– 1, 3] & β ∈ [– 2, 3]
Q.20 If one vertex of an equilateral triangle of side
‘a’ lies at the origin and the other lies on the line
Q.14 In a triangle ABC, side AB has the equation 2x + 3y
= 29 and the side AC has the equation, x + 2y = 16. If the x – 3y = 0 then the co-ordinates of the third vertex
mid-point of BC is (5, 6), then the equation of BC is are:
 3a a   3a a 
(A) x – y = – 1 (B) 5x – 2y = 13 (A) (0,a) (B)  , –  (C) (0, – a) (D)  – , 
 2 2   2 2
  
(C) x + y = 11 (D) 3x – 4y = – 9
Q.21 Three vertices of a triangle are A(4, 3); B(1, –1) and
Q.15 The vertex of a right angle o triangle lies on the C(7, k). Value(s) of k for which centroid, orthocentre,
straight line 2x + y – 10 = 0 and the two other vertices, incentre and circumcentre of the ABC lie on the same
at point (2, – 3) and (4, 1) then the area of triangle in straight line is/are:
sq. units is - 19
33 (A) 7 (B) – 1 (C) – (D) None
(A) 10 (B) 3 (C) (D) 11 8
5
Q.22 Equation of a line through (7, 4) and touching the
Multiple Correct Choice Type circle x2 + y2 – 6x + 4y – 3 = 0 is
(A) 5x – 12y + 13 = 0 (B) 12x – 5y – 64 = 0
Q.16 The area of triangle ABC is 20 cm2. The co-
ordinates of vertex A are (– 5, 0) and B are (3, 0). The (C) x – 7 = 0 (D) y = 4
vertex C lies upon the line, x – y = 2. The co-ordinates
of C are Q.23 The circumcentre of the triangle formed by the
lines, xy + 2x + 2y + 4 = 0 and x + y +2 = 0 is
(A) (5, 3) (B) (– 3, – 5) (C) (– 5, – 7) (D) (7, 5)
(A) (– 2, – 2) (B) (–1 , – 1)
(C) (0, 0) (D) (– 1, – 2)
M a them a ti cs | 8.47

Q.24 The sides of a triangle are x + y = 1, 7y = x and Match the conditions expressions in column I with
– 3y + x = 0. Then the following is an interior point of statement in column II
the triangle.
(A) Circumcentre (B) Centroid Q.2 Consider the lines given by  (2008)

(C) Incentre (D) Orthocentre L1: x + 3y – 5 = 0 ; L2: 3x – ky – 1 = 0


L3: 5x + 2y – 12 = 0
Q.25 Equation of a straight line passing through the
1 Column I Column II
point (2, 3) and inclined at an angle of tan−1   with
the line y + 2x = 5 is 2
(A) L1, L2, L3 are concurrent, if (p) k = – 9
(A) y =3 (B) x = 2
(B) One of L1, L2, L3 is parallel to at least 6
(q) k = –
(C) 3x + 4y – 18 = 0 (D) 4x + 3y – 17 = 0 one of the other two, if 5

(C) L1, L2, L3 form a triangle, if 5


Q.26 A ray of light travelling along the line x + y = 1 (r) k =
6
is incident on the x-axis and after refraction it enters
(D) L1, L2, L3 do not form a triangle, if (s) k = 5
the other side of the x-axis by turning π/6 away from
the x-axis. The equation of the line along which the
refracted ray travels is
Q.3 Three lines px + qy + r = 0, qx + ry + p = 0 and
(A) x + (2 – 3)y =
1 (B) (2 – 3)x + y =
1 rx + py + q = 0 are concurrent, if  (1985)
(C) y + (2 + 3)x =2 + 3 (D) None of these (A) p + q + r = 0 (B) p2 + q2 + r2 = pr + rq
(C) p3 +q3 + r3 = 3 pqr (D) None of these
Q.27 Consider the equation y – y1 = m(x – x1). If m &
x1 are fixed and different lines are drawn for different
Q.4 All points lying inside the triangle formed by the
values of y1, then
points (1, 3), (5, 0) and (– 1, 2) satisfy  (1986)
(A) The lines will pass through a fixed point
(A) 3x + 2y ≥ 0 (B) 2x +y – 13 ≥ 0
(B) There will be a set of parallel lines
(C) 2x – 3y – 12 ≤ 0 (D) – 2x + y ≥ 0
(C) All the lines interest the line x = x1
(D) All the lines will be parallel to the line Q.5 Let L1 be a straight line passing through the origin and
L2 be the straight line x+y=1. If the intercepts made by the
circle x2 + y2 – x + 3y = 0 on L1 and L2 are equal, then which
Previous Years’ Questions of the following equation can represent L3? (1999)
(A) x + y = 0 (B) x – y = 0
The codes (A), (B), (C) and (D) deformed as follows.
(A) Statement-I is true, statement-II is also true; (C) x + 7y = 0 (D) x – 7y = 0
statement-II is the correct explanation of statement-I.
(B) Statement-I is true, statement-II is also true; Q.6 If the distance between the plane Ax – 2y + z = d and
statement-II is not the correct explanation of x –1 y –2 z –3
statement-I. the plane containing the lines = = and
2 3 4
(C) Statement-I is true; statement-II is false. x–2 y –3 z– 4
= = is 6 , then |d| is……….. (2010)
(D) Statement-I is false; statement-II is true. 3 4 5

Q.1 Lines L1: y – x = 0 and L2: 2x + y =0 intersect the line L3: Q.7 The straight line 2x – 3y = 1 divides the circular
y + 2 = 0 at P and Q, respectively. The bisector of the acute region x2 + y2 ≤ 6 into two parts. If
angle between L1 and L2 intersects L3 at R.  (2007)
 3   5 3   1 1   1 1  
Statement-I: The ratio PR: RQ equals 2 2 : 5 . S =  2,  ,  ,  ,  , –  ,  ,   ,  (2011)
 4   2 4   4 4   8 4  
Because
Then the number of point(s) lying inside the smaller
Statement-II: In any triangle, bisector of an angle
part is……………
divides the triangle into two similar triangles.
8 . 4 8 | Straight Line

Q.8 For points P = (x1, y1) and Q = (x2, y2) of the Q.11 Using coordinate geometry, prove that the three
coordinate plane, a new distance d(P,Q) is defined by altitudes of any triangle are concurrent. (1998)
d(P, Q) = | x1 – x2 | + | y1 – y2 | . Let Q = (0, 0) and A =
(3, 2). Prove that the set of points in the first quadrant Q.12 A Straight line L through the point (3, -2) is
which are equidistant (with respect to the new distance) inclined at an angle 60° to the line 3x + y =. 1 If L
from O and A consists of the union of a line segment also intersects the x-axis, then the equation of L is
of finite length and an infinite ray. Sketch this set in a  (2011)
labelled diagram.  (2000)

Q.13 For a > b > c > 0, the distance between (1, 1) and
Q.9 A straight line L through the origin meets the line x the point of intersection of the lines ax + by + c = 0 and
+ y = 1 and x + y = 3 at P and Q respectively. Through bx + ay + c = 0 is less than 2 2 . Then (2013)
P and Q two straight lines L1 and L2 are drawn, parallel
to 2x – y = 5 and 3x + y = 5 respectively. Lines L1 and L2 (A) a + b – c > 0 (B) a – b + c < 0
intersect at R, show that the locus of R as L varies, is a (C) a – b + c > 0 (D) a + b – c < 0
straight line.  (2002)

Q.14 For a point P in the plane, let d1 (P ) and d2 (P ) be


Q.10 Let S be a square of unit area. Consider any the distance of the point P from the lines x – y = 0 and x
quadrilateral which has one vertex on each side of S. If a, b, + y = 0 respectively. The area of the region R consisting
c and d denote the length of the sides of the quadrilateral, of all points P lying in the first quadrant of the plane
prove that 2 ≤ a2 + b2 + c2 + d2 ≤ 4.  (1997) and satisfying 2 ≤ d1 (P ) + d2 (P ) ≤ 4  (2014)

MASTERJEE Essential Questions

JEE Main/Boards JEE Advanced/Boards

Exercise 1 Exercise 1
Q.10 Q.13 Q.23 Q.2 Q.6 Q.10
Q.24 Q.26 Q.27 Q.15 Q.19 Q.24
Q.27
Exercise 2
Q.11 Q.12 Q.16 Exercise 2
Q.18 Q.2 Q.6 Q.8
Q.12 Q.18 Q.21
Previous Years’ Questions Q.23 Q.26
Q.2 Q.7 Q.10
Q.11 Q.13 Previous Years’ Questions
Q.2 Q.5 Q.7
Q.9 Q.10
M a them a ti cs | 8.49

Answer Key

JEE Main/Boards
Q.14 x = 2, x + 9y – 14 = 0, 7x – 9y – 2 = 0
Exercise 1
Q.15 (– 4, – 3)
1
Q.1 – 36 45
6 Q.16 – ,–
7 7
Q.4 (7, 0) or (2, 5) Q.17 5

Q.5 (b – b')x + (a'– a)y + ab'– a'b Q.18 30° or 150°


= 0,(b – b')x + (a – a')y + a'b'– ab= 0 22
Q.19
= θ tan–1 ( − )
3
Q.6 5 : 97 externally
Q.20 D = (– 7, 2), C = (– 5, 7)
3   9  9 3 3 3  Q.21 3x – y – 7 = 0 and x + 3y – 9 = 0
Q.7 E  ,0  , F  ,0  , G  ,  , H  , 
 2   4   4 4  2 4  Q.22 52x + 89y + 519 = 0 or 4x + y = 1
Q.8 3x – 8y = 0, 3x – 2y = 0 Q.23 29x – 2y – 31 = 0
Q.9 3y – 2x = 2xy Q.24 (– 16, – 2)
Q.10 (7, 5) and (– 1, – 1) Q.25 26x – 122y – 1675 = 0
Q.11 5 Q.26 x + y + 2 = 0
 3 3  3 13  3 1 3  Q.27 P (2, – 1)
Q.12 C =  – ,  – ,, A
, ,=–,   , – 
 2 2  2 22  2 2 2 
2
Q.28
Q.13 OT → y = x, OS → y = –x, SP → y = x + 4, 3
PQ → y = –x + 4, PR → y = (2 – 3 )x + 4

Exercise 2

Single Correct Choice Type

Q.1 C Q.2 A Q.3 A Q.4 C Q.5 C Q.6 A


Q.7 D Q.8 B Q.9 A Q.10 C Q.11 A Q.12 B
Q.13 A Q.14 C Q.15 D Q.16B Q.17 B Q.18 B
Q.19 D Q.20 D Q.21 D

Previous Years Questions


Q.1 A Q.2 C Q.3 C Q.4 A Q.5 A Q.6 C
Q.7 D Q.8 D Q.9 D Q.10 D Q.11 A Q.12 D
Q.13C Q.14 C Q.15 A Q.16 B Q.17 C Q.18 B
Q.19 C Q.20 B
8 . 5 0 | Straight Line

JEE Advanced/Boards Q.15 (1, – 2), yes ( −1, −2 )


Q.16 x +y = 1; x + 9y = 1
Exercise 1
Q.17 (i) area – 6 sq. units, (ii) diagonals ar 5 & 53
Q.1 8
Q.18 6x2 – xy – y 2 – x – 12y – 35 = 0
Q.2 (A) R; (B) S; (C) Q
Q.19 (i) α =2 and β =3; (ii) 9 2 ; (iii) x – y = 0
Q.3 x + 4y = 4 ; 5x + 2y = 8
Q.20 400 sq. units
Q.4 a = 11, c = 78
Q.21 3 units
Q.5 7x + 24y - 182 = 0 or x = – 2
Q.22 c = – 4; B (2, 0) ; D (4, 4)
 5
Q.6 (0, 0) or  0,  Q.23 x + 5y + 5 2 = 0 or x + 5y – 5 2 = 0
 2
Q.7 5 Q.24 x – 3y – 31 = 0 or 3x+ y + 7 = 0
Q.8 2x – y +3 =0, 2x + y – 7 =0, x – 2y – 6 =0 Q.25 14x + 23y = 40
Q.9 y = 2x Q.26 15
 2t –t   t 
Q.10 B  – .  .C  .t  Q.27 6
 3 6  2 
Q.28 63
Q.11 (y12 – δ2 )x2 – 2x1 y1 xy + (x12 – δ2 )y 2 =
0
Q.29 533
Q.12 (a) 74 ; (b) 50 ; (c) 47
3
Q.30 (a) 5 ; (b) 2 ; (c)
Q.13 24 2
50 63 3
Q.14 (a) ; (b) ; (c) (8 5 – 5 10 )
7 10 10

Exercise 2
Single Correct Choice Type

Q.1 A Q.2 C Q.3 C Q.4 B Q.5 D Q.6 D


Q.7 D Q.8 B Q.9 B Q.10 D Q.11 C Q.12 A
Q.13 D Q.14 C Q.15 B

Multiple Correct Choice Type

Q.16 B, D Q.17 B, C Q.18 A, B, D Q.19 B, C Q.20 A, B, C, D Q.21 B, C

Q.22 A, C Q.23 B, C Q.24 B, C Q.25 B, C Q.26 A, C Q.27 B, C

Previous Years Questions


Q.1 C Q.2 A → s; B → p, q; C → r; D → p, q, s Q.3 A , C Q.4 A, C
Q.5 B , C Q.6 6 Q.7 2 Q.13 A Q.14 6
M a them a ti cs | 8.51

Solutions

JEE Main/Boards 5
−1 +
2 3 1
Slope of GH = = =
Exercise 1 17 5
− 12 4
2 2
Sol 1: Slope of line joining [4, –6] & [–2, –5] 5
4+
−5 + 6 1 Slope of EH = 2 = −13
= = 5
−2 − 4 −6 7
−1 −
2
Sol 2: Line 1 joining (2, –3) and (–5, 1) EF || GH and FG || EH, hence midpoints E, F, G, H form a
parallelogram.
4
⇒ Slope = −
7
Sol 4: AB || DC
 2 → line joining (7, –1) and (0, 3)
2−0
−4 Slope of AB = = –1
⇒ Slope = 0−2
7
7−y y −7
 3 → line joining (4, 5) and (0, –2) Slope of DC = =
0−x x
7 y – 7 = –x ⇒ x + y = 7 – x …(i)
⇒ Slope =
4
1 ||  2 and  m2 ⋅ m3 =
−1 A B

i.e. 1 and  2 are perpendicular to  3

Sol 3: A(–4, 2) ; B(2, 6) ; C(8, 5); D(9, –7) mid points of D C


Trapezium is isosceles i. e. AD = BC
 −4 + 2 2 + 6 
AB = E  ,  = (–1, 4) (x – 2)2 + y2 = 25
 2 2 
x2 + y2 – 4x = 21 …(ii)
2 + 8 6 + 5  11 
BC = F  ,  = 5,  x2 + y2 + 2xy = 49
 2 2   2
4x + 2xy = 28 ⇒ 2x(2 + y) = 28 from equation (i)
 8 + 9 5 − 7   17 
CD = G  ,  =  , − 1
 2 2  2  y= 7-x

 9 − 4 −7 + 2   5 −5  ⇒ x(9 – x) = 14 ⇒ x2 – 9x + 14 = 0
AD = H  , =  , 
 2 2  2 2  ⇒x = 7, 2

 11  If x = 7, y = 0 and if x = 2, y = 5
 2 − 4 3 1
Slope of EF =   = = Ans is (7, 0) and (2, 5)
 5 + 1  12 4
 
Sol 5:
11 D C y = b’
+1 −13
13
Slope of FG = 2 = = y=b
17 −7 7 A B
5−
2

x=a x = a’
8 . 5 2 | Straight Line

Coordinates are A(a, b), B(a′, b), C(a′, b′) and D(a, b′) It is a square hence b – a = c
b′ − b H lies of x = 2y ⇒ a = 2c ⇒ b = 3c
Slope of line AC =
a′ − a G lies on x + y = 3 ⇒ b + c = 3
b′ − b 4c = 3
Slope of line BD =
a − a′
3 9 6
Equation of line AC ⇒c= ; b = ; a=
4 4 4
 b′ − b  Coordinates of square are
⇒y–b=   (x – a)
 a′ − a 
3  9  9 3 3 3 
⇒ (b′ – b)x + (a – a′)y – ab′ + a⇒b = 0 E  ,0  , F  ,0  , G  ,  , H  , 
 2   4   4 4  2 4 
Equation of line BD
b′ − b
⇒y–b= (x – a′) Sol 8:
a − a′
⇒ (b′ – b)x – (a – a′)y – a'b' + ab = 0 (0,3) A
B

Sol 6: 1 (line joining (2, 3) and (4, –5)) (4,0) 3x+4y=12

 2 (line joining (6, 8) and (–3, –2))

(6, 8)  1(4,0) + 2(0,3)  4 6


A=  ⇒  , 
 3  3 3
m 1
 2(4,0) + 1(0,3)  8 3
(2, 3) F (4, -5) B=  ⇒  , 
 3  3 3
(-3, -2) 3x
Line OA → y = ⇒ 3x – 2y = 0
2
Lets assume it divides 1 in ratio m : 1
3x
Line OB → y = ⇒ 3x – 8y = 0
 4m + 2 3 − 5m  8
Coordinates of F are  , 
 m+1 1+m 
Sol 9:
y −8 10
Equation of  2 is =
x−6 9
3 − 5m
−8
10 −5 − 13m 10 B
⇒ 1+m = ⇒ =
4m + 2 9 −2m − 4 9
−6 A
m+1
−5
⇒ 45 + 117m = 20m + 40 ⇒ m = (3,-2)
97
i. e. in ratio 5 : 97 (externally).
y+2
=m
x −3
Sol 7:
y + 2 = m(x – 3)
B  2 
H G A ⇒  3 + ,0  [x intercept]
 m 
A E F C B ⇒ (0, –2 – 3m) [y intercept]

 2 
 3 + m −2 − 3m 
E = (a, 0), F = (b, 0), G = (b, c) Mid point of AB  , 
 2 2 
H = (a, c)  
M a them a ti cs | 8.53

−3m − 2 3m + 2 −3
y= ,x= Slope of AC =
2 2m 2
 2y + 2  BD = 13
m = – 
 3  Length of side =
13
 −2y − 2   −2y − 2  2
⇒ 2x   = 3  +2
 3   3  Let slope of AB be m

⇒ –4xy – 4x = –6y – 6 + 6 2
m−
tan 45º = 3 =1
⇒ 2xy + 2x – 3y = 0 2m
1+
3
y −2 3
Sol 10: = and y = 3x – 1 m 5 5m −1 −1
x −3 4 = ⇒ m = 5 or = ⇒m=
3 3 3 3 5
Points which are 5 units away from (3, 2) are
 13 13 
A = 1 + cos θ, 1 + sin θ 
(3 ± 5 cos θ, 2 ± 5 sin θ )
 2 2 
3 −1 −5
tan θ = ⇒(3 ± 4, 2 ± 3)⇒ [7, 5] [–1, –1] Now cos θ = , sin θ =
4
26 26

Sol 11: Coordinates of A at r units from B  1 −3 


⇒A  , 
A(2 + r cosθ, 5 + r sin θ) 2 2 
−1 1 −5
m= , sin θ = , cosθ =
B 5 26 26
2,5
3/4  13 13 
r C = 1 + cos θ,1 + sin θ 
 2 2 
3x + y + 4 = 0
A
 3 3
3 ⇒ C = − , 
tan θ =  2 2
4
 4r 3r  Sol 13: OT → y = x [O(0, 0), T (2, 2)]
A 2 + ,5 + 
 5 5
OS → y = –x [O(0, 0), S (–2, 2)]
A lies on given line SP → y = x + 4 [as OP = 4]
 4r  3r PQ → y = –x + 4 [as OQ = 4]
⇒ 3 2 +  + 5 + +4=0
 5  5
PR → y = mx + 4
15 + 3r = 0 ⇒ r = –5
3 −1
A[2 – 4, 5 – 3] = [–2, 2] m = tan 150 → =2– 3
3 +1
r = 5 units ⇒ y = (2 – 3 )x + 4
OR → y = mx
Sol 12:
R[4 + 4 2 cos75, 4 2 sin 75]
(-2,-1)
D C  ( 3 − 1) ( 3 + 1) 
R 4 + 4 2 ,4 2 
m  2 2 2 2 
A B (1,1)
R[4 + 2( 3 –1), 2( 3 +1)]
2
Slope of BD = R[2( 3 +1), 2( 3 +1)]
3
⇒y=x
8 . 5 4 | Straight Line

Sol 14 : A Sol 16: A (3, -1)

x-3y=2 x+y-6=0 O
G E
F (0, 0)
C B B C
5x-3y+2=0
(-2, 3) (a, b)
Solving
Orthocentre is (0, 0)
x + y – 6 = 0, x – 3y – 2 = 0 gives A[5,1]
1
x + y – 6 = 0, 5x – 3y + 2 = 0 gives B[2,4] (slope)OA = −
3
x – 3y – 2 = 0, 5x – 3y + 2 = 0 gives C[–1,–1] b−3
(slope)BC = 3 =  …. (i)
 7 5  1 3  a+2
Midpoints are E  ,  , F  ,  , G[2, 0]
2 2 2 2 −3
(slope)OB =
2
Equation of median
2 b +1
1 (slope)AC = =  …. (ii)
y −1 3 a−3
AF ⇒ = 2 ⇒ x + 9y = 14
x −5 9 Solving (i) and (ii), we get

2 b = 3a + 9 and 3b = 2a – 9
y−4 1
BG ⇒ = ⇒x=2 9a + 27 = 2a – 9
x−2 0
36 −108 + 63 45
y +1 7 a= − ; b= = −
CE ⇒ = ⇒ 7x – 9y = 2 7 7 7
x +1 9
 36 45 
⇒ C − , − 
Sol 15:  7 7 
C(-1,3)
Sol 17: Lines are concurrent intersection of 1 and M  3
gives x = 1 y = –1
It lies on  2 ⇒ p – 2 – 3 = 0 ⇒ p = 5
A(0,0)
B(2,-1)
x−6
−4 Sol 18: 1 ⇒ y = 3x + 5 2 ⇒ y =
Slope of line BC = 3
3 1
m1 = 3 m2 =
3 3
Slope of line perpendicular to BC =
4 1
3−
⊥ bisector through A 3 1
tan θ = =
3x 3 3
y= …. (i) 1+
4  3
Slope of line AC = –3
⇒ θ = 30º, 150º
1
Slope of line perpendicular to AC =
3 Sol 19:
⊥ bisector through B
3
A(2, –1) mAB = −
3y = x – 5 …. (ii) 2
Solving (i) & (ii), we get 1
B(0, 2) mBC =
x = –4; y = –3 3
−3
C(3, 3) mCD =
Ans is (–4, –3) 2
M a them a ti cs | 8.55

1
D(5, 0) mAD = A (2, -7)
3
BC || AD and AB || CD B
4x+y=1
⇒ ABCD is a parallelogram
C
⇒ Diagonals are AC & BD 3x-4y+1=0
−1 − 3
Slope of AC → =4 Solving equations we get B
2−3
3x + 1
2 − 0 −2 = 1 – 4x ⇒ 19x = 3
Slope of BD → = 4
0 −5 5 3 7
−2 ⇒x= ,y=
m1 = 4; m2 = 19 19
5
2 3 7
4+ ∴ B , 
5 22 −22  19 19 
tan θ = = =
8 −3 3
1− 2 2
5  3   7 
AB = 2 −  +  + 7
 19   19 
 −22 
φ = tan–1  
 3  (35)2 + (140)2 35
= = 17
19 19
Sol 20: A(–2, 0) ; B(0, 5)
C = (2 + r cosθ, –7 + r sinθ)
5
mAB = r = 29 3(2 + r cosθ) – 4(–7 + r sinθ) + 1 = 0
2 AB
35 + 3r cosθ – 4r sinθ = 0
C = 0 + 29 cos θ,5 + 29 sin θ 
  3 17 4 17
1+ cosθ – sinθ = 0
D =  −2 + 29 cos θ,0 + 29 sin θ  19 19
  19
–3 cosθ + 4 sinθ =
−2 2 −5 17
tan θ = ; sin θ = ; cos θ =
5 29 29 −52
tan θ = –4 or
C = [–5, 7] D = [–7, 2] 89
y+7 −52
Equation of AC ⇒ = –4 or =
x−2 89
y −2
Sol 21: =m 52x + 89y + 519 = 0 or 4x + y = 1
x −3

1 Sol 23:
m−
2 m=m
tan 45 = m=1/2 m=–2/3
m
1+
2 x–2y–3=0
2m − 1
= ±1
m+2
1 m=3/2 3x–2y–5=0
m = +3, m = −
3
Points of intersection is (1, –1) and both the lines
⇒y – 2 = 3x – 9 ⇒ y = 3x – 7 x – 2y – 3 = 0 and reflected Line are equally inclined to
normal on 3x – 2y 5 = 0
1
& y – 2 = − (x – 3) ⇒ x + 3y = 9 2 −2 1
3 m+ −
3 = 3 2
2m 1
Sol 22: 1− 1−
3 3
8 . 5 6 | Straight Line

3m + 2 −7 13
⇒ = ⇒ Slope of BC =
3 − 2m 4 61
33
⇒ 2(6m + 4) = –21 + 14m y −b 13 y+
⇒ = ⇒ 2 = 13
⇒ 2m = +29 x−a 61 x + 13 61
29 61 × 33
⇒ m=+ ⇒ 61y = 13x + 13×13 –
2 2
29 ⇒ 26x – 122y – 1675 = 0
⇒ (y + 1) = (x – 1)
2
⇒ 2y + 31 = 29x Sol 26: The equation of line passes through point of
intersection of x+3y+y=0 and 3x+y+4=0 is
Sol 24:
3x+y+4 + λ (x+3y+4) = 0
(-8, 12)
(λ+3)x + (1+3 λ)y + 4 + 4 λ =0
The obtained line is equally inclined to axes, then
4x+7y+13
Slope of line = ± 1
λ+3 λ+3
(a, b) − 0
= and − −1
=
1 + 3λ 1 + 3λ

a+8 b − 12 65 ⇒ λ + 3 = - 1 -3 λ or λ + 3 = 1 +3 λ
= = –2 .
4 7 65 ⇒ λ = -1 or ⇒λ=1
a+8 b − 12 Eqn of line is (-1 + 3)x + (1 - 3)y + 4 - 4=0
⇒ = = –2
4 7
⇒ 2x − 2y= 0 ⇒ x= y and
⇒ a = –16 & b = –2
(1+3)x + (1 + 3)y + 4 + 4=0
Image ≡ (–16, –2)
⇒ 4x + 4y + 8 =0
⇒ x+y+2 = 0
Sol 25: A

Sol 27: x(a + 2b) + y(a + 3b) – a – b = 0


3x-2y+6 4x+5y=20 a(x + y – 1) + b(2x + 3y – 1) = 0
(1,1)
Fixed point is the intersection of given 2 lines
B C x + y = 1 & 2x + 3y = 1
 10 84 
A=  ,  ⇒ y = –1; x = 2; P (2, – 1)
 23 23 
B = (a, b) ⇒ 3a – 2b + 6 = 0 Sol 28:
b −1 5 (2, -1)
= ⇒ 4b = 5a – 1
a−1 4
Solving these, we get
60
6a + 12 = 5a – 1
x+y=2
−33
a = –13 b= −1 1
2 Distance of vertex from line is =
84 2 2
−1 Length of base = 2 a
−61
AB slope = 23 =
10 13
−1 2
23 =2
3
M a them a ti cs | 8.57

2 1 1 1
1 2 m+ − −
sin 60º = ⇒a= = 3
2a 3 3 = 3 7
m 1
1− 1−
3 21
Exercise 2
3m + 1 −10 3m + 1 −1
⇒ = ⇒ =
Single Correct Choice Type 3−m 20 3−m 2

⇒ 6m + 2 = m – 3 ⇒ m = –1
Sol 1: (C) 3x – 8y – 7 = 0
 −1 7 
3(0) − 8( −1) − 7 =
1 It passes through  , 
(A)  different sides  10 10 
3(0) − 8(0) − 7 =−7  7
y−
10 = –1⇒ x + y = 7 – 1 ⇒ 5x + 5y = 3
3(0) − 8(1) − 7 =−15
(B)  different sides 1 10 10
3(3) − 8(0) − 7 =2  x+
10
3( −1) − 8(1) − 7 =−2 
(C)  same sides Sol 5:(C) X = x – h = 3 – 4 = –1
3(3) − 8(7) − 7 =−54 
Y = y – k = –5 + 3 = –2
Sol 2: (A) 3x – 4y + 7 = 0; 12x + 5y – 2 = 0
x y
p1p2 + q1q2 = 36 – 20 = 16 > 0 Sol 6:(A) + =1
a b
3x − 4y + 7 −(12x + 5y − 2) 3 4
= + = 1 are concurrent at a fixed point.
5 13 5a 5b
39x – 52y + 91 = –60x – 25y + 10 3 4
Point is x = and y =
5 5
99x – 27y + 81 = 0 ⇒ 11x – 3y + 9 = 0
3 4 
⇒  , 
Sol 3: (A) 5 5 
(5,3)
(1,2) C
A Sol 7: (D) P (1, 0) ; Q (–1, 0) ; R (2, 0)
 
2SP2 = SR2 + SQ2
B
(x+1)2+y2+(x–2)2+y2 = 2(x–1)2+2y2
⇒ x2+1+2x+x2+4–4x=2(x2+1–2x)
C’
(5,-3)

⇒ 5 – 2x = 2 – 4x
C’ is the reflection of C w.r.t. x-axis. 3
2x = –3 ⇒ x = −
∴ Eq. of AB = Eq. of AC’ 2

y +3 2+3 5 Which is a straight line parallel to y-axis.


AC’ = = −
x −5 1 −5 4
Sol 8: (B) ax ± by ± c = 0
⇒ 5x + 4y = 13
x y
⇒ ± ± 1
=
Sol 4: (C) ca cb
x-7y+5=0 (m = 1/7)
1 c c  2c2
∴ Area = 4  ⋅ ⋅  =
2 a b ab

x+3y=2 (m=-1/3)
 Sol 9: (A) Lets assume 2 lines are x and y axis so the
distances from 2 lines are …. .
8 . 5 8 | Straight Line

y
2a 1 + m2
2r = =2a ⇒r=a
1 + m2
x
Sol 13: (A) 12x2 – 16xy + 9xy – 12y2 = 0
4x(3x – 4y) + 3y(3x – 4y)
(4x + 3y)(3x – 4y) = 0
|x|+|y|=1
2y=x
It forms a square.
3x-4y=0

Sol 10: (C) Given 3 lines are concurrent m

1 2 9 3 3 1
3 −5 5 = 0 m− −
4
= 4 2
a b 1 3m 3 1
1+ 1+ ×
4 4 2
–5 – 5b – 2(3 – 5a) + 9(3b + 5a) = 0
–5 – 5b – 6 + 10a + 27b + 45a = 0 4m − 3 2
= ⇒ 44m – 33 = 8 + 6m
55a + 22b – 11 = 0 4 + 3m 11
41x
5a + 2b = 1 38m = 41 ⇒ y =
38
The straight line 5x + 2y = 1 passes through (a, b)
Sol 14: (C) A = (4, –1)
Sol 11: (A)
B is symmetric to A w. r. t. y = x
B is [–1, 4]

AB = 25 + 25 = 5 2

Sol 15: (D) A(–4, 0); B(0, 2); C(–3, 2)


C(-3,2)

px2 + 2axy + qy2 = r (homogenising) m

px2 + 2axy + qy2 = r[ax + by]2 (-4,0)A B(0,2)


D
x2(p – ra2) + y2(q – rb2) + 2axy – 2abxy =0 (-2,1)

Lines are perpendicular i. e. a + b = 0 y −1 −1


Eq. of CD ⇒ =
p + q = r(a2 + b2) x+2 1
y – 1 = –2 – x
Sol 12: (B) y – y1 = m(x – x1) ± a 1 + m2 x + y = –1
1
C1 m1 mAB = mAC = +2
2
1
2r m−
2−m 2 = 2m − 1
C2 ⇒ =
1 + 2m m 2+m
m2 1+
C1 − C2 2
2r =
1 + m2 ⇒ 4 – m2 = 4m2 – 1 ⇒ m = ± 1
y
=1
x+4
M a them a ti cs | 8.59

Eq. AD y=x+4 Sol 20: (D) x2 + y2 – 4xy = 0


Intersection point –1 – x = x + 4 ⇒ 2x = –5; 2y = 3 m1 . m2 = 1
+4
m 1 + m2 = = +4
Sol 16: (B) 2x – y + 5 = 0 m=2 1
x + y –5 = 0 m = –1 ⇒ m1 = 2 – 3
1 m2 = 2 + 3 & m3 = −1
x – 2y – 5 = 0 m=
2 It forms an equilateral triangle.
1 1
−1 − −2
3 2
∑ tan θ = + + 2
1−2 1 1+1
1−
2
3 3 27
=3+3+ = 6+ =
4 4 4

Sol 17: (B) y = 0.2x


m = 0.2
tan θ = 0.2
2 tan θ 0.4 0.4 5
tan 2θ = = = =
2
1 − tan θ 1 − 0.04 0.96 12 Sol 21: (D) y = x (A lies on line 1)
5x A(x, x)
⇒y=
12 y = 7x (B lies on this line  2 )
(y, 7y)
Sol 18: (B) x2 + y2 – 10x – 14y – 151 = 0
M(–7, 2) OA = OB ⇒ x 2 = y 50

49 + 4 + 70 – 28 – 151 < 0 x = 5y

Point is inside 7y − x 2y −1
Slope of line AB = = =
y−x −4y 2
A(–7, +2) and Centre (5, 7)

r= 25 + 49 + 151 = 15

AC = 144 + 25 = 13 Previous Years’ Questions


(r – AC) = 2 (minimum distance)
Sol 1: (A) The point O(0,0) is the mid point of A(-a,-b)
and B(a,b).
Sol 19: (D) ax2 + 2hxy + by2 = 0
Therefore, A,O,B are collinear and equation of line AOB
Image by line mirror y = 0
b
is y = x
a
Since, the fourth point D(a2, ab) satisfies the above
equation. Hence, the four points are collinear.

Sol 2: (C) Given lines, x+2y–3=0 and 3x+4y–7=0 intersect


at (1,1), which does not satisfy 2x+3y–a=0 and 4x+5y–
Replace y by (–y) 6=0. Also, 3x+4y–7=0 and 2x+3y–4=0 intersect at
ax2 – 2hxy + by2 = 0 (5,-2) which does not satisfy x+2y–3=0
4x+5y–6=0. Lastly, intersection point of x+2y-3=0 and
2x+3y-4=0 is (-1,2) which satisfy 4x+5y-6=0. Hence,
only three lines are concurrent.
8 . 6 0 | Straight Line

Sol 3: (C) Let B, C, D be the position of the point A(4, 1) Sol 9: (D) Since, S is the mid point of Q and R.
after the three operations I, II and III respectively. Then,
7 + 6 3 − 1 13
B(1, 4), C(1+2, 4) ie, (3, 4). The point D is obtained from ∴ S≡ , = ,1
2 2 2
C by rotating the coordinate axes through an angle π/4
in anticlockwise direction. 2 −1 2
Now, slope of PS = m = = −
Therefore, the coordinates of D are given by 2 − 13 / 2 9

π 1 Now, equation of the line passing through (1, -1) and


x = 3cos= − parallel to PS is
4 2
π π 7 2
and Y =3sin + 4 cos = y + 1 =− (x − 1)
4 4 9
2
⇒ 2x + 9y + 7 =0

 1 7 
∴ Coordinates of D are  − ,  Sol 10: (D) Let the vertices of triangle be A (1, 3) , B(0,
 2 2 0 ) and C (2, 0).
Here AB = BC = CA = 2.
Sol 4: (A) The point of intersection of three lines are
A(1, 1), B(2, -2), C(-2, 2). Therefore, it is an equilateral triangle. So the incentre
coincides with centroid.
Now, | AB |= 1+9 = 10 ,| BC |= 16 + 16 = 4 2,
and | CA |= 9 +1 = 10 0 +1+ 2 0 + 0 + 3   1 
∴ I= ,  =  1, 
 3 3   3
 
∴ Triangle is an isosceles
Sol 11: (A) On solving equations 3x + 4y = 9 and y =
Sol 5: (A) By the given condition, we can take two mx + 1, we get
perpendicular lines as x and y axes. If (h, k) is any point 5
on the locus, then |h|+|k|=Sol 1 Therefore, the locus is x=
3 + 4m
|x|+|y|=1 This consist of a square of side 1.
Hence, the required locus is a square. Now, for x to be an integer
3 + 4m = ± 5 or ± 1
Sol 6: (C) Orthocentre of right angled triangle is at
the vertex of right angle. Therefore, orthocentre of the The integral values of m satisfying these conditions are – 2
triangle is at (0, 0). and – 1

Let y cos x cos(x + 2) − cos2 (x + 1)


Sol 7: (D)= Sol 12: (D) Let lines OB : y = mx
= cos(x + 1 − 1)cos(x + 1 + 1) − cos2 (x + 1) CA : y = mx + 1
2 2 2
= cos (x + 1) − sin 1 − cos (x + 1) BA : y = nx + 1
y = − sin2 1. and OC : y = nx
This is a straight line which is parallel to x-axis It passes
The point of intersection B of OB and AB has x-coordinate
through ( π / 2, − sin2 1).
1
.
Sol 8: (D) Slope of line x+3y=4 is -1/3 m–n y

And slope of line 6x-2y=7 is 3. A


 −1 
Here, 3 ×   = −1 C D B
 3  x’ x
Therefore, these two lines are perpendicular which O
show that both diagonals are perpendicular.
Hence, PQRS must be a rhombus.
y’
M a them a ti cs | 8.61

Now, area of parallelogram 8 14 30


2× + k ⇒
= k
=
OBAC = 2 × area of DOBA 5 5 5

1 1 1 ⇒k=
6
= 2 × × OA × DB = 2 × ×
2 2 m–n
1 1 Sol 15: (A) Sides OA = OB = 2 and AB = 2 2
= =
m–n |m–n| X-coordinates of incentre of ∆ OAB

Sol 13: (C) The equation of the line passing through


points P(-1, 0) and Q(0, 0) is;
Y = 0 … (i)

R(3, 3 3)

Q(0, 0)
P(-1, 0) 2 2 ×0 + 2 × 0 + 2 × 2

2 2 +2+2
Equation of the line passing through points Q(0, 0) and
R(3, 3 ) is; 4 2
= = = 2− 2
4 + 2 2 2+ 2 2
y −0 3 3 y 3 3
= ⇒=
x−0 3−0 x 3
 6 + 7 −1 + 3   13 
⇒ y =3x  … (ii) Sol 16: (B) S ≡  , ; ≡  , 1
 2 2   2 

Therefore, the equations of the bisector of the angle 2 −1 2


Slope of PS = = −
PQR is 13 9
2−
2
y 3x − y 3x − y
=± ⇒ y=±
1 2 3+1 2

3x − y 3x + y
⇒ y = or y=

2 2
⇒ 3x
= 3x + y 0 .
− 3y 0 or =

2
Sol 14: (C) Point T is given by Equation of line passes through having slope −
9
 3 × 2 + 2 × 1 3 × 4 + 2 × 1   8 14  2
T ≡
 3+2
,
3+2
 ≡ ,
 5 5 
 y + 1 =−
9
( x − 1) ⇒ 9y + 9 =−2x + 2
⇒ 2x + 9y − 7 =0

Sol 17: (C) The given equation of lines are


4ax + 2ay + c = 0 and 5bx + 2by + d = 0
Since, these lines intersect in fourth quadrant and point
is equidistant from axes, then

Which lies on the line 2x + y = k


8 . 6 2 | Straight Line

The point can be of form (k, -k), k > 0 On solving, we get


⇒ 4ak − 2ak + c =0 and 5bk − 2bk + d =0 B ≡ (1, 2 )
⇒ 2ak + c =0 and 3bk + d =
0 We know that diagonals in rhombus bisect each other, so
c d D ≡ ( −3, − 6 )
⇒− =−
2a 3b
Since BC || AD
⇒ 3bc − 2ad =
0
⇒ Equation of AD 7x − y + λ1 = 0 , Passes through
Sol 18: (B) (2x - 3y + 4) + k (x – 2y + 3) = 0 (-3, -6)

Is a family of equation passes through T (1, 2) ⇒ 7x − y + 15 =0

From figure

Similarly, AB || DC
PT = P’T
⇒ Equation of DC x – y + λ2 = 0, passes through
(P' T )
2
⇒ PT2 = (-3, -6)
⇒ (h − 1 ) + (k − 2 ) =( 2 − 1 ) + ( 3 − 1 )
2 2 2 2
⇒ x − y −3 =0
⇒ (h − 1 ) + (k − 2 ) = 1 + 1 = 2
2 2

1 8  7 4
⇒ ( x − 1) + ( y − 2) =
2
2
2 ⇒ C  , −  and A  − , − 
3 3  3 3
Locus is a circle.

Sol 19: (C) Equation of AB, x + y = 41


JEE Advanced/Boards
On line x = 1, there are 39 points inside ∆ OAB
Similarly Exercise 1
Sol 1: OA = 2AB = 4BC

A 1 2
S G H
(7/2, 5/2) (2, 4)

2+7 4 +5
G= , = (3, 3)
3 3
A = (3, 3) = (a, a)
On line x = 2, there are 38 points inside ∆ DAB  3a a
B =  ,a − 
Total points  2 2 
= 39 + 38 + ….. + 2 + 1
 a a a a
39 ( 40 ) C =  a + + ,a − + 
= = 780  2 4 2 4
2  a a a a a a 
⇒  a + + + + ...,a − + − ....x 
 2 4 8 2 4 8 
Sol 20: (B) Let two sides AB and BC be x – y + = 0 and
7x – y – 5 = 0 respectively.
M a them a ti cs | 8.63

  A = (–2, 3)
 a a   2a 
N=  ,  = 2a,  ⇒ Eq. of AB is x + y = 1  … (i)
1 − 1 1 + 1   3
  Eq. of AC is x + 2y = 4  … (ii)
 2 2
8a 3 5
α+β= =8 S=  , 
3 2 2
⇒ Eq. of perpendicular bisector of AB and AC are
Sol 2: y = x + 1 … (iii)
A(-3,1)
1
y = 2x –  … (iv) respectively.
2
(,) Eq. (i) and (iii) ⇒ E = (0, 1) [Midpoint of A, B]

7x-4y=1 2x+y=3 Eq. (ii) and (iv) ⇒ F = (1, 3/2) [Midpoint of A, C]



(2+3,2b-1) ⇒ B = (2, –1) and C = (4, 0)
B m
y −0 1−0
Eq. of median to AB is = ⇒ x + 4y = 4
7(2α + 3) – 4(2β – 1) = 1 x−4 0−4
14α – 8β + 24 = 0 ⇒7α – 4β + 12 = 0 Eq. of median to AC is
3
2α + β – 3 = 0 y +1
+1
= 2 ⇒ 5x + 2y = 8
α = 0, β = 3 ⇒ C = (3, 5) x−2 1−2
2
Slope of AC =
3 Sol 4: A(-8,5)
7 7 2
m− −
4 = 4 3 ⇒ 4m − 7 = 13 = 1  
7m 7 2 7m + 4 26 2
1+ 1+ . m
4 4 3
8m – 14 = 7m + 4 ⇒ m = 18
B C
y −5 2 (1, -7)
Equation of AC = = (-15, -19)
x −3 3
24
⇒ 3y – 9 = 2x mAB =
7
y −5
Equation of BC = = 18 12 −4
x −3 mAC = =
−9 3
⇒ 18x – y = 49
24 4
−m m+
2x + y = 3; 18x – y = 49 7 3
=
26 −11 24m 4m
x= ,y= 1+ 1−
10 5 7 3
24 − 7m 3m + 4
 26 −11  ⇒ =
B=  ,  24m + 7 3 − 4m
 10 5 
−11 ⇒ 72 – 117m + 28m2 = 72m2 + 117m + 28
−1
y −1 −16 −4
Equation of AB = = 5 = = ⇒ 44m2 + 234m – 44 = 0
x+3 26 28 7
+3
10 ⇒ 22m2 + 117m – 22 = 0
⇒ 7y – 7 = –4x – 12 ⇒ 4x + 7y + 5 = 0 ⇒ (11m – 2) (2m + 11) = 0
⇒ m = 2/11, –11/2
Sol 3: x – y = 4 [Line perpendicular to AB]
y −5 2 11
2x – y = 5 [Line perpendicular to AC] = or –
x+8 11 2
8 . 6 4 | Straight Line

⇒ 11y – 55 = 2x + 16 or 2y – 10 = –11x – 88 ⇒ y – 7x – C2 = ±(5y – 5x – 5C1)


⇒ 2x – 11y + 71 = 0 or 2y + 11x + 78 = 0 ⇒ 4y+2x–5C1+C2=0 and 6y – 12x – 5C1 – C2=0
⇒a = 11; c = 78 go through point (1, 2) (given)
⇒ 10 – 5C1 + C2 = 0 and 5C1 + C2 = 0
Sol 5: 4x+3y=12 –2C2 = 10 ⇒ C2 = –5

9/5 3 C1 - C2
=
9 –5C1 = 5 ⇒ C1 = 1
2
a +b
2
5
m ⇒ Diagonals are
4x+3y=3
4y + 2x – 10 = 0 ⇒ 2y + x – 5 = 0
(-2, -7)
6y – 12x + 0 = 0 ⇒ y = 2x
(–2+r cosθ, –7+r sinθ) lies on 4x + 3y = 3 If A vertex is at y-axis ⇒ x = 0
–2+(r+3)cosθ, –7+(r+3)sinθ lies on 4x+3y = 12 y = 2(0) = 0 ⇒ (0, 0)
4(rcosθ – 2) + 3(rsinθ – 7) = 3  …. (i) 2y + 0 – 5 = 0 ⇒y = 5/2 ⇒(0, 5/2)
4((r+3)cosθ – 2) + 3((r+3)sinθ–7)=12  ….(ii)
On solving (i) and (ii), we get Sol 7: O(0, 0) ; A(6, 0) ; B(3, 3)

–12cosθ – 9sinθ = –9 B(3,3)


⇒ 4cosθ + 3sinθ = 3 23 I 23
⇒ 9+16cos2θ – 24 cosθ = 9 – 9 cos2θ A
O (6,0)
⇒ 25 cos2θ = 24 cos θ
⇒ cos θ = 0 or cos θ = 24/25 d(P, OA) = d(P, OB) if P lies along the angular bisector of
−7 AOB. Similarly if d(P, OA) = d(P, AB) then P lies along the
⇒ sin θ = 1 or sin θ = angular bisector of OAB
24
−7 Hence the region where
⇒ m = ∞,
24
d(P, OA) ≤ minimum {d(P, OB), d(P, AB)} is OAI.
y+7 −7
⇒ = or ∞ 1  OA AOB 
x+2 24 Area of OAI = OA  tan 
2  2 2 
⇒ x = –2 or 24y + 168 = –7x – 14
⇒ 7x + 24y = 182 or x = –2
=
1
2
(
6 3tan150 ) = 9 (2 − 3 )
⇒a+b=2+3=5
Sol 6:
1 Sol 8:
(4,-1)
A
(0,a) x=1 A
(1,2) x-y=1
7 (2,4-a) (1,) 1
2
 O
 1 (1, )
(, -1) B C
Sides of the rhombus are
Reflection of (4, –1) about both of bisectors lie on the
y = x + C1; y = 7x + C2 line BC
Eq. of diagonals So reflection about x – 1 = 0 is A’(2, 1)
y − 7x − C2  y − x − C1  Reflection about x – y = 1 is A’’(0, 3)
⇒ = ±  
50  2  Hence Eq. of BC is y – 3 = 2x
M a them a ti cs | 8.65

B and C are intersections of BC with Sol 10:


x – y – 1 = 0 and x = 1 respectively A(t, -t)

⇒ B = (4, 5); C = (1, 5) (-1/2)


(-1)
⇒ Eq. of AB is x – 2y = 6 F
O
E
(4)
Eq. of AC is 2x + y = 7
B C
Sol 9: The equation of line D 1

(y – 2) = m(x + 1) …. (i) mBC = 1


 −2  mAC = 4 ⇒ y = 4x – 5t  … (i)
⇒A=  − 1,0  ; B = (0, 2 + m)
 m  −1
mAB= ⇒ 2y+ x + t = 0  …(ii)
P 2
(-1,2) B Solving AB and BE we get x = –4y
Q
t  −4t t 
A t = 2y ⇒ y = ⇒B=  , 
2  2 2
CF and AC ⇒ y = 2x
Assume Q = (h, k)
 5t 
Q is on the line 2x = 5t ⇒ C =  ,5t 
2 
⇒ k = 2 + m(h + 1)
G = (h, k)
PQ = (h+ 1)2 + (k − 2)2 5t 4t t
3h = t + − ; 3k = t + 5t +
2 2 2
= (h+ 1)2 + m2 (h+ 1)2 = | h+ 1 | m2 + 1 t 3t
h= ;k= ⇒ k = 3h
2 2
2
 −2  2
PA = 
2
− 1 + 1 + 2 = 1 + m2
m  m Sol 11: y = m1x; y = m 2x
y2 – (m1 + m2)xy + m1m2x2 = 0
PB = 1 + m2
m1 x1 − y1 m2 x1 − y1
It’s given that PA, PQ, PB are in H. P. = =δ
1 + m12 1 + m22
2 1 1
⇒ = +
PQ PA PB (mx1 – y1)2 = d2(1 + m2)
2 m 1 1 m2x12 + y12 – 2mx1y1 = d2 + d2m2
⇒ = +
| h+ 1 | m2 + 1 2 1 + m2 1 + m2 m2(x12 – d2) + m(–2x1y1) + y2 – d2 = 0
2 1 k −2 2x1 y1 y 2 − δ2
⇒= +1 m 1 + m2 = ; m 1m 2 =
| h+ 1 | 2 h + 1 x12 − δ2 x12 − δ2

⇒ 2|x + 1| + |y – 2| = 4 2x1 y1 y12 − δ2


y2 – xy + x2 = 0
If m < 0 then y =2x x12 −δ 2
x12 −δ 2

If m > 0 then 2x + y = 4 (If x > 1 and y > 2) (x12 – d2)y2 + (y12 – d2)x2 – 2x1y1xy = 0

= 4 (If x < 1 and y < 2)


8 . 6 6 | Straight Line

Sol 12: A(1,-2) β −5


3− 
 2  11 − β
⇒ –1 = ⇒ –1 =
β +1 3−β
2x+y=0 2− 
P(2,3)  2 
x-y=3
14
⇒ β – 3 = 11 – β ⇒ β = =7
(,-2)B x+py=q C(,-3) 2
⇒ 7 + 4p = q
If P is centroid
⇒ –13 + 26p = 5q
⇒α+β+1=6 …(i)
⇒ 35 + 20p = 5q
⇒ –2 – 2α + β – 3 = 9
⇒ –48 + 6p = 0
⇒ 2α – β = –14 …(ii)
⇒ p = 8 ; q = 39
⇒α+β=5
⇒ p + q = 47
⇒ α = –3; β = 8
B and C lie on line BC Sol 13: x2 – 3y2 – 2xy + 8y – 4 = 0
α−
⇒ α 2pα
− 2p α= q
=q Assume eq. are
+ p(β − 3) =q
⇒ β + p(β − 3) =
β q (x + ay + c)(x + by + d) = 0
⇒ –3 + 6p = q ⇒ db = – 3; dc = – 4
⇒ 8 + 5p = q ⇒ a+b = –2; bc+ad = 8
⇒ p = 11; q = 63 ⇒ p + q = 74 ⇒ (a–b)2= 4 + 12 =42d + c = 0
If P is orthocentre a – b = 4⇒ d = –c
m(AP) = 5 ⇒ a = 1⇒ –c2 = –4
−1 −1 ⇒ d = –3 c = ±2
mBC = = ⇒p=5
5 p bc – ac = 8
m(AC) = 1 8
(b – a) =
3 + 2α c
m(BP) = –1 =
2−α ⇒ c = –2, ⇒ d = 2
⇒ α = –5 Eq. are (x + y – 2)(x – 3y + 2) = 0
⇒ –5 – 10(–5) = q ⇒ q = 45 x-3y+2=0
⇒ p + q = 50 A
-2 (1,1)
If P is circumcentre
(0,0) x y
+ =1
 α + 1 −2 − 2α   α + 1 
Mid point of AB =  , =  , − (α + 1)  (-5,-1) 2 2
 2 2   2 
1 3 − ( −α − 1)
m(PM) =
= 
2  α +1
2− 
 2  For A ⇒x + y = 2 ⇒ x = 2 – y
1 2(4 + α )
⇒ = ⇒2 – y – 3y + 2 = 0
2 (3 − α )
−13 4y = 4 ⇒ y = 1
⇒ 3 – α = 16 + 4α⇒α =
5 ⇒x = 2 – 1 = 1

1 + β β −5 1
Mid point of AC =  , Slope of L1 [(–5, –1) to (0, 0)] =
 5
 2 2 
M a them a ti cs | 8.67

x y −2
Slope of + =1⇒ = –1
2 2 2 1 3 1
1
Range (–1, 1/5) Area = 4 12 1
2
1 18
(∴ Third line is go through (0, 0) and for triangle parallel − 1
5 5
x y
to + = 1 meet at infinity)
2 2  
 1 = 1 1  12 − 18  − 3  4 + 1  + 1  18.4 + 1.12  
 −1,  = (a, b) 2  5   5   5 5 
 5
1  42 3.21 72 + 12 
 1  =  − + 
⇒  a + 2  = –1 + 52 = –1 + 15 = 24 2 5 5 5 
 b 
1  84 + 42 − 63  63
=   = 10
Sol 14: 2x + 3xy – 2y – 10x + 15y – 28 = 0
2 2 2 5 

y = 3x ⇒ a1 = 9 + 81 = 3 10
(-1/5,18/5)
36 9 3
C B ⇒ a2 = + = 45 = 45
(4,12) 25 25 5
A
(1,3) 2 2
 21   18  441 + (42)2 21
⇒ a3 =   +  12 −  = = 5
 5   5  25 5
2x2+9x2–2(9x2)–10x+15(3x)–28 = 0
Incentre will be
–7x2 + 35x – 28 = 0
x2 – 5x + 4 = 0 ⇒ x = 1, 4 1 × 21 5 4 × 3 5 1
+ − × 3 10
5 5 5 ,
∴ y = 3, 12
15 10 3 5 21 5
+ +
∂p 5 5 5
= 4x + 3y – 10 = 0
dx
∂p 3 × 21 5 12 × 3 5 18
= –4y + 3x + 15 = 0 + + 10
dy 5 5 8
18 24 5 + 15 10
25y = 90 ⇒ y = 5
5
54  11 5 − 10 33 5 + 18 10 
10 − =  , 
x= 5 = −1
 8 5 + 5 10 8 5 + 5 10 
4 5
3 Radius = distance of incentre from any of the sides.
−1
⇒ m1 = 3, m2 = 5 = , 4 − 3x 18 10 + 3 10 21
−6 2 = =
5 10 10(8 5 + 5 10 ) 8 5 + 5 10
18
12 −
5 42 21(8 5 − 5 10 ) 3
⇒ m3 = = =2 = = (8 5 − 5 10 )
1 21 70 10
4+
5
3−2 1 Sol 15: y = mx + c
⇒ tanq1 = =
1+6 7  y − mx   y − mx 
3x2–y2– 2x   +4y   =0
1  c   c 
3+
⇒ tanq2 = 2 = –7
2xy 2mx2 4y 2 4mxy
3 3x2–y2 – + + − =0
1− c c c c
2
1 50
⇒ cotq1 + cotq2 + cotq3 = 17 + +0=
7 7
8 . 6 8 | Straight Line

 4m 2  Sol 17: x2 + 4xy + 4y2 – 5x – 10y + 4 = 0


x2  3 + 2m  + y2  4 − 1  + xy  − −  =0
 c  c   c c ⇒ y2 – 4y + 3 = 0
a
Now = –1 ⇒ y2 – 3y – y + 3 = 0 ⇒ y = 1, 3
b
For y = 1
3c + 2m
⇒ = –1 ⇒ x2 + 4x + 4 – 5x – 10 + 4 = 0
4−c
x2 – x – 2 = 0 ⇒ x = –1, 2
⇒ 3c + 2m = c – 4
For y = 3
⇒ 2c + 2m = –4
⇒ x2 + 12x + 36 – 5x – 30 + 4 = 0
⇒ c + m = –2
⇒ x2 + 7x + 10
Point is (1, –2)
⇒ x = –5, –2
for 3x2 + 3y2 + 2x + 4y = 0
equation will be

 3m  24   −4m 2  2
x2  3 −  + y  − 1  + xy  +  =0
-1
 c   c   c c

3c − 2m The points are (–1, 1) (2, 1)


= – 1 ⇒3c – 2m = c – 4
4−c
(–5, 3) (–2, 2)
c – m = –2 ⇒ point is (–1, –2)
−1 1
y
Sol 16: y = m(x – 1) – +x=1 2 1
m 1 1
−2 3 = [–1+6–5–6–(2–2–15–3)]
2 2 2
 y  y  y −5 3
⇒ x2+y2+6x  x −  –10  x −  + 1 x −  = 0
 m  m  m −1 1

6xy 10y 2 1
⇒ x2 + y2 + 6x2 – – 10xy + + x2 = [–6 – (–18)] = 6 units
m m 2
Diagonals are = 49 + 4 , −1 + 4 = 53 , 5
y2 2xy
+ – =0
m2 m
Sol 18: 6x2 – xy – y2 + x + 12y – 35 = 0
 10 1   6 2
⇒ 8x2 + y2 1 + + + xy  − − 10 −  = 0 (y – m1x – c1)(y – m2x – c2) = 0
m 2 m m
 m   
y2 – m2xy – c2y – m1xy + m1m2x2
a 8
⇒ = = –1 + m1xc2 – c1y + c1m2x + c1c2
b 10 1
1+ +
m m2 m1m2x2 + y2 + xy(–m2 – m1) + x(m2c1 + m1c2)

1 10 + y(–c1 – c2) + c1c2 = 0


⇒ –8 = + +1
m2 m m1m2 = –6; m1 + m2 = –1
⇒ –9m2 = 1 + 10m This gives m1 = 2 and m2 = –3, –3, 2
⇒ 9m2 + 10m + 1 = 0 ⇒ c1c2 = 35
⇒ (9m2 + 9m + m + 1) = 0 ⇒ c1 + c2 = 12
⇒ 9m(m + 1) + 1(m + 1) = 0 ⇒ m2c1 + m1c2 = –1
−1 ⇒ –3c2 + 2c1 = –1
⇒ m = –1,
9 ⇒ c2 = 5, c1 = 7
⇒ y = 1 – x, 9y = 1 – x
M a them a ti cs | 8.69

y = –3x + c1 …. (i) i. e. α = 2, β = 3


y = 2x + c2 …. (ii) OA + OB + OC
y = –3x + 7 …. (iii) 8 1 + m2 m2 + 1
= + β + α 1 + m2
y = 2x + 5 …. (iv) 1+m m2
(, )
8 1 + m2 3
(1) = + 1 + m2 + 2 1 + m2
1+m m
(0,0)
(3) = 4 2 +3 2 +2 2 = 9 2
(-, - )
Equation is y = mx ⇒ y = x
(4) (2)

⇒ 3α + β + 7 = 0 Sol 20: Area(ABC) = 3 Arc (AMB)


3α + β = c1 In ∆ACB A
⇒ c1 = –7 AF=BF=CF=
AB
= 30
2
y = 2a + 5
CM:MF=2:1 E F
–y = –2a + c2 M
⇒ MF =
10
c2 = –5
By Apollonius theorem C D B
Combined equation will be
6x2–xy – y2 + x(–1) + y(–12) – 35 = 0
2 2
AM + BM = 2 CF + AF ( 2 2
)
AM2 + BM2 =
2000

Sol 19: Let ∠AMB =


θ

, MBE − MAD 2 −1 1


tan
= θ = =
B (/m,) 1 + MBEMAD 1 + 2 3
(,m)
A 33 11
⇒ cosθ
⇒Cos
Cosθθ and
and sinθ
Sin
Sin
θθ==
=
10
10 10
10
(0,0) (y,my) In ∆AMB
y = mx AM2 + BM2 − AB2
cosθθ =
Cos
8 2AM.BM
y=
1+m 3 2000 − 3600
=
OA. OB. OC = 48 10 2AM.BM

β2 1600
y 2 + m2 y 2 β2 + α2 + α2m2 = 48 ⇒ AM.BM =
− 10
6
m2
1
8
αβ
(1 + m2 )3/2
= 48
=
Area of ∆AMB
2
( AM)(BM) Sin θ
(1 + m) (m)
1 1600 1 400
= × 10 × =
⇒ (1 + m2)3/2 = 2 m(1 + m) 2 6 10 3
⇒ (1 + m2)3 = 2m2(1+m2+2m) Area of ABC 3 × Area of ∆AMB
⇒ m + 1 + 3m + 3m = (m + m + 2m )
6 2 4 4 2 3 2
400

= 400 Sq. units
=
m + m + m + 1 = 4m
6 4 2 3
3
⇒m=1
β 3
f(α, β) = − + (3x − 2y)2 + e(x − 2) + 2(y − 3) = 0
α 2
8 . 7 0 | Straight Line

Sol 21: c2 = 2; c=± 2


(0,24)
30 5y + x = ± 5 2
24
A(0,3)
(0,0) 18 (18,0)
x+y=3 7x-y+3=0
18 24
Centroid = , = (6, 8)
3 3 B (,3-) C(,7+3)
D(1,-10)
 18 24 
Circumcentre =  ,  = (9, 12) mBD = mBC
 2 3 
Incentre 13 − β 7α + 13
=
 24.18 18.24   24.18  β −1 α −1
=  , =  ,6  = (6, 6)
 30 + 24 + 18 30 + 24 + 18   72  13α–13–aβ+β = 7aβ–7α+13β–13
6 6 1 8aβ = 20α – 12β
1
Area = 6 8 1
2 β2 + β2 = α2 + (7α )2
9 12 1
1 1 2b2 = 50a2
= [6(–4)–6(6–9)+1(72–72)] = [–24 + 18] = 3 sq. units
2 2 β = ±5α
Case I: 40a2 = 20α – 60α
Sol 22:
40a2 = –40α
(1,3)
(,2+3) α = –1; β = –5
y + 10 13 + 5
(5,1) = = –3
(,2+c) x −1 −6
y + 3x = –7
Case II: β = –5α
 2β + c − 1   2β + c + 3 
    = –1 –40a2 = 80α
 β −5   β −1 
α = –2; β = +10
b2 – 6β + 5= –(2β + c – 1)(2β + c – 3)
y + 10 3 1
= =
α+β 2α + 2β + 2c x −1 9 3
= 3; =2
2 2 3y + 30 = x – 1
α + β = 6; α+β+c=2 x – 3y = 31
c = –4
b2 – 6β + 5 = –(2β – 5)(2β – 7) = –(4b2 – 24β + 35) Sol 24: Let slope of BC is m, then equations is
y + 10 = m(x -1)
5b2 – 30β + 40 = 0
∠ACB =
∠ABC
β = 2, 4 A(0,3)
tan ∠ACB =tan ∠ABC
Coordinates are (2, 0) and (4, 4) 7x-y+3=0 x+y-3=0
7 −m m − (1)
=
Sol 23: 5x – y = 1 1 + 7m 1 + m ( −1) C
 
B(1,-10)
−1
y= x+c 7 −m m +1
5 ⇒ =
1 + 7m 1 − m
A(0, 0) ; B(0, c) ; C(5c, 0)
1 ⇒ 7 − 7m − m + m2 =1 + m + 7m2 + 7m
× 5c2 = 5
2
M a them a ti cs | 8.71

Sol 26:
⇒ 6m2 + 16m − 6 =0
⇒ 3m2 + 8m − 3 =0
P
⇒ 3m2 + 8m − 3 =0
⇒ 3m2 + 9m − m − 3 =0
⇒ (3m− 1)(m+ 3) =
0 Q

⇒m= −3, 1/ 3 R
y=-x

Equations
P(3, 2), Q(3, –2), R(2, –3), S = (–2, 3)
3x + y=
+ 7 0 or x − 3y −
= 31 0
3 2
Sol 25:
3 −2
1
Area of PQRS will be, A = 2 −3
A(1,-2) 2
−2 3
3 2
m=-1 1 1
D E = [–6–9+6–4–(6–4+6+9)] = [–13 – 17] = 15 units
m=2 2 2

(,-1-)B C ( -10 5
,
3 3 ) Sol 27:
(,2-4)
x+2y=0 x-y+5=0
l3
l4
y+2 (0,1)
Eq. of AB = = –1
x −1 A
(1,0)
y+2=1–x (-1,0)
(0,0)
B
C
x+y+1=0 D l1
y+2 l2
Eq. of AC = =2
x −1
y – 1 = mx → 1
y + 2 = 2x – 2 ⇒ y + 4 = 2x
y = m(x + 1) →  2
 α + 1 −α − 3   β + 1 2β − 6 
D→  , ; E→  ,  −1x
 α α   2 2  y= → 3
m
α +1  α +3 −1(x − 1)
+  +5 = 0 y= → 4
2  2  m
−x
α + 7 = 0 ⇒ α = –7 1 intersection  3 → mx + 1=
m
−m 1
β +1  2β − 6  x= ,y=
= −2   1+m 2
1 + m2
2  2 
11 −(x − 1)
β + 1 = –4β + 12; β= 1 intersection  4 → mx +1 =
5 m
 11 2  −x 1
(–7, 6)  ,  mx + 1 = +
m m
 5 5
y −6 28 −14  m2 + 1  1−m
= =  x =
x+7 −46 23  m  m
23y + 14x = 138 – 98 1−m
x=
14x + 23y = 40 1 + m2
8 . 7 2 | Straight Line

m − m2 m+1 5
⇒ x1 + x2 + x3 = … (iii)
y= +1 =
1 + m2 m2 + 1 8
⇒ y1 + y 2 + y 3 = … (iv)
 1−m m+1 
B 2 ,  From (i), (ii), (iii) and (iv), we get
 m + 1 m2 + 1 
−x ⇒ 4 ( x 2 + x3 ) − 3 ( y 2 + y 3 ) + 6 =0
2 intersect 3 → m(x + 1) =
m ⇒ 4 (5 − x1 ) − 3 ( 8 − y1 ) + 6 =0
x
mx + = –m
m ⇒ 20 − 4x1 − 24 + 3y1 + 6 =0
2
−m
x= ⇒ −4x1 + 3y1 + 2 =0 … (v)
1 + m2
 −m2 Now AH ⊥ BC
m m 
y= C  , 
1 + m2 2 2 y1 − 2 4
 1 + m 1 + m  × =−1
x1 − 1 3
−1
2 intercept  4 ⇒ m(x + 1) = (x − 1)
m ⇒ 4y1 − 8y =
−3x1 + 3
x 1
⇒ mx + = –m ⇒ 3x1 − 4y1 =
11
m m … (vi)
(1 − m2 ) From (v) and (vi), we get
x=
2
1+m  41 38 
(x
( x1,1, yy11)) =  , 
 2   25 25 
y = m
2 
1 +m  2 2
 41   38 
 (1 − m2 ) 2m  Radius = AO= 2 −  + 3 − 
 25   25 
⇒ D , 
2 2
 1 + m m + 1 
2 58 m
 m−1  = =
Area of square =   =
(m − 1)2 ( 25) n
 1 + m2  m2 + 1
AB = BD
⇒ m 58, =
= n 25 ⇒ m + n =83
m2 + m4 = 2m(1 + m2) Sol 29: (–6, 1) (6, 10), (9, 6), (–3, –3)
m2 = 2m
(6, 10)
m=2 B

1 p
Area = =
5 q A
C
(9, 6)
(-6, 1)
p+q=6 E
(1,0)
2 1
Sol 28: P(2,3)
D
H(1,2) C (-3,-3)

5 3 Area of rectangle – Area of ∆ =a / b


A
Centroid (c) =  , 
3 3 Area of rect = 81 + 144 16 + 9 = 15. 5 = 75
Let coordinates of vertices of y +3 3
∆ABC be A (x, y), B(x2, y2), C (x3, y3) Eq. of CD =
B C x+3 4
4x-3y+3=0
4x2 - 3y2 + 3=0 … (i) Point E = (1, 0)
4x3 - 3y3 + 3=0 … (ii) y +3 −4
Eq. of AD = =
x1 + x2 + x3 5 x+3 3
and =
3 3 −9 −21
− 3 = x; x=
4 4
M a them a ti cs | 8.73

α – 2 = 2α + 3 ⇒ α = –5
1 1  25 
[x1 − x2 ] + 3 = Area of ∆ =   × 3 = 75
2 2 4  8 and –8β + 8α = 5β + 10α + 10

75 × 7 525 a −13β − 10
Area = = = α= ,β=0
8 8 b 2
β(–5, +10)C, (0, 2)
a + b = 533
−8
m(BC) =
5
Sol 30:
y − 2 −8
A(1,2) =
x 5
y intercept = 2
L1=0 L2=0
(iii) L1 = x + y – 1 = 0

(,2)B C(,3-) L2 = 2x – y + 4 = 0
L=0
A
(i) L1 = 2x – y = 0
L2 = x + y = 3 E F
S
G = (2, 3), A(1, 2) (2,1)
⇒α+β+1=6 B C
(,1 - ) D
(,2+4)
⇒ 2 + 2α + 3 – β = 9
⇒ 2α – β = 4 ⇒ 3α = 9 S(2, 1)
α = 3, β = 2 A(–1, 2)
B(3, 6)C(2, 1)  α + β 2β − α + 5 
D , 
y −1  2 2 
Eq. ⇒ =5  α −1 3 − α  β −1 
x−2 E=  , ; F=  ,β + 3
 2 2   2 
y + 5 = 5x
m=5 Now, m(SE) = 1
(ii) If H = (2, 3) α −1
−2
L1 = 2x + y = 0 / L2 = x – y + 2 = 0 ⇒ 2 =1
3−α
−1
−2 4  −2 4  2
x= y= A  , 
3 3  3 3
⇒ α – 5 = 1 – α; α = 3
B(α, –2α), C (β, β + 2) −1
m(SF) =
4 2
3−
3 5
Slope of AH = = β −1
2 8 −2
β −5
2+
3 ⇒ 2 = = –2
β + 3 −1 2(β + 2)
−8 B + 2α + 2
Slope of BC = = –4(β + 2) = β – 5
5 β−α
3 + 2α −3
Slope of BH = 5β = –3β =
2−α 5
2  −3 14 
β+ β = (3, –2), C  , 
α−2 3 =1
Slope of AC = =  5 5 
3 + 2α 2
β+ y+2 −24 y+2 −4
3 = ⇒ =
x −3 18 x −3 3
8 . 7 4 | Straight Line

3 3 2 1
⇒ x intercept 3 – = sin θ = , cos θ =
2 2 5 5
C = (6, 4)
Exercise 2 D = 0 + 2 5 cosθ, 2 + 2 5 sinθ = (2, 6)
Eq. of AC
Single Correct Choice Type
y −2 1
= ⇒ x = 3y – 6 ….(i)
Sol 1: (A) a(3x + 4y + 6) + b(x + y + 2) = 0 x−0 3
(2, 3) is situated at greater distance y −0
Eq. of BD = = –3
x−4
a(6 + 12 + 6) + b(2 + 3 + 2) y
D= x=4– ….(ii)
2 2
(3a + b) + (4a + b) 3
Solving eq. (i) & (ii) we get the required Point is (3, 3)
24a + 7b 24T + 7
D= =
(3a + b)2 + (4a + b)2 (3T + 1)2 + (4T + 1)2 Sol 3: (C)
A
 a
Where  T = 
 b 
px+qy=1 qx+py=1
dD
=0
dT
B C
⇒ (3T + 1)2 + (4T + 1)2 × 24 (p,q)

(24T + 7)2[(3T + 1)3 + 4(4T + 1)] px + qy = 1 × q


=
2 (3T + 1)2 + (4T + 1)2 qx + py = 1 × p

⇒ 24[(3T+1)2 + (4T+1)2 pqx + q2y = q

= (24T + 7)(9T + 16T + 7) pqr + p2y = p

⇒ 24[25T2 + 2 + 14T] = [24T + 7][25T + 7] ----------------

⇒ 48 + 24×14T = 24T×7+25×7T+7×7 y(q2 – p2) = (q – p)


1
⇒ 24×7T = 49 – 48 + 25×7T y=
p+q
−1 a
⇒ –7T = 1 ⇒ T = = 1
7 b x=
p+q
⇒ –(3x + 4y + 6) + 7(x + y + 2) = 0
 1 1 
A=  , 
4x + 3y + 8 = 0 p + q p + q
Median through AB
Sol 2: (C)
1
q−
y −q p+q
2
5 ⇒ =
x −p 1
p−
(0,2) A C p+q
2 5
5 2 y −q  pq + q2 − 1 
⇒ =  
B x −p 2
 p + pq − 1 
(4,0)
(p2 + pq – 1)y – q(p2 + pq – 1)
C = 4 + 2 5 cosθ, 2 5 sinθ = x(q2 + qp – 1) – p(q2 + pq – 1)
tan θ = 2 x(pq+q2–1) + p = q + y(p2 + pq – 1)
M a them a ti cs | 8.75

= (2pq – 1)(px + qy – 1) 9 7
⇒ 4β – 8 = 3α – ⇒ 3α – 4β + =0
= (p2 + q2 – 1) (qx + py – 1) 2 2

 27 36  25β − 36 4
Sol 4: (B) 3x + 4y = 9 × 4 Mid point of AC=  ,  = =
 25 25  25α − 27 3
⇒ 12x + 16y = 36 (Slope of ⊥ to AC)
4x – 3y + 12 = 0 × 3 4α
⇒ 100α – 108 = 75β – 108 ⇒β =
⇒12x – 9y = –36 3
3
25y = 72 α= β=2
2
72
y= 9 5
25 Circumradius = +4 =
4 2
4 × 72
9−
25 4 × 24 21
⇒x = = 3− = − a 1 1
3 25 25
Sol 5: (D) 1 b 1 = 0
A(3,0) 1 1 c

a(bc – 1) – 1(c – 1) + 1(1 – b) = 0


(,) abc – a – b – c + 2 = 0
(0,4)B [ 21 72
C - 25 , 25 ] a + b + c = abc + 2
1 1 1
+ +
 21 72  1−a 1−b 1−c
A(3, 0) B(0, 4]C  − 25 , 25 
  1 + bc − b − c + 1 + ab − a − b + 1 + ac − a − c
=
(1 − a)(1 − b)(1 − c)
3 0 1
1 3 + ab + bc + ca − 2a − 2b − 2c
DPAB = 0 4 1 =
2 1 − a − b − c + ab + bc + ca − abc
21 72
− 0
25 25 3 − 2(abc + 2) + ab + bc + ca
=
1 − abc − (abc + 2) + ab + bc + ca
1  72   84  
= 3 4 −  + 10 +  −1 − 2abc + ab + bc + ca
2  25   25   = =1
−1 − 2abc + ab + bc + ca
1  3 × 28 84  84
=  +  = 25
2  25 25 
Sol 6: (D) A(a, 0) ; B(0, b) ; C (c, 0) ; D(0, d)
72
−72 −3 d c
25 = lines are not parallel i. e. not a trapezium, not
mAC = = = a b
21 96 4
− −3
25 a parallelogram

72 OA OC = OB OD (For concyclic points)


4−
mBC = 25 = 24 = 8
Assuming origin as centre
21 21 7
25 ac = bd i. e. ABCD are concyclic.
4
mAB = −
3 Sol 7: (D) 7x2 + 8y2 – 4xy + 2x – 4y – 8 = 0
3  β−2 3 Homogenising
Mid point of AB  ,2 ⇒ = (Slope of ⊥ AB)
2  3 4  3x − y 
α−
2 7x2 + 8y2 – 4xy + 2x  
 2 
8 . 7 6 | Straight Line

 3x − y   3x − y 
2 12x2 + y2(3k2 – 2k – 6) + xy(6k + 4k + 10) = 0
– 4  y – 8  =0
 2   2  −2h −(10k + 10)
m 1 + m2 = = =0 ⇒ k = –1
7x2 + 8y2 – 4xy + 3x2 – xy – 6xy b 3k 2 − 2k − 6

+ 2y2 – 2(9x2 + y2 – 6xy) = 0 Sol 10: (D) R = (h, k)


⇒ –8x2 + 8y2 – 11xy + 12xy = 0
h+6 k −6
⇒ 8x2 – 8y2 – xy = 0 Centroid is  , 
 3 3 
2 h2 − ab
⇒ tan θ = =∞ h+6 k −6
a+b 9  + 7  +4=0
 3   3 
1
⇒ h = − a = 8b = –8
2 9h + 7k + 54 – 42 + 12 = 0
π 9h + 7k + 24 = 0
⇒θ=
2
parallel to N.
Sol 8: (B) x2 – 4xy + 4y2 + x – 2y – 6 = 0
Sol 11: (C) A(1, 2) & B(7, 5)
y = mx + C1y = mx + C2
3 1
(y – mx – a)(y – mx – C2) = 0 AB line ⇒m = =
6 2
y2 – mxy – C2y – mxy + m2x2
2 1
+ C2mx – C2y + C1mx + C1C2 = 0
A 2 C 1B
m x + y – 2mxy + x (C1m + C2m)
2 2 2
A C (7,5)B
(1,2)
+ y(–C1 – C2) + C1C2 = 0 (1,2) (7,5)

1 2 1  14 + 1 10 + 2 
=4⇒ − = –4 ⇒ m = + C=  ,  ⇒ (5,4)
m 2  3 3 
m2
2 2m − 1
(C1 + C2) + =1 = ±1
a m+2
1 2m – 1 = –2 – m
(C1 – C2) =
2 1 1
m= − or m = 3(m = − is not possible)
6 3 3 3
C 1C 2 = − = −
4 2 ⇒ y = 3x – 11 ⇒ 3x – y – 11 = 0
C1 − C2
Distance between lines =
1 + m2 Sol 12: (A) ax2 + 2hxy + by2 = 0
1
C 1 + C2 =
2 m2
−6
C 1C 2 =
4 m
1
+6
C1 − C2 4 5
= = = 5
1 + m2 1 5 m1
1+ 2
4 4
m − m1 m2 − m
=
1 + mm1 1 + m2m
Sol 9: (B) 5x2 + 12xy – 6y2 + 4x(x + ky) [Homogenising]
⇒ m + m 2m 2 – m 1 – m 1m 2m
– ay(x + ky) + 3(x + ky)2 = 0
= m2 – m + mm1m2 – m2m1
9x2 – 6y2 – 2ky2 + 10xy + 4kxy + 3(x2 + k2y2 + 2kxy) = 0
⇒ 2m – (m1 + m2) + (m1 + m2)m2 = 2m1m2m
M a them a ti cs | 8.77

2h 2h 2 2a mAB. mBC = –1
⇒ 2m + – m = m
b b b 9 − 2x
mAB =
⇒ 2mb – 2am = –2h + 2hm 2 x−4

hm2 + m(a – b) – h = 0 13 − 2x
mBC =
x−2
Sol 13: (D) (9 – 2x)(13 – x) = –1(x2 – 6x + 8)
3x-y=-3 117 – 18x – 26x + 4x2 = –x2 + 6x – 8
5x2 – 50x + 125 = 0
(0,3)
2x-3y = 6 x2 – 10x + 25 = 0 ⇒ x = 5; y = 0
1 1
Area = × BC × AB = 9+9 1+1 = 3
(-1,0)
2 2
(3,0)

(0,-2)
3x+4y = 12 Multiple Correct Choice Type

Sol 16: (B, D) Let the co-ordinate of the vertex C is (h,


From figure h – 2) then area of triangle will be
α ∈ [–1, 3] −5 0 1
1
β ∈ [–2, 3] ⇒ 3 0 1 =
20
2
h h− 2 1
Sol 14: (C)
⇒ (h – 2)8 = ± 40
A
⇒ h – 2 = ±5

2x+3y=29 x+2y=16 ⇒ h = 7 or –3
∴ Co-ordinate are (7, 5) or (–3, –5)

(,)B C(10-,12-)
(5, 6)
Sol 17: (B, C) y + 3x = 2
2α + 3β = 29
y– 3x = 2
10 – α + 24 – 2β = 16
x = 0, y = 2
α + 2β = 18  …(i)
2α + 4β = 36  …(ii) -1/3 A
m
4/3
On solving, we get 1/3 60
β = 7, α = 4 2/3
So we have, B(4, 7) A
y −7
Eq. of BC = = –1 ⇒ x + y = 11 2/3
x−4

Sol 15: (B) 1 1


−m m+
3 3 1 − 3m 3m + 1
A(4,1) = ⇒ =
m m m+ 3 3 −m
1+ 1−
3 3

B
C(2,-3) ⇒ 3 – m –3m+m2 3 =m2/ 3 +m+3m+ 3
(x,10-2x)
2x+y=0
8 . 7 8 | Straight Line

⇒m=0
B(0, b)
Bisector line is y = 2
(, )
P 2
⇒ cos 60º = ; P= A’(-a’, 0) A(0, a)
4/ 3 3

 2  B’(0, b’)
⇒ Given point is  ,2 
 3  −b
m(AB) =
a
Or the other possibility is m = ∞
b
⇒ Foot of perpendicular = (0, 0) m(BA′) =
a′
Let orthocentre be at (α, β)
Sol 18: (A, B, D)
A(1,3) b−β
=∞⇒α=0
0−α

β−0 a
P(x,y) =
α + a′ b
(5,6)B C(-1,2)
aa′
β= = b′ [From (i)]
y −6 3 b
AB - =
x −5 4
Sol 20: (A, B, C, D)
4y – 24 = 3x – 15 ⇒ 3x – 4y + 9 = 0
y −6 2 C(0,a)
BC - =
x −5 3 B(x,x/3)
60
3y – 18 = 2x – 10 ⇒ 2x – 3y + 8 = 0 30
y −3 (0,0) -30
1
AC - = x2 + x2/3 = a2
x −1 2
2y – 6 = x – 1 ⇒ x – 2y + 5 = 0 (0,-a) x=3a/2

C and P on same side C either lies on y axis so C(0,a) or C(0,–a)


3(–1) – 4(2) + 5 < 0 3x – 4y + 9 < 0 1
or on y = −
B and P on same side of AC 3x
5 – 2(6) + 5 < 0 x – 2y + 5 < 0  −1 
C  x, 
 3x 
A and P on same side of BC
2(1) – 3(3) + 8 > 0 2x – 3y + 8 > 0 x2
⇒x2 + = a2
3
So we can see [A,B,D] are correct.
±a 3
x=
x y 2
Sol 19: (B, C) + = 1A (a, 0) & B(0, b)
a b a
y= 
x y 2
line + = 1 C(–a′, 0) D(0, –b′)
a′ b′  3a −a   − 3a a 
⇒ (0, a), (0, –a),  ,   , 
AB A′B′ are concyclic  2 2   2 2
   
i. e. OAOA′ = OBOB′
aa′ = bb′
aa′
b′ =
b
M a them a ti cs | 8.79

Sol 21: (B, C) Vertices are (4, 3)(1, –1) & (7, k) Sol 23: (B, C) (x + 2) (y + 2) = 0; x + y + 2 = 0

A(4,3) Circumcentre lies on mid-point of hypotenuse i. e. AB


( −2,0) + (0, −2)
= (–1, –1)
2

x=-2 y
(1,-1)B C(7,k)

 2+k  A
C =  4,  x
 3 
That occurs only in isosceles ∆ B C y=-2
(1 – 4)2 + (3 + 1)2 = (k – 3)2 + 9
y=-x-2
AC = AB
Sol 24: (B, C) x + y = 1; x = 7y; x = 3y
16 = (k – 3)2
Centroid and In centre always Lie inside of the triangle.
⇒ k – 3 = ±4
⇒ k = 7, –1 or (7/8, 1/8)
25 = (k + 1) + 36
2

≠ not possible (AB = BC) (3/4, 1/4)


or (k – 3) + 9 = (k + 1) + 36
2 2

(0,0)
(BC = AC)
Sol 25: (B, C)
18 – 6k = 37 + 2k
y+2x=5
−19
⇒ 8k = –19 ⇒ k = m
8
For k = 7AB = 5
AC = 5, BC = 10
[∆ is not possible] m − ( −2)
tan θ =
1 + m( −2)
−19
So k = –1 or m+2 1
8 =±
1 − 2m 2
Sol 22: (A, C) (7, 4) 2m + 4 = ±(1 – 2m)
y – 4 = m(x – 7) ⇒ 2m + 4 = 2m – 1 ⇒(m = ∞)
Centre ≡ (3, –2) ⇒ x = 2 (B)
(3 − 7)m + 4 + 2 2m + 4 = 1 – 2m
⇒ =4
1 + m2 3
4m = –3 ⇒ m = −
⇒ (–4m + 6) = 16(1 + m )
2 2
4
⇒ 9 + 4m2 – 12m = 4 + 4m2 y −3 −3
=
x−2 4
5
⇒ m= or m = ∞
12 3x + 4y = 18 (C)
5
⇒ y–4= (x – 7)
12
⇒5x – 12y + 13 = 0 and x = 7
8 . 8 0 | Straight Line

Sol 26: (A, C)


2 2
=
   5
y
y=-2x
(1,0) y=x
45
/4
/6 x’ x
O
L1 /2 L2

/2
y – a = m(x – 1) L3 P R Q
y=-2
(-2,-2) (1,-2)
y = m(x – 1)
y’
− sin75º
m = – tan 75º =
cos75º
Sol 2: (A) Solving equations L1 and L2.
−( 3 + 1)
= = – (2 + 3) x y 1
( 3 − 1) ⇒ = =
–36 + 10 –25 + 12 2 – 15
y = – (2 + 3) (x – 1) ∴ x = 2, y = 1
(2 + 3) x + y = (2 + 3) (C) L1, L2, L3 are concurrent if point (2, 1) lies on L2

x + (2 − 3) y = 1 ∴ 6 – k – 1= 0 ⇒ k = 5
(A) → (S)
Sol 27: (B, C) y – y1 = m(x – x1) (B) Either L1 is parallel to L2 or L3 is parallel to L2, then
y = y1 + m(x – x1) 1 3 3 –k
= or =
(B) Set of parallel lines 3 –k 5 2
–6
(C) All these lines pass through x = x1 ⇒ k = – 9 or k =
5
(B) → (p, q)
(C) L1, L2, L3 form a triangle, if they are not concurrent,
or not parallel.
6 5
∴ k ≠ 5, – 9, – ⇒k=
5 6
(c) → (r)
(D) L1, L2, L3 do not form a triangle. If
6
Previous Years’ Questions k = 5, – 9, –
5
(D) → (p, q, s)
Sol 1: (C) It is not necessary that the bisector of an
angle will divide the triangle into two similar triangle, Sol 3: (A, C) Given lines px + qy + r = 0, qx + ry + p = 0
therefore, statement-II is false.
Now we verify statement-I and rx + py + q = 0 are concurrent.

∆ OPQ, OR is the internal bisector of ∠POQ p q r


∴ q r p =0
PR OP
∴ = r p q
RQ OQ

PR 22 + 22 Applying R1 → R1 + R2 + R3 and taking common from R1


⇒ =
RQ 12 + 22 1 1 1
⇒ (p + q + r) q r p
r p q
M a them a ti cs | 8.81

⇒ (p + q + r) (p2 + q2 + r2 – pq – qr – pr) = 0 Given plane is


⇒ p3 + q3 + r3 – 3pqr = 0. x – 2y +z = d  …….(ii)
Equations (i) and (ii) are parallel.
Sol 4: (A, C) Since, 3x + 2y ≥ 0 ….(i)
Clearly, A = 1
Where (1, 3) (5, 0) and (– 1, 2) satisfy Equation (i)
Now, distance between plane
∴ Option (a) is true.

Again 2x + y – 13 ≥ 0 d
= = 6
is not satisfied by (1, 3), 1+ 4 +1
⇒ |d| = 6
∴ Option (b) is false.
2x – 3y – 12 ≤ 0, Sol 7: x2 + y2 ≤ 6 and 2x – 3y = 1 is shown as
is satisfied for all points,
∴ Option (c) is true.
L
And – 2x + y ≥ 0,
is not satisfied by (5, 0), 1/2

∴ Option (d) is false, 1/3

Sol 5: (B, C) Let equation of line L1 be y =mx. Intercepts


For the point to lie in the shaded part, origin and the
made by L1 and L2 on the circle will be equal ie, L1 and L2
point lie on opposite side of straight line L.
are at the same distance from the centre of the circle.
∴ For any point in shaded part L > 0 and for any point
Centre of the given circle is (1/2, – 3/2). Therefore, inside the circle S < 0.
m 3  3
+ Now, for
|1 / 2–3/ 2–1|
= 2 2  2,  , L : 2x – 3y – 1
 4
1+1 m2 + 1 9 3
L : 4 – – 1= >0
4 4
2 |m+3|
⇒ = 9
2 2 and S : x2 + y 2 – 6, S : 4 + –6<0
2 m +1 16
⇒ 8m + 8 = m2 + 6m + 9
2  3
⇒  2,  lies in shaded part.
⇒ 7m2 – 6m – 1 = 0  4
5 3
⇒ (7m + 1) (m – 1) = 0 For  ,  L : 5 – 9 – 1 < 0 (neglect)
2 4
–1
⇒=m = ,m 1 1 1 1 3
7 For  ,–  L : + – 1 > 0
Thus, two chords are x + 7y = 0 and y – x = 0 Therefore, 4 4 2 4
(b) and (c) are correct answers. 1 1
∴  –  lies in the shaded part.
4 4
Sol 6: Equation of plane containing the given lines is 1 1  1 3
For  ,  L : – – 1 < 0 (neglect)
x –1 y –2 z –3 8 4 4 4
2 3 4 =0
⇒ Only 2 points lie in the shaded part.
3 4 5

⇒ (x – 1)(– 1) – (y – 2)(– 2) + (z – 3)(– 1) = 0 Sol 8: Note: d : (P, Q) = | x1 – x2 | + | y1 – y2 |. It is new


method representing distance between two points
⇒ – x +1 + 2y – 4 – z + 3 = 0
P and Q and in future very important in coordinate
⇒ x + 2y – z = 0  ……(i) geometry.
8 . 8 2 | Straight Line

Now, let P(x, y) be any point in the first quadrant. We  1 m 


have P≡ , 
m+1 m+1
d(P, 0) = | x – 0 | + | y – 0 | = | x | + | y | = x + y ( x,
y > given)  3 3m 
Q≡ , 
 m + 1 m +1
d (P, A) = |X – 3| + |Y – 2| (given)
y
d(P, 0) = D(P, A)
⇒ x + y = |x – 3| + |y – 2| L1
Case I : When 0 < x < 3, 0 < y < 2 R Q
In this case I Eq.(i) becomes L2 L

x + y= 3 – x + 2 – y P
x
O
5
⇒ 2x + 2y = 5 ⇒x+y=
2 x+y=1 x+y=3

Case II : When 0 < x < 3, y ≥ 2


m–2
Now, Eq. (i) becomes Now, equation of L1 : y – 2x =  …. (i)
m+1
x + y = 3 – x + y – 2 3m + 9
And equation of L2 : y + 3x =  ….(ii)
1 m+1
⇒ 2x = 1 ⇒x=
2 By eliminating ‘m’ from Equ. (i) and (ii), we get locus of
R as x – 3y + 5 = 0, which represents a straight line.
Case III : When x ≥ 3, 0 < y < 2

Infinite segment
Sol 10: Let the square S is to be bounded by the lines x
y 1 1
x=1/2 =± and y = ±
2 2
(1/2,2)

Finite segment

y=2
2 2 2
x+y=5/2,

 1 1 
We have,
= a2  x1 –  +  – y1 
 2 2 
1
y

x A(x1,1/2) 1/2
O 1/2 (5/2, 0) 3
D(-1/2,y2) d
a
1/2
Now, Eq.(i) becomes x + y = x – 3 + 2 – y x’ O
x
-1/2 B(1/2, y1)
c
1
⇒ 2y = – 1 or y= – b
2 C(x2-1/2) -1/2
Hence, no solution. y’
Case IV : When x ≥ 3, y ≥ 2
1
= x12 – y12 – x1 – y1 +
In this case (i) changes to x + y = x – 3 + y – 2 2
⇒0=–5 1
Similarly,
= b2 x22 – y12 – x2 + y1 +
2
Which is not possible.
1
Hence, this solution set is {(x, y)} | x = 12, y ≥ 2} ∪ {(x, y)} | c2 = x22 – y 22 + x2 + y 2 +
2
x + y = 5/2, 0 < x < 3, 0 < y > 2 } 1
d2 = x12 – y 22 + x1 + y 2 +
The graph is given in adjoining figure. 2

∴ a2 + b2 + c2 + d2= 2(x12 + y12 + x22 + y 22 ) + 2


Sol 9: Let the equation of straight line L be
1
y = mx Therefore 0 ≤ x12 , x22 , y12 , y 22 ≤
4
M a them a ti cs | 8.83

0 ≤ x12 + x22 + y12 + y 22 ≤ 1 Sol 11: Let the vertices of a triangle be, O(0, 0) A(a, 0)
and B(b, c) equation of altitude BD is x =b.
⇒ 0 ≤ 2(x12 + x22 + y12 + y 22 ) ≤ 2
c
Slope of OB is .
But 2 ≤ 2(x12 + x22 + y12 + y 22 ) + 2 ≤4 b
b
Alternate Solution Slope of AF is – .
c
2
c= x22 + y 22  ….(i) Now, the equation of altitude AF is
y
y
B(b,c)
A(x1,1)
a
d B(1,y1) F 90
90 E
P
(0,y2)D
b x’ x
c O D(b,0) A(a,0)
x’ x y’
O C(x2,0)
b
y – 0 = – (x – a)
y’ c

=b2 (1 – x2 )2 + y12  …. (ii) Suppose, BD and OE intersect at P.

=a2 (1 – y1 )2 + (1 – x1 )2  …..(iii)   a – b 
Coordinates of P are b,b  
 c 
2
d= x12 + (1 – y 2 ) 2
.....(iv) 
On adding Eqs. (i), (ii), (iii) and (iv), we get a–b
Let m1 be the slope of OP =
2 2
a +b +c +d = 2 2
{x12 + (1 – x1 ) } 2
+ {y12 2
+ (1 – y1 ) } c

+ {x22 + (1 – x2 )2 } + {y 22 + (1 – y 2 )2 } c
and m2 be the slope of AB =
b–a
Where x1, y1, x2, y2 all vary in the interval [0, 1].
 a – b  c 
Now, consider the function y = x2 + (1 – x)2, 0 ≤ x ≤ 1 Now, m1m2 =
=   –1
 c  b – a 
dy
Differentiating ⇒ ⇒ 2x – 2(1 – x) . For maximum
= We get that the line through O and P is perpendicular
dx
dy to AB.
or minimum =0.
dx
⇒ 2x – 2(1 – x ) = 0 ⇒ 2x – 2 + 2x = 0 Sol 12: Since, line L make 60° with line 3 x + y =
1, then
1
⇒ 4x = 2 ⇒x=
2
d2 y
Again, =2+2 = 4
dx2
⇒ Which is positive.
1
Hence, y is minimum at x = and its minimum value
1 2
is . − 3 + tan 60°
= m1 = 0
4
Clearly, value is maximum when x = 1.
1 − − 3 ( tan 60° ) ( )
− 3 − tan 60° −2 3 −2 3
1 1 1 1 m2 = = = = 3
∴ Minimum value of a2 + b2 + c2 + d2 = + + + =2
2 2 2 2 ( )
1 + − 3 tan 60° 1−3 −2

And maximum value is 1 + 1 + 1 + 1 = 4


Equation of line having slope 3 and passes through
(3, -2)
8 . 8 4 | Straight Line

y +=
2 3 ( x − 3) α −β α+β
⇒2≤ + ≤4
⇒ y + 2= 3x − 3 3 2 2
⇒ y − 3x + 2 + 3 3 =0

Sol 13: (A) The point of intersection of lines ax + by + c


= 0 and bx + ay + c = 0 is
 −c −c 
P , 
a+b a+b
Given that distance of point P from (1, 1) is less than
2 2
2 2
 c   c 
1 +  + 1 +  <2 2
 a+b  a+b
Since, Point P lies in the first quadrant α ; β > 0
a+b+c
⇒ 2 <2 2 Case 1: α≥β
a+b
α −β α+β
a+b+c a+b+c 2≤ + ≤4
⇒ < 2 ⇒ −2 < <2 2 2
a+b a+b

⇒ a + b + c < 2a + 2b ⇒ a + b − c > 0 ⇒ 2≤α≤2 2

Case 2: α<β
Sol 14: Let P be ( α , β ) , then
−α + β α+β
α −β 2≤ + ≤4⇒ 2 ≤β≤2 2
d1 (P ) = 2 2
2
( ) − ( 2) = 8 – 2
2 2
α+β Area = 2 2
d2 (P ) =
2 = 6 sq. unit

⇒ 2 ≤ d1 (P ) + d2 (P ) ≤ 4

You might also like